Sie sind auf Seite 1von 150

MULTIPLE CHOICE QUESTIONS-1

1. A 60-year-old woman presented with a small right pupil, right ptosis and impaired sweating
over the ipsilateral forehead. Sweating on the rest of the face was unaffected. Where is the most
likely site of this lesion?
A) Cervical spinal cord
B) Common carotid artery
C) Hypothalamus
D) Internal carotid artery
E) Lateral medulla

Answer: D

2. A 53-year-old man presented with hypertension of 150/110 mmHg. He is generally


asymptomatic and has no previous medical history of note. He is a smoker of 5 cigarettes daily
and drinks modest quantities of alcohol. He takes no prescribed medications. Examination
reveals a BMI of 33.5 kg/cm2 but nil else. Investigations:
Serum sodium 146 mmol/l (NR 133-145); Serum potassium 3.2 mmol/l (NR 3.5 - 5); Urinary
potassium excretion 42 mmol/l (NR less than 30)
What is the likely diagnosis?
A) adrenocortical adenoma
B) Bartter's syndrome
C) Liddle's syndrome
D) liquorice ingestion
E) pheochromocytoma

Answer: A

3. A previously healthy 54-year-old man presents to the emergency department complaining of


chest pain. His ECG shows an acute inferior wall myocardial infarction. His blood pressure is
90/60 mm Hg. On physical examination, he has jugular vein distention and clear lungs. You
should treat him immediately with which of the following:
A) Intravenous fluids.
B) Norepinephrine.
C) Dopamine.
D) Nesiritide.
E) Nitroprusside.

Answer: A

4. While caring for a patient with suspected pericardial tamponade, you would expect to find:
A) Equalization of right and left ventricular pressures.
B) Isolated systolic hypertension.
C) A hyperdynamic precordium.
D) Poor R wave progression on the 12-lead electrocardiogram.
E) Mitral regurgitation.
Answer: A

5. A female patient aged 30 has a 5 years history of difficulty getting upstairs and out of a low
chair and mild upper limb weakness but no pain. There is no family history. She presented with
severe type 2 respiratory failure. EMG showed evidence of myopathy.
The most likely diagnosis is:
A) Polymyositis
B) Inclusion body myositis
C) Acid maltase deficiency
D) Miller-Fisher Syndrome
E) Lambert-Eaton Myasthenic syndrome

Answer: C

6. A teenage girl presents with Guillain-Barre syndrome. Her weakness continues to worsen after
admission to hospital. Which of the following should be used to monitor her?
A) Arterial blood gases
B) Chest expansion size
C) FEV1/FVC ratio
D) PEFR
E) Vital capacity

Answer: E

7. A 19-year-old woman presents to the clinic having had 5 blackouts over the last year, all while
she is standing up. She gets warnings of blurred vision, nausea, feeling hot. She had been
witnessed twice to have jerking of all limbs while she is unconscious. The attacks last 30-60
seconds. She recovers quickly after the attacks. She has never bitten her tongue or sustained any
injuries. Physical examination and an ECG are normal. Her grandmother and sister suffer from
epilepsy. Which of the following investigations is the most appropriate?
A) EEG
B) 24 hour ECG recording
C) CT brain
D) ECHO
E) Tilt table test

Answer: E

8. A 21-year-old female presented with a sudden onset of left sided head and neck pain. 24 hours
later she presents with sudden onset of right hemiparesis, facial weakness and homonymous
hemianopia and left Horner's syndrome. A CT brain showed a left middle cerebral artery
territory infarction. The most likely diagnosis is:
A) Cardiac embolism
B) Migrane
C) Left carotid artery dissection
D) Antiphospholipid syndrome
E) Systemic vasculitis

Answer: C

9. A female presents with headache, lethargy and weight loss. Which of the following would
make the diagnosis of giant cell arteritis unlikely?
A) A normal ESR
B) Bilateral headache
C) Non-tender temporal arteries
D) Papilloedema on fundoscopy
E) The patient is 50 years old

Answer: D

10. A 60-year-old man presents with a 2 month history of progressive confusion, gait disturbance,
and urinary incontinence. Examination reveals mild global cognitive dysfunction and gait ataxia.
CT brain shows enlarged ventricles with no evidence of obstruction to CSF outflow. Lumbar
puncture reveals normal CSF pressure and constituents. Which one of the following management
steps is likely to be most helpful?
A) CSF drainage via repeated lumbar puncture
B) EEG
C) Intracranial pressure monitoring
D) MRI brainstem
E) Serum B12 and folate levels

Answer: A

11. A 72-year-old man presents with an acutely painful right knee. On examination, he had a
temperature of 37°C with a hot, swollen right knee. Of relevance amongst his investigations, was
his white cell count which was 12.6 x109/l and a knee X-ray revealed reduced joint space and
calcification of the articular cartilage. Culture of aspirated fluid revealed no growth. What is the
most likely diagnosis?
A) Gout
B) Psoriatic monoarthropathy
C) Pseudogout
D) Rheumatoid arthritis
E) Septic arthritis

Answer: C

12. A 40-year-old female who has been prescribed thyroid replacement therapy has routine
thyroid function tests. On examination she appeared clinically euthyroid with no abnormal
findings. Her TFTs revealed: TSH 3.2 mU/L (0. 35 - 5.0) Total T4 20 nmol/L (55 - 144) free T4
2.6 pmol/L (9 - 24) Total T3 2.5 nmol/L (0. 9 - 2.8).
Which one of the following statements is correct?
A) Her thyroid hormone replacement is adequate.
B) Investigation of pituitary function is required.
C) She has tertiary hypothyroidism.
D) She has thyroiditis.
E) She has sick euthyroid syndrome.

Answer: A

13. Which of the following viral causes of acute hepatitis is most likely to cause fulminant
hepatitis in a pregnant woman?
A) Hepatitis A
B) Hepatitis B
C) Hepatitis C
D) Hepatitis D
E) Hepatitis E

Answer: E

14. A 35-year-old healthy woman has a faint systolic murmur on physical examination. An
echocardiogram is performed, and she is found to have a bicuspid aortic valve. In explaining the
meaning of this finding to her, the most appropriate statement is that?
A) An aortic valve replacement is eventually likely to be required
B) Other family members are likely to have the same condition
C) She should be treated with a cholesterol lowering agent
D) The problem resulted from past injection drug usage
E) This is one manifestation of an underlying autoimmune condition.

Answer: A

15. A 54 year old man presents with central crushing chest pain. Examination is normal. 12-lead
ECG shows ST segment elevation in leads II, III, aVF and ST depression in V1, V2 and V3.
Which coronary artery is occluded?
A) Circumflex.
B) Left anterior descending.
C) Left main stem.
D) Obtuse marginal.
E) Right coronary artery.

Answer: B

16. Primary hyperparathyroidism may occur in association with the following conditions
A) Chronic renal failure
B) Vitamin D deficiency
C) Gastrinoma
D) Autoimmune polyendocrine syndrome
E) Sjogren's syndrome
Answer: C

17. A 57-year-old man with a history of diabetes mellitus and chronic kidney disease with a
baseline creatinine of 1.8 mg/dL undergoes cardiac catheterization for acute myocardial
infarction. He is subsequently diagnosed with acute kidney injury related to iodinated contrast.
All of the following statements are true regarding his kidney injury EXCEPT:
A) Fractional excretion of sodium will be low.
B) His creatinine is likely to peak within 3–5 days.
C) His diabetes mellitus predisposed him to develop contrast nephropathy.
D) Transient tubule obstruction with precipitated iodinated contrast contributed to the
development of his acute kidney injury.
E) White blood cell casts are likely on microscopic examination of urinary sediment.

Answer: E

18. A 25 year-old male presents with an eight week history of difficulty walking. On
examination he had increased tone and pyramidal weakness of the right leg. There was
impairment of pinprick sensation in the left leg up to the groin. Which one of the following is the
cause of these signs?
A) A central cauda equina lesion.
B) A cervical spinal cord lesion.
C) A lesion at the foramen magnum.
D) A right-sided thoracic spinal cord lesion.
E) Bilateral cerebral hemisphere lesions.

Answer: D

19. An 81 -year-old man admitted with a stroke becomes increasingly drowsy after receiving
nasogastric feeding for five days. Which biochemical abnormality is the most likely cause of his
drowsiness?
A) Hyperglycemia.
B) Hypermagnesemia.
C) Hypernatremia.
D) Hypocalcemia.
E) Hypophosphatemia

Answer: C

20. One of the nurses working on the Care of the Elderly ward sustains a needlestick injury while
taking blood from a patient. What is the most appropriate immediate management?
A) Administer prophylactic hepatitis B immunoglobulin regardless of vaccine status.
B) Exclude the nurse from performing exposure-prone procedures for three months until a
negative HIV antibody test has been obtained.
C) Immediately take the nurse’s blood to test for antibodies to hepatitis B, hepatitis C and human
immunodeficiency viruses.
D) Promptly administration of antiretroviral therapy.
E) Wash the wound with soap under running water.

Answer: E

21. A 65-year-old male with left ventricular systolic dysfunction was dyspneic on climbing stairs
but not at rest. The patient was commenced on Ramipril and Furosemide.
Which one of the following drugs would improve the patient’s prognosis further?
A) Amiodarone
B) Digoxin
C) Diltiazem
D) Metoprolol
E) Isosorbide dinitrate.

Answer: D

22. All of the following are recognized feature of pulmonary embolism except?
A) S1Q3T3
B) An increase in serum troponin levels
C) An arterial pH greater than 7.2
D) Increased PCO2 on room airk,m
E) Positive D-dimer levels

Answer: D

23. Which of the following concerning diabetic retinopathy is correct?


A) Is unusual in Type 2 diabetic patients.
B) Improved glycemic control is more effective than hypertensive control in reducing
progression of disease.
C) Normal visual acuity is seen in proliferative retinopathy.
D) Progression may be reduced with statin therapy.
E) Soft exudates are a feature of background retinopathy.

Answer: C

24. A 52 year old female presents with tiredness. There are no specific abnormalities noted on
examination, but investigations reveal a T4 of 21.1 (NR 9.8 - 23), a T3 of 5.2 pmol/l (NR 3.3 -
5.5) and a TSH of 0.05 mU/l (NR 0.1 - 5 mU/l). Thyroid autoantibody titers are all undetectable.
These results suggest a diagnosis of:
A) De Quervain's thyroiditis.
B) Sick euthyroid syndrome.
C) Solitary toxic nodule.
D) Grave's disease.
E) Hashimoto's thyroiditis.

Answer: C
25. A 46-year-old woman with a 10-year history of primary progressive multiple sclerosis is
admitted to the hospital for surgical debridement and wound management of a sacral pressure
ulcer. The patient is bedbound and is cared for at home by her husband. Her medications include
sertraline, baclofen, and oxybutynin. The remainder of the medical history is noncontributory.
On physical examination, she is thin, in no distress, and has normal vital signs. There are
contraction deformities of the lower extremities, and a 6- × 8-cm sacral ulcer that extends to the
fascia, with minimal purulent exudates and no evidence of cellulitis. Laboratory studies include a
hematocrit of 34%, leukocyte count of 15,000/µL (15 × 109/L) with 80% neutrophils, and a
platelet count of 425,000/µL (425 × 109/L). Subcutaneous prophylactic unfractionated heparin is
administered. Six days after initiation of heparin, her platelet count decreases to 210,000/µL (210
× 109/L), and on the ninth day after therapy, the platelet count has decreased to 95,000/µL (95 ×
109/L). Which of the following is the most appropriate next step in the management of this
patient?
A) Discontinue unfractionated heparin.
B) Discontinue sertraline, baclofen, and oxybutynin.
C) Discontinue unfractionated heparin and begin therapy with a direct thrombin inhibitor.
D) Discontinue unfractionated heparin and begin low-molecular-weight heparin.
E) Discontinue unfractionated heparin and begin novel oral anticoagulants.

Answer: C

26. A 75 year old man has a history of Chronic Lymphocytic Leukemia. He has had treatment
with several courses of chemotherapy and has now been admitted to hospital with pneumonia.
His past medical history revealed that he had suffered several previous upper respiratory tract
infections over the previous six months.
Which of the following components of his immune system is likely to be deficient?
A) complement
B) immunoglobulin G
C) macrophages
D) mast cells
E) T lymphocytes

Answer: B

27. A 42-year-old man presented to the hospital with right upper quadrant pain. He was found to
have multiple masses in the liver that were found to be malignant on H&E staining of a biopsy
sample. Your initial history, physical examination, and laboratory tests, including prostate-
specific antigen, are unrevealing. Lung, abdominal, and pelvic CT scans are unremarkable. He is
an otherwise healthy individual with no chronic medical problems. Which immunohistochemical
markers should be obtained from the biopsy tissue?
A) α-Fetoprotein
B) Cytokeratin
C) Leukocyte common antigen
D) Thyroglobulin
E) Thyroid transcription factor 1
Answer: B

28. A 47-year-old man is evaluated for 1 year of recurrent episodes of bilateral ear swelling. The
ear is painful during these events, and the right ear has become floppy. He is otherwise healthy
and reports no illicit habits. He works in an office and his only sport is tennis. On examination,
the left ear has a beefy red color, and the pinna is tender and swollen; the earlobe appears
minimally swollen but is neither red nor tender. Which of the following is the most likely
explanation for this finding?
A) Behçet's syndrome.
B) Cogan's syndrome.
C) Hemoglobinopathy.
D) Recurrent trauma.
E) Relapsing polychondritis.

Answer: E

29. One of the following pulmonary function test results is with the respiratory disorder chronic
obstructive pulmonary disease.
A) Increased total lung capacity (TLC), decreased vital capacity (VC), decreased FEV1/FVC
ratio.
B) Decreased TLC, decreased VC, decreased residual volume (RV), increased FEV1/FVC ratio,
normal maximum inspiratory pressure (MIP).
C) Decreased TLC, increased RV, normal FEV1/FVC ratio, decreased MIP.
D) Normal TLC, normal RV, normal FEV1/FVC ratio, normal MIP.
E) Decreased TLC, normal RV, normal FEV1/FVC ratio, increased MIP.

Answer:A

30. A 23 year old single woman referred with 3-month history of weight loss and heat
intolerance. On examination pulse is 120 beat/min regular, blood pressure 120/72 mmHg, weight
58 kg, height 165 cm. she had diffuse goiter with bilateral exophthalmos. Investigations: serum
free T4 3.9 ng/dl (N: 0.9-2.0), serum TSH 0.001 mu/l (N: 0.4-4.0), TSH receptor antibody 8
U/L(N:<2). She was started on methimazole 30 mg/day. Six weeks later she developed severe
migratory arthralgia associated with the presence of palpable purpura over both legs and dermal
infarcts in the finger tips. Laboratory studies shows ESR 54 mm/h, urine showed microscopic
hematuria, ANCA titer was 1:640, antimyeloperoxidase antibodies 112U/l (N:0-9),
antiproteinase 3 antibodies 6.4 U/l (N:0-3.5). The best action is:
A) Continue methimazole and add NSAID.
B) Continue methimazole and add prednisolone 40 mg per day.
C) Substitute propylthiouracil for methimazole.
D) Refer the patient for definitive therapy with radioactive iodine.
E) No intervention.

Answer: E
31. A 34-year-old man comes for neurologic consultation because of paroxysmal episodes of
speech difficulty that have occurred recently. The best way to distinguish whether these are
seizures or other types of events is:
A) History
B) Neurologic examination
C) Brain MRI
D) Routine EEG
E) Empiric anticonvulsant trial.

Answer: A

32. A 23-year-old woman is diagnosed with a lower extremity deep venous thrombosis. Which
of the following medical conditions represents a contraindication to therapy with low-molecular-
weight heparin (LMWH)?
A) Pregnancy
B) Obesity
C) Dialysis-dependent renal failure
D) Uncontrolled diabetes mellitus
E) Jaundice.

Answer: C

33. All the following forms of glomerulonephritis (GN) have associated normal serum
complement C4 levels EXCEPT:
A) lupus nephritis stage IV
B) poststreptococcal GN
C) hemolytic-uremic syndrome
D) membranoproliferative GN type II
E) endocarditis-associated GN

Answer: A

34. A 60-year-old man attends clinic because of hypertension. His BP in clinic is 70/90 mmHg
and his echocardiogram shows mild LVH and mild LA dilatation. He is not diabetic and has no
other medical history of note.Which one of the following medications is most effective in
preventing AF?
A) ACE inhibitors
B) Beta-blockers
C) Calcium-channel antagonists
D) Diuretics
E) Alpha-blocker

Answer: A

35. A 35-year-old male is seen in the clinic for evaluation of infertility. He has never fathered
any children, and after 2 years of unprotected intercourse his wife has not achieved pregnancy.
Sperm analysis shows a normal number of sperm, but they are immotile. Past medical history is
notable for recurrent sinopulmonary infections, and the patient recently was told that he has
bronchiectasis. Chest radiography is likely to show which of the following?
A) Bihilar lymphadenopathy
B) Bilateral upper lobe infiltrates
C) Normal findings
D) Situs inversus
E) Water balloon–shaped heart

Answer: D

36. All of these findings are consistent with a chronic unilateral urinary tract obstruction
EXCEPT:
A) anemia
B) dysuria
C) hypertension
D) pain with micturition
E) pyuria

Answer: D

37. Which of the following is more likely to present as a myopathic disorder of gastrointestinal
motility rather than as a neuropathic disorder?
A) Parkinson disease
B) Diabetes mellitus
C) Multiple sclerosis
D) Anticholinergic medication use
E) Dermatomyositis

Answer: E

38. Regarding ophthalmic complications in a patient with diabetes, which ONE of the following
statements is INCORRECT?
A. There is a higher risk of vitreous haemorrhage than in the general population
B. Ocular haemorrhage is common in patients with diabetic retinopathy, following thrombolysis
for acute myocardial infarction
C. Diabetic retinopathy may cause retinal detachment
D. Measurement of intraocular pressure is important because glaucoma is more common in
diabetics

Answer: B

39. Carcinoid tumours of the foregut may be associated with which of the following
A) Cushing's syndrome
B) hypercalcemia
C) pellagra
D) pulmonary hypertension
E) carcinoid syndrome

Answer: A

40. A 69-year-old woman with rheumatoid arthritis presented with swollen ankles. She was
diagnosed as having rheumatoid arthritis over 18 years ago and had been relatively well
controlled on non-steroidal antiinflammatory drugs until six months ago, when her joint pains
and swelling required the addition of penicillamine to control her symptoms. The patient had a
past history of hypertension, for which she took bendroflumethiazide. On examination she had
symmetrical joint deformities consistent with rheumatoid arthritis. The heart rate was 90
beats/min and irregular. Her blood pressure measured 140/90 mmHg. The JVP was not raised.
Both heart sounds were normal and the chest was clear. Abdominal examination was normal.
Inspection of the lower limbs revealed pitting oedema.
Investigations are shown.
Hb 11 g/dl
WCC 5 X 109/l
Platelets 190 X109/l
Sodium 134 mmol/l
Potassium 4.5 mmol/l
Urea 6 mmol/l
Creatinine 70 micromol/l
Bilirubin 11micromol/l
Alkaline phosphatase 100 iu/l
Albumin 26 g/l
Urinalysis Protein +++
What is the management?
A) Stop penicillamine.
B) Start prednisolone.
C) Start ACE inhibitor therapy.
D) Arrange renal biopsy.
E) Arrange IVU.

Answer: A

41. You have received a blood test report with the following results: serum calcium 3 mmol/l,
serum phosphate 0.6 mmol/l (normal range 0.8-1.4 mmol/l); plasma parathyroid hormone (PTH)
5.8 pmol/l (normal range 0.9-5.4 pmol/l). Which one of the following disorders could be
associated with this abnormal blood test?
A) Addison's disease
B) DiGeorge syndrome
C) Mucocutaneous candidiasis
D) Medullary-cell carcinoma
E) Magnesium deficiency

Answer: D
42. Vitiligo is associated with which one of the following disorders?
A) Nelson's syndrome
B) Hyperparathyroidism
C) Amiodarone therapy
D) Addison's disease
E) Chronic renal failure

Answer: D

43. A 50-year-old woman is admitted with acute diarrhoea and dehydration. Just prior to her
admission, she had a bout of lower urinary tract infection and is on antibiotics. Stool culture is
negative and sigmoidoscopy reveals pseudomembranous colitis. Which one of the following
statements is most characteristic of this patient's condition?
A) Bloody diarrhoea, abdominal pain and tenderness is the most common presentation
B) The detection of Clostridium difficile bacilli in the stool is diagnostic
C) A severe form of the disease is often associated with gentamicin therapy
D) It is caused by a Gram-positive anaerobic bacterium
E) Intravenous vancomycin for 2 weeks is an effective treatment

Answer: D

44. A 45-year-old man is being investigated for persistent dyspepsia and heartburn. Acid
secretion studies show gastric acid hypersecretion. Which one of the following conditions is the
most likely cause of this patient's symptoms?
A) Pernicious anaemia
B) Large-bowel resection
C) Vasoactive intestinal polypeptide- (VIP-) secreting tumour
D) Systemic mastocytosis
E) Cushing's syndrome

Answer: D

45. A 19-year-old student presents with acute shortness of breath and haemoptysis. She reported
left loin pain and was found to have haematuria 2 weeks ago. She is under regular review by the
renal physicians for nephrotic syndrome. Ultrasound of the kidney shows left renal vein
thrombosis and a computed tomography pulmonary angiogram confirms acute pulmonary
embolism. Which one of the following is the most likely cause of this patient's increased
tendency for developing venous thrombosis?
A) Factor V Leiden mutation
B) High levels of anticardiolipin antibodies
C) Protein S deficiency
D) Protein C deficiency
E) Antithrombin III deficiency

Answer: E
46. Which of the following statements is most accurate with regard to Wilson's disease?
A) The primary defect is believed to be enhanced intestinal absorption of copper
B) An alternative diagnosis should be considered if chorea occurs in the absence of Kayser-
Fleischer rings
C) Chronic liver disease and autoimmune haemolytic anaemia are recognised features
D) Raised serum copper levels are evident at birth
E) A sibling with biochemical evidence of the disease is treated only when he or she becomes
symptomatic

Answer: B

47. Which one of the following conditions is most likely to be associated with pyoderma
gangrenosum?
A) Tuberculosis
B) Leprosy
C) Chronic myeloid leukaemia
D) Sulphonamide therapy
E) Cushing's syndrome

Answer: C

48. A retired 69-year-old man presents to the Emergency Department with a temperature of 39
°C. The white cell count (WCC) is 45 x 109/l with 90% neutrophils. Which one of the following
findings would be most useful for differentiating chronic myeloid leukaemia (CML) from a
leukaemoid reaction?
A) Philadelphia chromosome
B) Splenic enlargement
C) Low leucocyte alkaline phosphatase score
D) Hypercellular bone marrow
E) Elevated platelet count

Answer: A

49. The blood gas analysis in a 40-year-old woman who has presented with fatigue shows: pH
7.51, Pao2 11 kPa, Paco2 6 Pa, bicarbonate 32 mmol/l. Which one of the following is the most
likely cause of this patient's clinical presentation?
A) Spironolactone therapy
B) Acetazolamide therapy
C) Conn's syndrome
D) Addison's disease
E) Chronic diarrhoea

Answer: C

50. A 30-year-old woman presents to the Emergency Department at 9 pm with severe headache.
Two hours earlier she had felt as if she was hit on the back of the head and she then experienced
severe occipital headache and vomiting. A diagnosis of subarachnoid haemorrhage is suspected.
Urgent computed tomography (CT) scan of the brain is normal. Which one of the following is
the most appropriate next step in the management?
A) Reassure and discharge after prescribing strong painkillers
B) Observe in hospital and repeat the CT brain scan the next morning
C) Perform a lumbar puncture and cerebrospinal fluid (CSF) analysis immediately
D) Observe in hospital and delay lumbar puncture and CSF analysis until the next morning
E) Arrange urgent magnetic resonance imaging (MRI) of the brain

Answer: D

MULTIPLE CHOICE QUESTIONS-2

1. A 42-year-old woman has a12-year history of ulcerative colitis that has responded well to
mesalamine and occasional corticosteroid enemas. Recent surveillance colonoscopy with
biopsies showed low-grade dysplasia. Which of the following would be the most appropriate
next step in the management of this lady?
A) No intervention, repeat colonoscopy in 6 months
B) No intervention, repeat colonoscopy in 1 year
C) Administer continuous corticosteroid enemas
D) Refer to the surgeon for colectomy
E) Administer a high-dose corticosteroid intravenously

Answer: D

2. A 28-year-old man undergoes an endoscopic transthoracic sympathectomy for palmar


hyperhidrosis. Postoperatively, he has developed ptosis of his eye. This is due to iatrogenic
damage to which one of the following nerves?
A) 8th cervical nerve
B) 1st thoracic nerve
C) Lower cord of brachial plexus
D) 2nd thoracic ganglion
E) 7th cervical nerve

Answer: B

3. A 51-year-old woman undergoes a successful bone marrow transplant from a matched


unrelated donor for refractory Hodgkin’s disease. She is discharged from the hospital on no
medications and is feeling well. At an appointment 6 months posttransplant, she is well with no
evidence of malignancy. Three weeks later, she travels to Florida with her family. She is cautious
of the sun but develops sunburn on her face, despite wearing sunscreen and a protective hat.
When she returns from her trip 5 days later, she presents with persistent erythema of her face.
She also states that her wrists and hands have been sore for the past 2–3 weeks. On examination,
her face is mildly tender to touch, and a rash is present. Her hands are diffusely swollen. She is
afebrile, and the remainder of her physical examination is benign. What is the most likely
diagnosis?
A) Graft-versus-host disease
B) Hypersensitivity to sunlight due to the patient’s antirejection regimen
C) New-onset systemic lupus erythematosus
D) Rosacea
E) Staphylococcal skin infection acquired during travel

Answer: A

4. A 35-year-old woman arrives on the floor after an uneventful hysteroscopy to evaluate her
long history of uterine fibroids. About 30 minutes after her arrival, she begins to complain of
nausea and has two episodes of vomiting. The physician administers 0.625 mg of droperidol and
400 mg of acetaminophen by mouth. On follow-up evaluation, the patient's neck is involuntarily
flexed to one side. She is alert, oriented, and conversant and has an otherwise normal neurologic
examination. Which of the following is the most likely diagnosis?
A) Cerebral vascular accident
B)Conversion disorder
C) Dystonic reaction to droperidol
D) Munchausen syndrome
E) Seizure

Answer: C

5. A 50-year-old woman with diabetes mellitus presents with backache and inability to walk
unaided. This came on suddenly, while she was trying to lift her shopping bag from the car boot.
Which of the following is most suggestive of a lesion of the sciatic nerve?
A) Absent knee tendon jerk
B) Foot drop
C) Inability to flex the hip
D) Decreased sensation on the anterior thigh and medial leg
E) Intervertebral disc prolapse at L2/L3 level

Answer: B

6. A 59-year-old man has a body mass index (BMI) of 42. As might be expected, his fasting
blood glucose level is high, 210 mg/dL, as was his HbA1c level, 9.8%. In addition, he suffers
from hypertension and dyslipidemia. His physician advised him to lose weight both for his
general well-being and to help control his diabetes. Consequently, for the past 4 years, he has
desperately tried to lose weight. He worked his way through an alphabet of popular and fad diets,
from the Atkins diet to the Weight Watchers diet. If he lost a few pounds while on a particular
diet, he gained back the pounds lost, plus a few more within a few months of terminating the diet.
Finally, he and his physician decided he should try bariatric surgery, the Roux-en-Y gastric
bypass procedure. A week after leaving the hospital, his fasting blood glucose level was 100
mg/dL. Three months later, it was 96 mg/dL and his HbA1c was 6.0%. Which of the following
choices is most likely to explain this remarkable improvement is his diabetes?
A) The reduction in level of his circulating glucagonlike peptide-1 (GLP-1)
B) The reduction in level of his circulating peptide YY (PYY)
C) The reduction in his mass of adipose tissue
D) The reduction in the level of his circulating ghrelin
E) The reduction in the level of his circulating leptin

Answer: D

7. A 51-year-old man has recurrent bouts of lightheadedness and mild confusion. Episodes of
hypoglycemia were recently documented that are improved with ingestion of food. The patient
has no headache, blurred vision, or double vision. He has gained approximately 4.5 kg (10 lb) in
the past 2 months. Medical and family histories are noncontributory. Physical examination is
normal. The following laboratory data are obtained after an overnight fast: plasma glucose 30
mg/dL (1.67 mmol/L), serum insulin 30 mlU/L (215.25 pmol/L), and an elevated serum C-
peptide level. Screening for sulfonylurea is negative and CT scan of the abdomen is normal.
Which of the following diagnostic studies should be done next?
A) Endoscopic retrograde pancreatography.
B) Transabdominal (Endoscopic) ultrasonography.
C) MRI of the abdomen.
D) Positron emission tomography.
E) Somatostatin receptor scintigraphy.

Answer: B

8. A 23-year-old married woman comes to the office after recent exposure to a person with
active hepatitis A. She has a long history of recurrent sinopulmonary infections and bronchial
asthma. In addition, after her last pregnancy, she received a blood transfusion for severe
postpartum hemorrhage. After receiving an intramuscular dose of immune serum globulin as
prophylaxis against hepatitis A, she develops an anaphylactic reaction. Which of the following is
the most likely cause of this patient’s reaction?
A) Immunoglobulin A (IgA) deficiency with anti-IgA antibodies
B) A hemolytic transfusion reaction
C) Contaminated immune serum globulin
D) A type IV hypersensitivity reaction against a protein in the immune serum globulin
E) A febrile reaction

Answer: A

9. A 47-year-old man recently consulted a physician about developing weakness, particularly in


his right hand. Upon providing a history, the man explained that he does house repair and has
been working on a neighborhood rehabilitation project for the past several months. In doing this,
he sandblasts and sands and scrapes by hand to remove the old paint. These
homes were first constructed in the 1920s and since have been covered with several layers of
paint. He also revealed that he habitually ate his lunch at the work site, which he described as
being dusted with old paint particles. In addition to the weakness in his arm, he admitted to
sporadic stomachaches, constipation, and said his wife had complained that he is always irritable.
He also states that, until recently, he had been in good health. Upon examination, he was found
to be 6 feet (19.7 m) tall and to weigh 170 lb (77.1 kg). His heart, lungs, and abdomen were
normal, as were most analytical values, but he did show signs of right wristdrop consistent with
radial nerve palsy and his complete blood count (CBC) showed a microcytic anemia; his serum
iron levels were found to be normal. Which of the following diagnostic tests would provide the
most useful information regarding the appropriate treatment?
A) Nerve conduction velocity (NCV) study of the right arm
B) Radiography of the right arm and wrist
C) Magnetic resonance imaging (MRI) scans of the right arm and wrist
D) Urine screen for heavy metals (lead, mercury, arsenic)
E) Screening for diabetes mellitus.

Answer: D

10. Which of the following clinical findings is least likely in a patient with infarction in the
distribution of the left anterior choroidal artery?
A) Right hemiparesis
B) Right hemi-sensory loss
C) Right hemianopia
D) aphasia.

Answer: D

11. A 76-year-old man presenting with dysphagia was found to have inoperable esophageal
adenocarcinoma. He has an endoscopically placed esophageal stent for palliation of his
symptoms, but unfortunately found it very painful, and it was removed a few days later. There is
no perforation. He asks whether there are any other treatment options to help with his symptoms.
Which of the following modalities is an appropriate first-line treatment option to discuss?
A) Band ligation
B) Botulinum toxin injection
C) Brachytherapy
D) Local ethanol injection
E) Photodynamic therapy.

Answer: C

12. An AIDS patient develops symptoms suggestive of a severe, persistent pneumonia with
cough, fever, chills, chest pain, weakness, and weight loss. The patient does not respond to
penicillin therapy, but goes on to develop very severe headaches. The presence of focal
neurologic abnormalities leads the clinician to order a CT scan of the head. This demonstrates
several metastatic brain abscesses. Biopsy of one of these lesions demonstrates beaded,
branching, filamentous gram-positive bacteria that are weakly acid fast. Which of the following
is the most likely causative organism?
A) Actinomyces
B) Aspergillus
C) Burkholderia
D) Francisella
E) Nocardia.
Answer: E

13. You are a hospitalist called to admit a 70-year-old man to the ICU. His wife states that he
was sitting at the table eating breakfast with her when he dropped his fork and had difficulty
speaking. Within a couple of minutes he was unable to move his right side. She called the
paramedics, who brought him to the hospital. Now in the ICU, his vital signs are as follows: BP
200/98 mm Hg, HR 100, RR 10, O2 saturation 94% on room air. He is afebrile. On physical
examination he is lethargic and unable to speak. His pupils are equal and round but sluggish. He
has flaccid paralysis of the right arm and leg with a Babinski sign present on the right. His heart
is irregularly irregular, and an ECG confirms atrial fibrillation. A CT of the head shows a large
bleed in the left frontoparietal area with mass effect and midline shift. You decide to intubate the
patient to protect his airway. What is the next most appropriate step in the treatment of this
patient while you are awaiting your urgent neurosurgical consult?
A) Hyperventilate the patient to a goal pCO2 of 20 mm Hg
B) Give a bolus of IV mannitol
C) Give a bolus of IV dexamethasone
D) Give sublingual nifedipine to decrease the BP
E) Anticoagulate with IV heparin because of the atrial fibrillation.

Answer: B

14. A 73-year-old woman is brought in by paramedics after fainting in the mall and hitting her
face. She does not remember any preceding symptoms, and she did not lose control of her bowel
or bladder. Witnesses at the scene say that she was down for less than a minute, then woke up
and was fairly alert. She was bleeding from a laceration on her chin and paramedics were called.
When she arrived at the hospital, her initial laboratory
values were normal and an EEG did not show epileptiform activity. She is placed on a cardiac
monitor. The following day, she becomes lightheaded and loses consciousness while lying in bed,
and her monitor shows tachycardia with the QRS complexes being uniformly longer than 120 ms.
What is the most common cause of this rhythm disturbance?
A) Uncontrolled hypertension
B) Distention of the pulmonary veins
C) Accessory pathway
D) Ischemic heart disease.

Answer: D

15. A 47-year-old woman presents at night to the Emergency Department with chest pain. She
states that the pain started that evening and has progressively been getting worse. She is
concerned that she is having a heart attack. The pain is described as a burning sensation
associated with a sour taste in her mouth, and it started shortly after she ate dinner; it has
occurred on previous occasions, but never as bad as it is now.
Previously, she used calcium carbonate tablets that were effective for the pain. She has no
history of heart disease or other medical problems, and she takes no regular medications. She
does not smoke cigarettes or use cocaine. Her vitals are normal, and her physical examination is
unremarkable. Initial laboratory tests and an ECG are normal. Which of the following is the most
likely cause of this patient’s chest pain?
A) Unstable angina
B) Myocarditis
C) Pulmonary embolism
D) Gastroesophageal reflux disease
E) Costochondritis.

Answer: D

16. A 28-year-old patient with end-stage renal disease (ESRD) on continuous ambulatory
peritoneal dialysis (CAPD) for two months presents with fever, abdominal pain and cloudy
dialysis fluid. There is no diarrhea or vomiting and the pain has been present for about 12 hours.
The patient has ESRD secondary to chronic glomerulonephritis, there is no history of diabetes,
urinary infections or antibiotic use. Examination reveals a temperature of 38.9 C (102 F), and
blood pressure of 110/70 mm Hg. The throat is clear, as are the lungs. Cardiac examination
reveals a grade 2/6 systolic murmur. Abdominal examination reveals decreased bowel sounds
with diffuse tenderness. There is mild rebound. There is no edema or skin rash. A complete
blood count shows a leukocyte count of 14,200/mm3, hemoglobin is 12.5 g/dL. Peritoneal fluid
is cloudy with 1,000 white blood cells, 85% of which are polymorphonuclear leukocytes. Gram's
stain of fluid is negative. Cultures of blood and peritoneal dialysis fluid are taken. Which of the
following is the most appropriate initial step in management?
A) Fluconazole
B) Immediate removal of dialysis catheter.
C) Intravenous vancomycin
D) Intravenous gentamicin
E) Oral ciprofloxacin

Answer: C

17.A 59-year-old lady is admitted with a 30 minute history of heavy central chest pain associated
with nausea and sweating. Her ECG shows ST elevation in leads V1, V2, V3 and V4.
Which of the following coronary arteries is most likely to be occluded?
A) Circumflex artery
B) Left anterior descending artery
C) Obtuse marginal artery
D) Posterior descending artery
E) Right coronary artery

Answer: B

18. A family physician cares for a family consisting of a 45-year old husband, 43-year-old wife
and a 12-year-old daughter. The family reports that recently the 77-year-old maternal
grandmother who lived with them died after a prolonged respiratory infection. Autopsy
subsequently confirms that she had active pulmonary tuberculosis at the time of death. The
organism tested sensitive to all anti-tuberculosis drugs. In responding to the grandmother's illness,
which of the following is the most appropriate step in management?
A) Obtain leukocyte counts on all family members
B)Obtain sputum cultures for acid fast bacilli
C)Obtain chest computerized tomograms on all members
D)Place protein purified derivative (PPD) test on all members
E)Schedule bronchoscopy lavage for the adults.

Answer: D

19. Which of the following extraintestinal manifestations is associated with Crohn disease but
not ulcerative colitis?
A) Ankylosing spondylitis
B) Erythema nodosum
C) Nephrolithiasis
D) Thromboembolic disease
E) Uveitis

Answer: C

20. A 30 year old woman presents with unprovoked left popliteal deep vein thrombosis. Her
family history is negative for venous thromboembolism. She has a history of SLE and takes
prednisone 10mg daily, hydroxychloroquine and supplements of calcium and vitamin D. She has
one child but has had two miscarriages. An aPTT test performed before starting anticoagulation
in this patient is prolonged, but her PT is normal.
Which statement is most likely to be correct?
A) Because her APTT is increased already, Heparin should be withheld and warfarin treatment
initiated at INR 2.5 to 3.5, perhaps forever
B) The patient should be encouraged to use oral contraceptive to prevent pregnancy during the
time she is anticoagulated
C) She will need Heparin and then anticoagulation with Warfarin at INR 2 to 3, perhaps
indefinitely
D) Start anti-coagulation with Heparin and then Warfarin at INR 2.5 to 3.5 for 3 to 6 months
E) She will need lifelong new oral anticoagulants.

Answer: C

21. A 65-year-old man has been stable on the general medical ward following an admission with
acute coronary syndrome several days previously. His drug history consists of aspirin, enalapril
and glyceryl trinitrate (GTN) spray. He has developed dyspnea over the last few hours. On
examination he has a raised jugular venous pressure (JVP) and crackles to his mid zones. His
electrocardiogram (ECG) shows a rate of 140 beats per minute in atrial fibrillation. Which of the
following is the most appropriate management?
A) Intravenous amiodarone
B) Intravenous digoxin
C) Intravenous flecainide
D) Observe and screen for MI
E) Synchronized DC cardioversion.
Answer: E

22. A 48-year-old man is admitted through the ER with a possible left lower extremity cellulitis.
The patient says that he has been having fever, swelling, and redness of his left lower extremities
that comes and goes spontaneously over the past couple of months. He also tells you about an
episode in which he lost vision in his left eye for several minutes a couple of weeks ago, but the
vision returned without incident. He had a urologic evaluation for penile trauma 3 months
ago. On physical examination, his BP is 125/80, HR 70, RR 14, and he is currently afebrile. His
examination is significant for a 3/6 systolic murmur heard at the left lower sternal border without
radiation while lying supine. Blood cultures return positive results for Enterococcus species, and
an echocardiogram reveals large mitral vegetation. You review treatment for enterococcal
endocarditis. Which of the following antibiotics always misses enterococcal infections?
A) Ampicillin-sulbactam
B) Nitrofurantoin
C) Cefipime
D) Vancomycin
E) Linezolid.

Answer: C

23. A 70-year-old woman with a history of type2 DM and CAD with a known left bundle branch
block on ECG is admitted to the ICU with sepsis from a urinary source. She is fluid resuscitated,
and empiric broad-spectrum antibiotics are begun after the appropriate cultures are obtained.
Despite what appears to be adequate resuscitative efforts with volume replacement, the patient
has had minimal urine output over the past couple of hours. You decide to place a pulmonary
artery catheter to help to determine the patient’s hemodynamic situation. Which of the following
complications of pulmonary artery catheter placement is the patient at increased risk for because
of her past medical history?
A) Pulmonary artery perforation
B) Pulmonary infarction
C) Complete heart block
D) Pneumothorax
E) Ventricular tachycardia.

Answer: C

24. The first four tests that should be ordered in the initial evaluation of patients with a suspected
coagulopathy are all of the following except
A) Platelets count
B) Bleeding time
C) Platelet aggregation studies
D) PT
E) aPTT

Answer: C
25. Which of the following is not true about tetanus?
A) The toxin affects inhibitory GABA and glycine receptors, leading to unopposed contraction
and spasm of skeletal muscle.
B) It is characterized by acute onset of skeletal muscle rigidity and convulsive spasm.
C) Initial symptoms involve lockjaw and risus sardonicus.
D) Fractures, dislocations, and rhabdomyolysis may occur due to forceful sustained muscle
E) Tetanus disease usually leads to long-lasting immunity.

Answer: E

26. A 55-year-old woman, who has never smoked, presents to you on the ward with a history of
weight loss, decreased appetite and finger clubbing. You are told that her chest x-ray revealed
opacity in the hilar region of the right lung suggesting a bronchogenic carcinoma. She is
currently awaiting a CT-chest with bronchoscopy to follow. From the list below, select the most
likely diagnosis:
A) Squamous cell carcinoma of the lung
B) Adenocarcinoma of the lung
C) Small cell carcinoma of the lung
D) Large cell carcinoma of lung
E) Carcinoid tumour of the lung.

Answer: B

27. A 68-year-old woman recently diagnosed with multiple myeloma presents to her GP with
progressively increasing breathlessness, exercise intolerance and ankle swelling. On examination,
there is bilateral pitting leg edema to her thighs, ascites and raised JVP. The apical impulse is
impalpable. An ECG shows diffusely diminished voltage. Chest X-ray is normal and the
echocardiogram shows small thick ventricles and dilated atria with a thickened interatrial septum.
The ventricular myocardium has a granular sparkling texture on echo, and minimal fluid in the
pericardial space is noted. What is the most likely diagnosis?
A) Chronic pericardial effusion with tamponade
B) Chronic pericardial effusion without tamponade
C) Constrictive pericarditis
D) Restrictive cardiomyopathy
E) Congestive heart failure.

Answer: D

28.A 65-year-old male patient is admitted with renal failure and is diagnosed with acute tubular
necrosis. Which of the following is least likely to be the cause of acute tubular necrosis?
A) Rhabdomyolysis
B) Paracetamol poisoning
C) Hypovolemia
D) Hypertension
E) Corticosteroid therapy
Answer: E

29. A 65-year-old woman with long-standing diabetes mellitus visits your clinic for follow-up 2
weeks after the initial visit. Her fasting home glucose monitoring shows elevated blood sugar
levels ranging between 200-250 mg/dl. Two weeks ago, her HbA1c was 7.2% and the fasting
plasma glucose was 212 mg/dl. You recommend the following to evaluate the discrepancy
between the fasting values and the HbA1c:
A) Order a CBC
B) Prescribe a new glucometer
C) Order a fructosamine
D) Repeat the HbA1c
E) Repeat the fasting plasma glucose.

Answer: A

30. A 43-year-old man presents to the clinic with complaints of fever, night sweats, anorexia,
cough, and chest pain. The chest x-ray reveals infiltrates in both the lower and upper lobes, with
possible cavitations in the apices. A presumptive diagnosis of tuberculosis is made on the basis
of finding acid-fast bacilli (AFB) on microscopic examination of sputum. The patient is started
initially on isoniazid, rifampin, pyrazinamide, and ethambutol.
What is the best way to monitor this patient?
A) Sputum acid-fast stains every month for 6 months
B) Sputum cultures every month until cultures become negative
C) Serial chest x-rays
D) Blood testing for drug toxicity
E) Observe for clinical deterioration.

Answer: B

31. A 49-year-old man with untreated hepatitis C infection develops persistent proteinuria.
Which of the following diagnoses is not typically associated with hepatitis C?
A) Membranous nephropathy
B) Focal segmental glomerulosclerosis
C) Membranoproliferative glomerulonephritis
D) Cryoglobulinemia
E) Polyarteritis nodosa

Answer: B

32. A 59-year-old woman is admitted with central abdominal pain. Serum amylase is 1800 IU/L.
Her initial Glasgow Coma Scale score is 4. You are asked to review her the next day as the
nurses have noticed that her urine output has been just 15 mL in the past 3 hours. The rest of her
observations are as follows:
• Blood pressure = 105/45 mmHg
• Pulse = 113 beats/min
• Respiratory rate = 28 breaths/min
• Saturation 93% on 8 L of oxygen
On auscultation of her chest you hear widespread crepitations. What is the most appropriate next
course of action?
A) Fluid restriction
B) Colloid bolus
C) Furosemide
D) Transthoracic echocardiogram
E) Central line insertion

Answer: E

33. Features of type 1 renal tubular acidosis include all of the following EXCEPT:
A) Normal AG acidosis
B) Hypokalemia
C) Ca phosphate stone formation
D) Urine pH decrease below 4, following oral ammonium chloride loading test
E) Raised serum PTH hormone

Answer: D

35. All of the following statements regarding non-alcoholic steatohepatitis (NASH) are correct
EXCEPT:
A) Liver biopsy is the only means of accurately diagnosing NASH.
B) Weight loss has been shown to result in improvement of elevated liver enzymes.
C) Patients with NASH can develop liver cirrhosis.
D) Histologically it is characterized by severe portal or peri-portal inflammation.
E) Laboratory studies cannot identify patients at risk of progression.

Answer: D

36. A 30-year-old male patient complains of fever and sore throat for several days. T he patient
presents to you today with additional complaints of hoarseness, difficulty breathing, and drooling.
On examination, the patient is febrile and has inspiratory stridor. Which of the following is the
best course of action?
A) Begin outpatient treatment with ampicillin.
B) Culture throat for β-hemolytic streptococci.
C) Admit to intensive care unit and obtain otolaryngology consultation.
D) Schedule for chest x-ray.
E) Obtain Epstein-Barr serology.

Answer: C

37. Regarding management of an episode of hypoglycaemia in a 65-year-old who is on


sulfonylurea therapy, which ONE of the following is TRUE?
A) Initial oral or intravenous glucose therapy can be omitted because it is likely to fail
B) Octreotide should be considered for recurrent or persistent hypoglycaemia
C) Hypoglycaemia in a stable diabetic on a regular sulfonylurea dose is not usually due to a
precipitating event
D) There is a more sustained response to intravenous glucose therapy in sulfonylurea-induced
hypoglycaemia than in insulin-induced hypoglycaemia

Answer: B

38. A 59-year-old man presents to the ED with left-sided chest pain and shortness of breath that
began 2 hours prior to arrival. He states the pain is pressure-like and radiates down his left arm.
He is diaphoretic. His BP is 160/80 mm Hg, HR 86 beats per minute, and RR 15 breaths per
minute. ECG reveals 2-mm ST-segment elevation in leads I, aVL, V3 to V6. Which of the
following is an absolute contraindication to receiving thrombolytic therapy?
A) Systolic BP greater than 180 mm Hg
B) Patient on Coumadin and aspirin
C) Total hip replacement 3 months ago
D) Peptic ulcer disease
E) Previous hemorrhagic stroke

Answer: E

39. The best pharmacologic treatment for hypotension related to the spinal anesthetic is
A) atropine
B) epinephrine
C) phenylephrine
D) calcium
E) labetalol

Answer: C

40. An ischemic stroke involving the right side of the pons could lead to which of the following
patterns of weakness?
A) Left facial weakness and right body weakness
B) Right facial weakness and left body weakness
C) Right facial weakness and right body weakness
D) Left arm weakness and right leg weakness
E) Right arm weakness and left leg weakness

Answer: B

41. A 49-year-old man who smokes two packs of cigarettes a day presents with a lung mass on
x-ray and recent weight gain. Laboratory examination shows hyponatremia with hyperosmolar
urine. The patient probably has
A) Renal failure
B) Pituitary failure
C) Conn’s syndrome
D) Cardiac failure
E) Inappropriate ADH

Answer: E

42. A 75-year-old woman is referred for assessment of her mental state. Her carer states that she
noticed general decline in her physical and mental state. She is more forgetful and her
concentration is poor. Sometimes her speech is incoherent. She trips frequently and had many
falls. On rare occasions she has imagined seeing people sitting in her room. Each of the
following features would help to distinguish between organic and functional disorders, except
A) Nystagmus
B) Visual hallucinations
C) Poor performance of the serial sevens test
D) Dysphasia
E) Hemianopia

Answer: C

43. A 40-year-old man with occasional dysphagia and who otherwise feels well undergoes
esophageal motility studies that show an LES amplitude of approximately 60 mmHg. The
esophagus relaxes completely when he swallows. The most likely diagnosis is
A) GERD (gastroesophageal reflux disease)
B) Achalasia
C) Hypertensive LES
D) Barrett’s esophagus
E) Esophageal spasm

Answer: C

44. A 60-year-old accountant complains of recurrent attacks of exquisite pain and swelling in the
left big toe. Each of the following conditions is likely to be associated with this disorder, except
A) Chronic alcoholism
B) Obesity
C) Rheumatoid arthritis
D) Diabetes mellitus
E) Diuretic therapy

Answer: C

45. A 60-year-old man was referred for further assessment of a cardiac murmur. On examination
it was noted that he has a weak and slow-rising carotid pulse. The most likely underlying cardiac
abnormality is
A) Aortic regurgitation
B) Alcoholic cardiomyopathy
C) Dissecting aneurysm
D) Hypertrophic obstructive cardiomyopathy (HOCM)
E) Aortic stenosis

Answer: E

46. A 50-year-old woman has a history of gastrinoma and pituitary tumour. She reports
increasing lethargy, drowsiness and constipation. The laboratory studies include raised calcium
(2.9 mmol/I), low phosphate and raised PTH levels. The most likely diagnosis is?
A) Carcinoma of the bronchus
B) Pseudohypoparathyroidism
C) Acromegaly
D) Sarcoidosis
E) Multiple endocrine neoplasia syndrome (MEN I)

Answer: E

47. A 60-year-old teacher is being investigated for increasing shortness of breath and diffuse
fibrotic changes found on plain chest X-rays. He was taking diuretics and anti-arrhythmic
treatment. The 12-lead ECG shows a prolonged QT interval. Each of the following clinical
situations could be responsible for this ECG abnormality, except
A) Heart failure
B) Hypokalaemia
C) Furosemide therapy
D) Hypercalcaemia
E) Amiodarone therapy

Answer: D

48. A 30-year-old mechanic presented with central chest pain, worse on lying flat. He claims that
he has had a flu-like illness for a week. Which one of the following ECG changes is most
characteristic of this disorder?
A) PR prolongation
B) ST depression
C) Peaked, tall T wave
D) Prominent U wave
E) PR-segment depression

Answer: E

49. A 16-year-old male treated for acute lymphocytic leukemia develops severe lumbar and
abdominal pain. His serum amylase is markedly elevated. Which of the following agents most
likely caused these findings?
A) 6-MP
B) Asparaginase
C) Doxorubicin
D) Methotrexate
E) Vincristine

Answer: B

50. A 58-year-old man was admitted with a three-weeks history of shortness of breath. The chest
X-ray demonstrates a right pleural effusion. Pleural fluid analysis shows 6600 mm3 WBC,40%
eosinophils. The condition least likely to be responsible this clinical presentation is
A) Pneumothorax
B) Haemothorax
C) Pulmonary infarction
D) Benign asbestos pleural effusion
E) Tuberculous pleural effusion

Answer: E

MULTIPLE CHOICE QUESTIONS-3

1. A 33-year-old anthropologist from New York had been doing research in a desert region of
Arizona for about 6 months. After returning home, he visits his physician complaining of an
influenza-like illness with cough, mild chest pain, and occasional fever. He says that the illness
started during the last few weeks of his stay in Arizona. Red, tender nodules are present on his
shins. Chest x-rays fail to reveal evidence of pulmonary infiltrates or pleural effusion. Which of
the following is the most appropriate next step in the management of this patient?
A) Delay treatment until culture results are obtained.
B) Begin treatment with fluconazole.
C) Begin treatment with amphotericin B.
D) Aspirate bone marrow and culture.
E) Institute immediate isolation.

Answer: A

2. A young man is referred by his GP following investigation for recurrent mouth ulceration.
Subsequent blood count shows HB of 13.2, WBC 3.8 (neutrophil 1.2, lymph 1.5) PLT 332,
examination reveals a fit young male with no evidence of organomegaly or lymphadenopathy,
further questioning reveals the history of mouth ulceration occurring over the previous 3-4 years.
Select the most likely diagnosis
A) AML
B) Post-viral neutropenia
C) HIV infection
D) Drug induced neutropenia
E) Cyclical neutropenia.

Answer: E
3. A 35-year-old woman presented with tender lesion on both legs. She had no respiratory
symptoms and was not on any medications. O/E: she was febrile, pulses 90 regular and blood
pressure 136/88. Her chest was clear. There were bilateral erythematous raised lesions on her
shins. Chest x ray shows bi-hilar lymphadenopathy. The most helpful test that would give a
definitive diagnosis would be:
A) Kveim test
B) Serum ACE level
C) Serum calcium
D) Skin biopsy
E) Transbronchial needle biopsy

Answer: E

4. A 78-year-old man complains of increasing fatigue and bone pain, especially around the knees
and ankles. He has a long-standing anemia with hemoglobin of 9 to 10 g/dL and MCV of 102.
He had not responded to therapeutic trials of iron and vitamin B12, but had been
symptomatically stable until the past month. Examination reveals pallor and spleen tip just
palpable at the left costal margin. CBC reveals hemoglobin of 8.2 g/dL, but for the first time his
platelet count is low (15,000); the white blood cell count is 14,000. What is the likely cause of
his worsening anemia?
A) Folic acid deficiency
B) Acute myeloid leukemia
C) Myelofibrosis
D) Tuberculosis
E) Viral infection

Answer: B

5. A 58-year-old man with history of chronic myeloid leukemia was admitted with pneumonia
and deep venous thrombosis. He was started on antibiotics and I.V. heparin. His condition
deteriorated and had acute respiratory distress syndrome and hypotension. Despite large amounts
of i.v. fluids and inotropes, he remained hypotensive. You are considering adrenal insufficiency.
What should you do next?
A) Order random cortisol then treat with hydrocortisone
B) Start hydrocortisone then do the ACTH stimulation test
C) Start dexamethasone then do the ACTH stimulation test
D) Start hydrocortisone
E) Do the ACTH stimulation test; treat according to the results

Answer: C

6. A 68-year-old diabetic woman with chronic renal insufficiency is admitted to the hospital with
urosepsis. She has a history of anaphylactic reaction to penicillin that required intubation for
severe bronchospasm. Urinalysis shows > 100 WBC, and Gram's stain shows 3+ plump gram-
negative rods.
Which of the following antibiotics would be most appropriate for this patient with a known
severe penicillin allergy?
A) Ampicillin
B) Ceftazidime
C) Vancomycin
D) Aztreonam
E) Imipenem

Answer: D

7. 65-year-old man with history of chronic AF presents to ER with acute memory impairment
(recent amnesia), confusion and impaired vision. Physical examination confirms the confusion
state and right homonymous hemianopsia. No paresis was noted. The current status is most
probably related to occlusion of:
A) Left posterior cerebral artery
B) Basilar artery
C) Left middle cerebral artery (posterior branch)
D) Left common carotid artery
E) Right vertebral artery

Answer: A

8. A 24-year-old man is in the operating room for a massive liver injury sustained when his
motorcycle hit a truck. After one hour of surgery he has received 15 units of packed cells and has
developed diffuse oozing from the surface of his liver. Clots are no longer forming. His body
temperature is 34oC.
Coagulation abnormalities expected in this patient include all of the following EXCEPT
A) Prolonged PT
B) Prolonged aPTT
C) Prolonged bleeding time
D) Low fibrinogen level
E) Viscoelastic test (point of care coagulation testing) is usually normal.

Answer: E

9. A 42-year-old female with a recent diagnosis of systemic sclerosis, is referred to hospital with
a complaint of headaches and blurred vision. She has a past medical history of asthma. On
examination, her blood pressure is 230/120, and there is bilateral papilloedema. Which of the
following medications should be prescribed immediately?
A) IV Furosemide
B) IV Labetolol
C) Oral angiotensin II receptor blockers plus IV Sodium Nitroprusside
D) Oral captopril plus IV Sodium Nitroprusside
E) Sublingual Nimodipine

Answer: D
10. A 55 year old patient with acute necrotizing pancreatitis has been in the Intensive Care unit
with multiple organ failure for approximately 4.5 weeks. Although his clinical situation
improved over the last few days, the patient now deteriorates. There is fever up to 104 °F, a
rising CRP of 398 and white blood cell count of 27 x 109/L. A contrast enhanced CT
demonstrates a large, heterogeneous, walled off, peripancreatic collection with gas inside. What
is the preferred treatment strategy at this point?
A) Conservative treatment
B) Image-guided percutaneous or endoscopic catheter drainage
C) Primary open necrosectomy
D) Primary minimally invasive surgical necrosectomy
E) Video assisted retroperitoneal debridement.

Answer: B

11. A 25-year-old female is admitted with acute dyspnea and chest pain. A diagnosis of
pulmonary embolism is confirmed and her investigations reveal urine dipstick protein ++ but no
blood, anti-double stranded DNA antibodies of 200 U/mL (0 - 73), with a 24 hour urinary protein
concentration of 5g (< 0.2). Which one of the following diagnoses is most likely to be found on
renal biopsy?
A) AA amyloid
B) Focal segmental glomerulonephritis
C) IgA nephropathy
D) Membranous nephropathy
E) Minimal change nephropathy.

Answer: D

12. A 46-year-old woman is referred for preoperative evaluation before undergoing resection of
newly diagnosed glioblastoma. Her HB is 12 gm/dl, leukocyte count 6.7 x109/L, and PLT 198
x109/L, the surgeon is particularly interested in an opinion concerning her risk of bleeding.
Which of the following will provide the best estimate of her surgical risk of bleeding?
A) Bleeding time
B) PLT function analyzer-100 analysis
C) PLT aggregation studies
D) Medical history, including outcomes of previous surgical procedures
E) PT and partial thromboplastin time.

Answer: D

13. A 42-year-old woman presents with acute onset of headache and neck pain vertigo, nausea
and vomiting. Neurological examination reveals left nystagmus, left Horner syndrome and
absent left gag reflex. Left appendicular ataxia and anesthesia to pin prick in the left face and
right arm and leg. Which of the following is the most likely diagnosis?
A) MCA
B) PCA
C) ACA
D) dissection of aorta
E) Vertebral artery dissection

Answer: E

14. A 30-year-old man is evaluated for a thyroid nodule. The patient reports that his father died
from thyroid cancer and that a brother had a history of recurrent renal stones. Blood calcitonin
concentration is 2000 pg/mL (normal is less than 100); serum calcium and phosphate levels are
normal. Before referring the patient to a surgeon, the physician should do which of the following?
A) Obtain a liver scan
B) Perform a calcium infusion test
C) Measure urinary catecholamines
D) Administer suppressive doses of thyroxine and measure levels of thyroid stimulating
hormone
E) Treat the patient with radioactive iodine

Answer: C

15. All of the following vaccine can be given to pregnant lady except:
A) Flu vaccine
B) H1N1
C) Hepatitis B vaccine
D) Rubella vaccine
E) Tetanus vaccine

Answer: D

16. A 78-year-old man is evaluated in the hospital for poor glycemic control before undergoing
femoral popliteal bypass surgery. He has been on the vascular surgery ward for 3 weeks with a
non-healing foot ulcer. The patient has an extensive history of arteriosclerotic CV disease,
including PVD, and a 20 year history of DM 2, his most recent Hb A1c obtained 2 months before
admission was 8.9%, his diabetes regimen consists of glibenclamide 15 mg/day. While in the
hospital his plasma glucose levels have generally been in the 200 to 250 mg/dl range he is eating
well. In addition to stopping glibenclamide, which of the following is the most appropriate
treatment for this patient?
A) Basal insulin and rapid acting insulin before meals.
B) Insulin infusion
C) NPH insulin twice daily
D) Sliding scale regular insulin
E) Insulin glargine once daily

Answer: A

17. One of flowing statement regarding COPD is true:


A) Most smokers develop COPD during their life
B) Tiotropium is as effective as smoking cessation in reduction the rate of decline in FEV1.
C) Long term O2 therapy improves survival in all COPD treatment with FEV1 less than 50% of
predicted.
D) Rehabilitation may improve survival in patients with severe COPD
E) Inhaled corticosteroids decreases exacerbation rate of COPD patients

Answer: E

18. What is the best laboratory goal to use to guide fluid resuscitation in acute pancreatitis?
A) Hematocrit
B) Blood urea nitrogen
C) Serum creatinine
D) Amylase
E) Lipase

Answer: A

19. A 62-year-old female with a history of a recent pulmonary embolus presents to your office
for follow-up on anticoagulation treatment. She takes warfarin on a daily basis. She reports that
for the last week she has noticed mild rectal bleeding and multiple bruises over the extremities
with minimal trauma. She is comfortable appearing with normal vital signs and is not orthostatic.
You ordered a stat CBC and PT/INR which revealed a mildly decreased Hgb at 11 g/dL and an
elevated INR of 7. Which of the following would be the most appropriate intervention?
A) Fresh frozen plasma
B) Withhold warfarin
C) Intravenous vitamin K
D) Reduce warfarin dose
E) Oral vitamin K

Answer: E

20. Which of the following produces the greatest increase in bone mineral density (BMD) in
patients with osteoporosis?
A) Estrogen
B) Calcitonin
C) Alendronate
D) Teriparatide
E) Raloxifene

Answer: D

21. A 34-year-old woman was found to have a 2-cm right thyroid nodule at the time of a well
woman examination. The remainder of the thyroid was palpably normal and there were no lymph
nodes palpable. There was no history of thyroid disease or radiation therapy to her head or neck.
She was clinically euthyroid. Thyroid-stimulating hormone (TSH) was normal. Which of the
following tests would be the most useful in establishing a specific diagnosis?
A) Ultrasound of the thyroid
B) Nuclear scan of the thyroid
C) Thyroid antibody studies
D) Fine needle aspiration of the nodule
E) CT of the neck

Answer: D

22. A slim, healthy 30-year-old woman is scheduled for a dental prosthodontic procedure and
was sent for medical evaluation of a known history of mitral valve prolapse (MVP). The patient
is a highly active individual and denies palpitations, chest pain, or shortness of breath. She
admits to having a family history of heart disease, notably her father, who had died of a heart
attack in his forties, and her mother, who had mitral valve prolapse. On physical examination,
the patient is comfortable and has normal vital signs. Auscultation of the heart reveals a normal
S1 and S2 and a prominent midsystolic click, which is accentuated in the standing position. No
systolic murmur is appreciated. What is your overall assessment and plan for this patient?
A) Get an echocardiogram to evaluate mitral valve motion and blood flow prior to clearing her
for the procedure
B) Prescribe empiric antibiotics for endocarditis prophylaxis and clear her for the procedure
C) Get a cardiology consultation prior to medical clearance because the patient has a significant
family history of heart disease
D) Clear her for the procedure without endocarditis prophylaxis
E) Clear her for the procedure with endocarditis prophylaxis

Answer: D

23. A 35-year-old man complains of chest pain, which began following the use of cocaine 30
minutes prior to arrival. The patient describes severe substernal chest pressure, radiating to the
left arm and jaw. It is associated with shortness of breath. Initial vital signs: temperature 100.4°F,
heart rate 120/min, respiratory rate 20/min, blood pressure 185/100 mm Hg, pulse oximetry 98%
on room air. An ECG is consistent with acute myocardial infarction. A drug which is
contraindicated is:
A) Oxygen.
B) Aspirin.
C) Lorazepam.
D) Bisoprolol.
E) Morphine

Answer: D

24. A 55-year-old man with a history of alcoholism complains of more than a month of malaise,
low-grade fever, and a productive cough with greenish sputum tinged with blood. Examination
shows periodontal disease with bad breath and clubbing of fingers. On chest x-ray, there is a 2
cm cavity with an air–fluid level in the posterior segment of the right upper lobe. Sputum smear
shows many neutrophils and a variety of bacteria. Appropriate treatment includes:
A) Isolate the patient and initiate a four-drug antituberculosis treatment.
B) Start intravenous clindamycin.
C) Refer the patient to a dentist for periodontal care.
D) Schedule a bronchoscopy for the next day.
E) Start administration of methicillin and tobramycin

Answer: B

25. A previously healthy 54-year-old man presents to the emergency department complaining of
chest pain. His ECG shows an acute inferior wall myocardial infarction. His blood pressure is
90/60 mm Hg. On physical examination, he has jugular vein distention and clear lungs. You
should treat him immediately with which of the following:
A) Intravenous fluids.
B) Norepinephrine.
C) Dopamine.
D) Nesiritide.
E) Nitroprusside

Answer: A

26. Inhalant abusers (“huffers” and “baggers”) are at risk for:


A) Ventricular dysrhythmias.
B) Acute lung injury.
C) Hypoglycemia.
D) Acidosis.
E) Kidney failure

Answer: A

27. A 37-year-old woman has a 2-week history of intermittent headache and general malaise.
Over the last 24 hours, she has developed back pain, hematuria, vomiting, fever, and confusion.
She denies recent travel or insect bites. There is no history of dysuria, urgency, frequency, or
kidney stones. Her oral temperature is 38°C and her heart rate is 100/min. Physical findings
include pale conjunctivae, borderline tachycardia, bilateral costovertebral tenderness, and several
purpuric skin lesions. Her urine dipstick is strongly positive for hemoglobin, but negative for
nitrites and leukocyte esterase. The test that will most likely reveal the correct diagnosis is:
A) Intravenous pyelogram.
B) CBC with differential and peripheral smear.
C) Complete urinalysis with microscopic examination.
D) Blood cultures.
E) Liver function tests

Answer: B

28. A 52-year-old man complains of severe right knee pain. He recalls no trauma, but attended a
wine tasting party last weekend. He denies past medical history and is afebrile with normal vital
signs. Physical examination reveals a swollen, red, painful right knee. Radiograph shows an
effusion, but no bony erosions. You perform arthrocentesis and obtain 27 mL of cloudy straw-
colored fluid, which you send to the laboratory for studies. The cell count is 50,000 WBCs/mm3
with 85% PMNs, glucose 120 mg/dL (serum 130), and protein 3.5 g/dL. The Gram stain shows
numerous WBCs, but is negative for organisms. Evaluation for crystals shows numerous needle-
shaped crystals with negative birefringence. A good treatment plan would be:
A) Oral colchicine 0.5 mg every hour until symptoms abate, GI toxicity develops or the
maximum dose of 6 mg in 24 hours is reached.
B) Subcutaneous colchicine 1 mg plus oral probenecid 250 mg bid for 1 week.
C) Sublingual colchicine 2 mg followed by oral steroids tapered over 5 days.
D) Intra-articular triamcinolone injection.
E) Oral allopurinol 500 mg tid for 1 week

Answer: A

29. A 35-year-old woman presents to her gynecologist with complaints of burning on urination
for the past 2 days. Dipstick test of her urine demonstrates marked positivity for leukocyte
esterase, but no reactivity for nitrite. Urine culture later grows out large numbers of organisms.
Which of the following bacteria are most likely to be responsible for this patient's infection?
A) Enterobacter sp.
B) Enterococcus faecalis
C) Escherichia coli
D) Klebsiella pneumoniae
E) Pseudomonas aeruginosa

Answer: B

30. A 23 year old single woman referred with 3-month history of weight loss and heat
intolerance. On examination pulse is 120 beat/min regular, blood pressure 120/72 mmHg, weight
58 kg, height 165 cm. she had diffuse goiter with bilateral exophthalmos. Investigations: serum
free T4 3.9 ng/dl (N: 0.9-2.0), serum TSH 0.001 mu/l (N: 0.4-4.0), TSH receptor antibody 8
U/L(N:<2). She was started on methimazole 30 mg/day. Six weeks later she developed severe
migratory arthralgia associated with the presence of palpable purpura over both legs and dermal
infarcts in the finger tips. Laboratory studies shows ESR 54 mm/h, urine showed microscopic
hematuria, ANCA titer was 1:640, antimyeloperoxidase antibodies 112U/l (N:0-9),
antiproteinase 3 antibodies 6.4 U/l (N:0-3.5).
The best action is:
A) Continue methimazole and add NSAID.
B) Continue methimazole and add prednisolone 40 mg per day.
C) Substitute propylthiouracil for methimazole.
D) Refer the patient for definitive therapy with radioactive iodine.
E) No intervention

Answer: D

31. A 16-year-old girl consults you because of progressive pallor and poor exercise tolerance.
She has had dysfunctional uterine bleeding for several months. Her haemoglobin is 6.5 g/dl.
Which one of the following would be inconsistent with iron-deficiency anaemia in this girl?
A) Thrombocytosis
B) Low serum iron
C) Low serum ferritin
D) Reticulocyte percentage 10%
E) Transferrin saturation 9%

Answer: D

32. A 48-year-old auto mechanic presents to the clinic with complaints of many years of “pins
and needles” in his left hand that initially occurred only while working but have worsened
substantially. He claims the pain wakes him almost every night. On examination, marked
weakness and wasting of the left hand muscles are evident. What is the patient’s most likely
diagnosis?
A) Amyotrophic lateral sclerosis
B) Angina
C) Carpal tunnel syndrome
D) Multiple sclerosis
E) Myasthenia gravis

Answer: D

33. Three months following laparoscopic cholecystectomy, a 50-yearold woman presents with
right upper abdominal quadrant pain, excessive flatulence and intolerance to fatty food.
Abdominal ultrasound and ERCP fail to detect any residual stone or biliary stricture. What is the
cause of this woman’s presentation?
A) Small common bile duct stones
B) Cholangiocarcinoma
C) Peri-ampullary adenocarcinoma
D) Post-cholecystectomy syndrome
E) Choledochal cyst

Answer: D

34. All of the following clinical findings are consistent with severe mitral stenosis except
A) atrial fibrillation
B) opening snap late after S2
C) pulmonary vascular congestion
D) pulsatile liver
E) right-ventricular heave

Answer: B

35. A 50-year-old man has a blood pressure of 160/105 mm Hg, which is repeatedly confirmed
to be high. He has migraine without aura for which he takes sumatriptan during pain episodes.
Which is the best antihypertensive medication for this patient to be given initially?
A) Amlodipin
B) Propranolol
C) Hydrochlorothiazide
D) Losartan
E) Alpha methyldopa

Answer: B

36. All of the following represent examples of hypothalamic-pituitary negative feedback


EXCEPT:
A) Cortisol on the CRH-ACTH axis
B) Gonadal steroids on the GnRH-LH/FSH axis
C) IGF-1 on the growth hormone–releasing hormone (GHRH)-GH axis
D) Renin-angiotensin-aldosterone axis
E) Thyroid hormones on TRH-TSH axis

Answer: D

37. You are investigating the cause for a patient’s anemia. He is a 50-year-old man who was
found to have a hematocrit of 25% on routine evaluation. His hematocrit was 47% 1 year ago.
Mean corpuscular volume is 80, mean corpuscular hemoglobin concentration is 25, mean
corpuscular hemoglobin is 25. Reticulocyte count is 5%. Review of the peripheral blood smear
shows marked numbers of polychromatophilic macrocytes. Ferritin is 340 μg/L. What is the
cause of this patient’s anemia?
A) Defective erythroid marrow proliferation
B) Extravascular hemolysis
C) Intravascular hemolysis
D) Iron-deficiency anemia
E) Occult gastrointestinal bleeding

Answer: A

38. A 23-year-old Japanese man attends a party where he drinks three glasses of wine. In a short
period of time, he develops facial erythema and experiences severe facial flushing. Which of the
following is the most likely diagnosis?
A) Alcohol dehydrogenase deficiency
B) Glucoronyl tranferase deficiency
C) Aldehyde dehydrogenase deficiency
D) Angioedema
E) Photosensitivity reaction

Answer: C

39. A 46-year-old woman with mitral stenosis presents with dense right-sided hemiplegia and
global aphasia. Her radial pulse is rapid and irregular. Brain CT scan shows cerebral infarction
and she is admitted to the neurology ward to receive treatment for embolic stroke. Which one of
the following arteries is likely to be occluded?
A) Main stem of the left middle cerebral artery
B) Lower posterior branch of the right middle cerebral artery
C) Left anterior cerebral artery
D) Top of the basilar artery
E Left vertebral artery

Answer: A

40. Which of the following is consistent with a diagnosis of subacute thyroiditis?


A) A 38-year-old female with a 2-week history of a painful thyroid, elevated T4, elevated T3,
low TSH, and an elevated radioactive iodine uptake scan
B) A 42-year-old male with a history of a painful thyroid 4 months ago, fatigue, malaise, low
free T4, low T3, and elevated TSH
C) A 31-year-old female with a painless enlarged thyroid, low TSH, elevated T4, elevated free
T4, and an elevated radioiodine uptake scan
D) A 50-year-old male with a painful thyroid, slightly elevated T4, normal TSH, and an
ultrasound showing a mass

Answer: B

41. A 65-year-old obese man presents with recurrent episodes of cough. His wife said his
coughing attacks disturb her sleep almost every night. He noticed that his voice is hoarse in the
morning. He has never smoked and has no history of cardiac or respiratory problems. There is no
history of allergy and the chest X-ray is normal. The procedure most likely to yield crucial
diagnostic information is
A) Pulmonary function tests
B) 24-hour pH monitoring of the lower oesophagus
C) High-resolution CT scan of the chest
D) Barium swallow
E Indirect laryngoscopy

Answer: B

42. A 38-year-old man has a six-month history of diarrhoea, abdominal pain and tenesmus.
Flexible sigmoidoscopy reveals proctitis. A biopsy specimen shows acute and chronic
inflammation. Each of the following statements is true about this disorder, except
A) There is an increased risk of gastrointestinal malignancy
B) Other family members are likely to develop the same illness
C) Steatorrhoea is likely to develop as a result of pancreatic insufficiency
D) The likelihood of renal stones will increase
E) The likelihood of gallstones will increase

Answer: C

43. A 56-year-old man is referred with dyspnoea and cough productive of putrid green sputum of
four weeks duration. The chest X-ray shows cavity formation in the left lower lobe. He has
advanced motor neurone disease. This clinical picture is most likely to be caused by infection
with
A) Legionella pneumophila
B) Mycoplasma pneumoniae
C) Streptococcus pneumoniae
D) Anaerobic bacteria
E) Common cold virus

Answer: D

44. A 50-year-old publican is referred with anuria. The initial renal function test showed the urea
at 45 mmol/I and the creatinine at 480 μmoI/l. A urine sample from a catheter was strongly
positive for myoglobin. This clinical picture can develop as a result of each of the following
clinical situations except
A) Alcohol ingestion
B) Hypokalaemia
C) Barbiturate overdose
D) Hypophosphataemia
E) Hypocalcaemia

Answer: E

45. A 75-year-old woman is admitted with left lower lobe pneumonia. She was given a course of
cephalosporin antibiotics and she improved. However, at the end of the course she started to
have diarrhoea, passing a watery stool. Investigations confirmed the diagnosis of
pseudomembranous colitis. Each of the following oral medications is beneficial in treating this
condition, except
A) Vancomycin
B) Bacitracin
C) Colestyramine
D) Metronidazole
E) Gentamicin

Answer: E

46. A 38-year-old woman has dyspnoea on exertion and orthopnoea. Examination reveals a loud
first heart sound, a diastolic rumble, and a large v wave in the jugular pulse. The most likely
diagnosis is
A) Mitral stenosis and tricuspid regurgitation
B) Isolated mitral stenosis
C) Aortic regurgitation
D) Mitral regurgitation and tricuspid regurgitation
E) Tricuspid regurgitation

Answer: A
47. A 40-year-old man has a six-month history of increasing shortness of breath and wheeze. The
full blood count shows an increased absolute eosinophil count. Chest X-ray and a CT scan of the
chest show right apical bronchiectasis. Which one of the following is the most likely diagnosis?
A) Loeffler's syndrome
B) Allergic bronchopulmonary aspergillosis
C) Polyarteritis nodosa
D) Nitrofurantoin hypersensitivity
E) Strongyloides stercoralis infection

Answer: B

48. A 22-year-old woman has a three-month history of fatigue of insidious onset, poor appetite
and athralgia. She has no previous history of liver disease. She takes thyroxine replacement
therapy, Physical examination reveals spider naevi and hepatomegaly, The ALT is five-times
higher than the normal. Her disease is most likely be associated with
A) Antimitochondrial antibodies
B) Low serum caeruloplasmin levels
C) Anti-parietal cell antibodies
D) Anti-smooth muscle antibodies
E) Hepatitis B surface antigen (1-HbsAg )

Answer: D

49. A 35-year-old woman has slurred speech and a progressive gait disturbance. Magnetic
resonance imaging (MRI) shows several focal abnormalities in the periventricular areas, normal
sized ventricles and no space occupying lesion. The cerebellum was also normal. The most likely
diagnosis is
A) Motor neurone disease
B) Friedreich's ataxia
C) Multiple sclerosis (MS)
D) Syphilis
E) Huntington's disease

Answer: C

50. An 85-year-old man is referred with a three-month history of dysphagia. Barium swallow
showed a filling defect in the oesophagus. Gastro-oesophagoscopy and biopsy confirmed the
presence of squamous cell carcinoma of the oesophagus. Each of the following conditions might
be associated with this tumour, except
A) Achalasia
B) Smoking
C) Tylosis
D) Barrett's oesophagus
E) Head and neck cancer

Answer: D
MULTIPLE CHOICE QUESTIONS-4

1. A 92-year-old man is referred from his nursing home for evaluation of lethargy. Examination
is unrevealing, but laboratory results are significant for a serum sodium level of 118 meq/L
(normal, 135 148). Serum osmolality is 260, urine osmolality is 450, and urine sodium is 80.
Which of the following is the most likely cause of this patient's lethargy?
A) hyperglycemia
B) hyperlipidemia
C) hyperproteinemia
D) SIADH
E) diabetes insipidus

Answer: D

2. Which one of the following is a recognized feature of polymyalgia rheumatica?


A) Weakness of distal muscle groups
B) Elevated serum creatine phosphokinase activity
C) An association with bronchial carcinoma
D) Weight loss
E) A peak incidence in the fourth decade of life

Answer: D

3. A 17-year-old girl notes an enlarging lump in her neck. On examination, her thyroid gland is
twice the normal size, firm to rubbery, multilobular, nontender, and freely mobile. There is no
adenopathy. Family history is positive for both hypo- and hyperthyroidism. Her serum
triiodothyronine (T3) and thyroxine (T4) levels are low normal, and serum thyroid-stimulating
hormone (TSH) is high normal. Technetium scan shows nonuniform uptake. Serum and
antithyroglobulin titer is strongly
positive. What will thyroid biopsy of this patient most likely disclose?
A) giant cell granulomas and necrosis
B) polymorphonuclear cells and bacteria
C) diffuse fibrous replacement
D) lymphocytic infiltration
E) parafollicular cells

Answer: D

4. A 60-year-old man presents with a nonproductive cough for a week and generalized malaise.
He also has noted some abdominal pain associated with diarrhea for the past few days. His
temperature is 101.5°F and c linical examination is unremarkable. ACXR shows a left lower lobe
infiltrate. His urinalysis shows 50 RBCs, and his BUN (30) and creatinine (1.6) are both mildly
elevated. In light of the extrapulmonary symptoms and signs, which of the following is the most
likely cause of his pneumonia?
A) Pseudomonas aeruginosa
B) S. aureus
C) H. influenzae
D) S. pneumoniae
E) Legionella

Aswer: E

5. A 78-year-old woman comes to your primary care office practice with her son who is
concerned about changes in her mood. She is less interested in going out to dinner and does not
want to visit family or friends. Her language skills seem to have deteriorated over the last few
years and her memory is not as sharp. Her gait and motor strength are normal. Which of the
following is the most likely diagnosis?
A) Parkinson's disease
B) anxiety disorder
C) meningioma
D) Alzheimer's disease
E) dysthymia

Answer: D

6. A 30-year-old woman who has been human immunodeficiency virus (HIV) positive for 4
years was recently diagnosed with AIDS. Which of the following immunologic abnormalities
would be expected?
A) increased numbers of CD4+ (helper) T cells
B) decreased number of CD8+ (suppressor) T cells
C) cutaneous anergy to usual skin test antigens
D) normal B-cell function
E) increased natural killer cell function

Answer: C

7. A 60-year-old previously healthy man presents with massive rectal bleeding. Which of the
following is the most likely diagnosis?
A) diverticulosis of the colon
B) ulcerative colitis
C) external hemorrhoid
D) ischemic colitis
E) carcinoma of the colon

Answer: A

8. A 59-year-old woman who lives independently and had been healthy, presents to the
emergency department with cough and fever. She related she was well until 2 days before when
she noted onset of fever, chills, and cough productive of yellow sputum. On examination, you
note a tired appearing woman with BP of 160/90, pulse of 105, and respiratory rate of 32. You
start her on ceftriaxone and azithromycin and admit her to the hospital. Which of the following
factors is a poor prognostic sign in community acquired pneumonia?
A) age less than 60
B) systolic BP = 160 mmHg
C) leukocytosis = 15,000
D) respiratory rate = 32
E) mycoplasma pneumonia infected

Answer: D

9. You see a 19-year-old Caucasian man in your clinic who presents with a history of transient
jaundice. On direct questioning, you ascertain that the jaundice is noticeable after periods of
increased physical activity and subsides after a few days. The patient has no other symptoms and
physical examination is unremarkable. Full blood count is normal (with a normal reticulocyte
count) and liver function tests reveal a bilirubin of 37 μmol/L. The most appropriate management
is:
A) Reassure and discharge
B) Start on a course of oral steroids
C) Request abdominal ultrasound
D) Request MRCP
E) Refer to Haematology

Answer: A

10. You see a 56-year-old man who was admitted for an elective upper GI endoscopy due to
longstanding GORD which has failed to improve on antacids and PPIs. Your registrar suspects
that this patient may have Barrett’s oesophagus and asks you to define what this is. The most
appropriate description of Barrett’s oesophagus is:
A) Metaplasia of the squamous epithelium of the lower third of the oesophagus to columnar
epithelium
B) Metaplasia of the columnar epithelium of the upper third of the oesophagus to squamous
epithelium
C) Metaplasia of the columnar epithelium of the lower third of the oesophagus to squamous
epithelium
D) Metaplasia of the squamous epithelium of the upper third of the oesophagus to columnar
epithelium
E) Metaplasia of the squamous epithelium of the middle third of the oesophagus to columnar
epithelium

Answer: A

11. You see a 48-year-old lorry driver, who presents to you with a three-month history of
heartburn after meals which has not been settling with antacids and PPIs. You suspect that the
patient has a hiatus hernia. The most appropriate investigation for diagnosing a hiatus hernia is:
A) Computer tomography (CT) scan
B) Chest x-ray
C) Upper GI endoscopy
D) Barium meal
E) Ultrasound

Answer: D

12. You see a 47-year-old man in clinic with a three-month history of epigastric dull abdominal
pain. He states that the pain is worse in the mornings and is relieved after meals. On direct
questioning, there is no history of weight loss and the patient’s bowel habits are normal. On
examination, his abdomen is soft and
experiences moderate discomfort on palpation of the epigastric region. The most likely diagnosis
is:
A) Gastric ulcer
B) Gastro-oesophageal reflux disease (GORD)
C) Duodenal ulcer
D) Gastric carcinoma
E) Gastritis

Answer: C

13. A 60-year-old farmer presented with non specific symptoms of chronic cough and
progressive dyspnea for the last six years. The patient had interstitial pneumonitis. Chest x-ray
revealed reticulonodular infiltrate associated with honey-combing of the lung. Pulmonary
function studies showed a restrictive pattern with loss of lung volumes, impaired diffusion
capacity, decreased compliance and exercise induced hypoxia. This farmer’s symptoms are most
likely secondary to the following environmental hazard:
A) The farmers cheese processing plant
B) The presence of spores of bacillus anthracis in the farmer’s farm
C) The harvesting of crops in rainy weather
D) The presence of asbestos in the water tanks.
E) The presence of antigenic detergents

Answer: C

14. Which of the following antiarrhythmic agents may promote AF?


A) Adenosine
B) Quinidine
C) Propafenone
D) Amiodarone
E) Atenolol

Answer: A

15. Torsades de pointes is characterized by all of the following except:


A) Results from triggered activity (early afterdepolarizations) that occurs during
phase 2 or 3 of the cardiac action potential
B) Prolonged QT interval
C) Exacerbation by bradycardia with short-long coupling intervals
D) Polymorphic VT
E) Often provoked during amiodarone administration

Answer: E

16. A 68-year-old patient is seen for a general examination. Current


recommendations for immunizations include
A) Tetanus booster every 5 years
B) Influenza vaccination yearly
C) Pneumococcal vaccination yearly
D) Hepatitis booster every 5 years
E) Meningococcal vaccination

Answer: B

17. Which of the following statements about familial periodic paralysis is true?
A) It is an autosomal-recessive transmitted disorder.
B) It involves disturbances of potassium regulation.
C) It is associated with permanent muscle weakness.
D) It is aggravated by administration of acetazolamide.
E) It most commonly affects the elderly

Answer: B

18. A 52-year-old man complains of severe right knee pain. He recalls no trauma, but attended a
wine tasting party last weekend. He denies past medical history and is afebrile with normal vital
signs. Physical examination reveals a swollen, red, painful right knee. Radiograph shows an
effusion, but no bony erosions. You perform arthrocentesis and obtain 27 mL of cloudy straw-
colored fluid, which you send to the laboratory for studies. The cell count is 50,000 WBCs/mm3
with 85% PMNs, glucose 120 mg/dL (serum 130), and protein 3.5 g/dL. The Gram stain shows
numerous WBCs, but is negative for organisms. Evaluation for crystals shows numerous needle-
shaped crystals with negative birefringence. A good treatment plan would be:
A) Oral colchicine 0.5 mg every hour until symptoms abate, GI toxicity develops or the
maximum dose of 6 mg in 24 hours is reached.
B) Subcutaneous colchicine 1 mg plus oral probenecid 250 mg bid for 1 week.
C) Sublingual colchicine 2 mg followed by oral steroids tapered over 5 days.
D) Intra-articular triamcinolone injection.
E) Oral allopurinol 500 mg tid for 1 week

Answer: A

19. A 38-year-old woman is brought to the emergency department by her spouse because of
decreased mental status. She had knee surgery 2 days ago and was prescribed oral oxycodone for
pain. Her spouse notes that she finished the entire 7-day supply during that day. He denies any
seizure activity.They have no other drugs or medications in the house. She is afebrile with blood
pressure of 130/75 mmHg, heart rate of 70 bpm, respiratory rate of 4 breaths/min, and SaO2 of
85% on room air. She barely responds to painful stimuli but moves all four extremities equally.
Which of the following medications is most likely to improve her mental status?
A) Albuterol
B) Alvimopan
C) Flumazenil
D) N-Acetylcysteine
E) Naloxone

Answer: E

20. A 45-year-man with longstanding type 1 diabetes mellitus complains of pain in his feet and
ankles that has been present for over a year. All of the following are consistent with neuropathic
pain due to diabetes EXCEPT:
A) Burning pain
B) Electric shock quality
C) Exacerbated by light touch
D) Pain referred to scrotum
E) Tingling

Answer: D

21. Which of the following sets of drug–drug interaction and mechanism is accurately described?
A) Ibuprofen and warfarin: increased risk of GI bleeding; ibuprofen inhibition of CYP2C9
B) Sotalol and furosemide: increased risk of QT prolongation and torsades de pointes;
furosemideinduced inhibition of CYP3A4
C) Sildenafil and sublingual nitroglycerin: increased risk of hypotension; sildenafil inhibition of
the phosphodiesterase type 5 isoform that inactivates cyclic guanosine monophosphate
D) Ritonavir and lovastatin: increased risk of myotoxicity; ritonavir inhibition of CYP2C19
E) Allopurinol and azathioprine: increased risk of blood dyscrasias; allopurinol inhibition of
Pglycoprotein

Answer: C

22. Mr. Jonas is a 47-year-old truck driver with a history of HIV, hypertension, coronary artery
disease, atrial fibrillation, and ischemic cardiomyopathy. He is on antiretroviral therapy. He
presents today complaining of a new rash on his chest and axilla, which you astutely diagnose as
tinea corporis. You
would like to prescribe a course of oral ketoconazole for therapy. You should consider dose
adjustment for all of the following medicines that he is already taking EXCEPT:
A) Carvedilol
B) Lovastatin
C) Mexiletine
D) Ritonavir
E) Saquinavir
Answer: A

23. A 24-year-old woman who has tested positive for human immunodefi ciency virus (HIV)
presents to your officewith mild fever and symptoms of an upper respiratory
tract infection. She is currently in her 26th week of pregnancy and has a recent positive
tuberculin skin test (TST) conversion with a 22-mm induration. Additionally, she has genital
ulcerative lesions that test positive for herpes simplex virus. Which antimicrobial agent
should you avoid prescribing for this patient?
A) Acyclovir
B) Amoxicillin
C) Tetracycline
D) Azithromycin
E) Isoniazid

Answer: C

24. Which of the following characteristics of Mycobacterium tuberculosis can be used to


distinguish it from Mycobacterium bovis ?
A) Nonchromogenic pigmentation
B) Positive pyrazinamidase test
C) Negative 68°C catalase test
D) Positive nucleic acid probe for M tuberculosis complex
E) Negative arylsulfatase test

Answer: B

25. A 65-year-old man presents with a left footdrop. On examination,


the left anterior tibialis, extensor hallucis longus, and posterior tibialis muscles are weak. The
strength of the quadriceps, gastrocnemius, and gluteus maximus muscles is normal. Stretch
reflexes of the quadriceps and ankle muscle are normal. To which structure does the lesion best
localize?
A) Peroneal nerve
B) L4 spinal root
C) L5 spinal root
D) Sciatic nerve
E) Thoracic spinal cord

Answer: C

26. A 55-year-old male who has a long history of marginally-controlled hypertension presents
with gradually increasing shortness of breath and reduced exercise tolerance. His physical
examination is normal except for a blood pressure of 140/90 mm Hg, bilateral basilar rales, and
trace pitting edema. Which one of the following ancillary studies would be the preferred
diagnostic tool for evaluating this patient? (check one)
A) 12-lead electrocardiography
B) Posteroanterior and lateral chest radiographs
C) 2-dimensional echocardiography with Doppler
D) Radionuclide ventriculography
E) Cardiac MRI

Answer: C

27. Which one of the following extraintestinal manifestations of inflammatory bowel disease do
not parallel the course of intestinal inflammation and do not improve in parallel with
improvement in intestinal symptoms?
A) peripheral arthritis
B) apthous ulcers
C) spondylitis and sacroiliitis
D) erythema nodosum
E) uveitis and iritis

Answer: C

28. An 18-year-old Asian woman is being treated for hepatitis B. Prior to therapy she was found
to have ALT 198 U/L, AST 91 U/L, normal bilirubin, albumin, and prothrombin time. Liver
biopsy results showed chronic hepatitis B, grade 3, stage 3. After 12 weeks of therapy, serum
ALT is found to have increased to 1,082 U/L, bilirubin 2.1 mg/dL but albumin and prothrombin
time remain normal. Apart from some fatigue, the patient is tolerating interferon well. The best
course of action is:
A) check for antinuclear antibodies and total immunoglobulin level in serum and consider
instituting corticosteroid therapy
B) stop interferon
C) recheck lab work again in 2 weeks time
D) add lamivudine to the regimen
E) ask the patient to skip three scheduled doses of interferon

Answer: C

29. A 53-year-old man presented with hypertension of 150/110 mmHg. He is generally


asymptomatic and has no previous medical history of note. He is a smoker of 5 cigarettes daily
and drinks modest quantities of alcohol. He takes no prescribed medications. Examination
reveals a BMI of 33.5 kg/cm2 but nil else. Investigations: Serum sodium 146 mmol/l (NR 133-
145); Serum potassium 3.2 mmol/l (NR 3.5 - 5); Urinary potassium excretion 42 mmol/l (NR
less than 30). What is the likely diagnosis?
A) adrenocortical adenoma
B) Bartter's syndrome
C) Liddle's syndrome
D) liquorice ingestion
E) pheochromocytoma

Answer: A
30. A female patient aged 30 has a 5 years history of difficulty getting upstairs and out of a low
chair and mild upper limb weakness but no pain. There is no family history. She presented with
severe type 2 respiratory failure. EMG showed evidence of myopathy.
The most likely diagnosis is:
A) Polymyositis
B) Inclusion body myositis
C) Acid maltase deficiency
D) Miller-Fisher Syndrome
E) Lambert-Eaton Myasthenic syndrome

Answer: C

31. A 28-year-man presents with a 1-week history of fever, headache, malaise, limb aching and
constipation. Examination reveals a pulse rate of 80 beats/minute and high fever. His fever is
gradually rising in a stepladder fashion. The anti-O and anti-H titres are high in the Widal test.
Which one of the following is uncommon during the second week of this man’s illness?
A) Cough
B) Diarrhoea
C) Erythematous spots on the upper abdomen
D) Melaena
E) Splenomegaly

Answer: D

32. Which one of the following diseases shares many clinical characteristics with sarcoidosis?
A) Lymphangioleiomyomatosis
B) Berylliosis
C) Asbestosis
D) Alpha-1 antiprotease deficiency
E) Pulmonary alveolar proteinosis

Answer: B

33. A 19-year-old man presents to the emergency department with 2 days of ascending weakness.
He had diarrhea 3 weeks earlier. He looks comfortable. On examination, you find moderate
weakness in all limbs, with normal strength in facial muscles. You suspect Guillain-Barré
syndrome and recommend admission to Neurology. What should be done first?
A) Spinal tap to look for albuminocytologic dissociation
B) Call for an emergency EMG to verify diagnosis
C) Send the patient for a whole spine MRI to rule out cord compression
D) Obtain pulmonary function tests, including FVC
E) Start high-dose steroids and then move to the floor

Answer: D
34. A 54-year-old woman is started on medical treatment following a diagnosis of acromegaly.
Which one of the following is indicative of active disease?
A) Excessive sweating
B) Polyuria
C) Abdominal pain
D) Arthropathy
E) Goitre

Answer: A

35. Which of the following statements regarding hepatitis B infection in pregnancy is true?
A) Most women of childbearing age with chronic hepatitis B have a high risk of the development
of complications of their disease during gestation.
B) Maternal-fetal transmission is responsible for most cases of hepatitis B worldwide.
C) Mothers who test negative for the hepatitis B e-antigen cannot transmit the virus to their
fetuses.
D) Women with hepatitis B can be treated with interferon during pregnancy.
E) Women with hepatitis B should not be treated with lamivudine during pregnancy.

Answer: B

36. All of the following conditions or drugs decrease theophylline clearance, resulting in
increased serum level of this drug EXCEPT:
A) Cimetidine
B) Cirrhosis
C) Allopurinol
D) Rifampin
E) Erythromycin

Answer: D

37. A 39-year-old man presents after tripping over his child’s toy and subsequently falling down
the stairs. The patient reports landing on his chest and now complains of pain over certain
regions of his right chest wall. He endorses some mild pain on deep inspiration. The patient is
otherwise healthy and denies alcohol or smoking. Physical examination demonstrates pain on
palpation over the sternum. A chest x-ray is ordered which confirms two right rib fractures. The
radiologist notes an incidental 2.25-cm coin-shaped lesion on the left upper lung. There is no
associated adenopathy or atelectasis noted. Which of the following is the most appropriate next
step with respect to the solitary lung lesion?
A) CT scan of the chest
B) Lung biopsy
C) Review of an old chest x-ray
D) Reassurance
E) C-guided biopsy

Answer: C
38. Which of the following renal pathologies is most likely to recur in a renal transplant patient?
A) Diabetic renal disease
B) Focal segmental glomerulosclerosis
C) Minimal change glomerulonephritis
D) Membranous glomerulonephritis
E) Mesangioproliferative glomerulonephritis

Answer: E

39. In which of the following patients is it appropriate to administer the vaccination against
herpes zoster?
A) A 35-year-old woman who has never had varicella-zoster infection who is 12 weeks pregnant
with her first child
B) A 54-year-old man who has never had varicella-zoster infection and is otherwise healthy
C) A 62-year-old man who had a car accident resulting in splenectomy
D) A 64-year-old woman with dermatomyositis-associated interstitial lung disease treated with
prednisone 20 mg daily and azathioprine 150 mg daily
E) A 66-year-old woman who was recently diagnosed with non-Hodgkin lymphoma

Answer: C

40. A 23-year-old pregnant woman in her first trimester develops hyperthyroidism. Which of the
following treatments is contraindicated?
A) thyroid surgery
B) propylthiouracil (PTU)
C) drugs that cross the placenta
D) radioactive iodine
E) glucocorticoids

Answer: D

41. An elderly patient who becomes acutely short of breath presents with the combination of
hypotension, elevated jugular venous pressure, and muffled heart sounds. This triad of symptoms
is most suggestive of
A) Chronic pericarditis
B) Chronic pericardial effusion
C) Cardiac tamponade
D) Dissecting aortic aneurysm
E) Right heart failure

Answer: C

42. A patient becomes markedly tetanic following a recent thyroidectomy. This symptom can be
rapidly reversed by the administration of
A) Vitamin D
B) Calcitonin
C) PTH
D) Plicamycin (mithramycin)
E) Calcium gluconate (CaG)

Answer: E

43. A 52-year-old female presents with nausea, fatigue, muscle weakness, and intermittent pain
in her left flank. Laboratory examination reveals an increased serum calcium and a decreased
serum phosphorus. The patient’s plasma parathyroid hormone levels are increased, but
parathyroid hormone–related peptide levels are within normal limits. Urinary calcium is
increased, and microhematuria is present. The patient’s abnormality is most likely caused by
A) Primary hyperparathyroidism
B) Primary hypoparathyroidism
C) Pseudohypoparathyroidism
D) Secondary hyperparathyroidism
E) Secondary hypoparathyroidism

Answer: A

44. A 55-year-old female given a general anesthetic for a surgical procedure develops
hyperthermia, hypertension, hyperkalemia, tachycardia, muscle rigidity, and metabolic acidosis.
Which of the following general anesthetics did she receive?
A) Ketamine
B) Midazolam
C) Thiopental
D) Propofol
E) Halothane

Answer: E

45. A patient with reduced VC, FRC, and RV is found to have a normal pH. A tentative
diagnosis of diffuse interstitial fibrosis is made. Which of the following characteristics are
consistent with this disease?
A) An increase in lung compliance
B) A decrease in respiratory rate
C) An increase in the V/Q ratio
D) A decrease in PaCO2
E) An increase in the FEV1/FVC ratio

Answer: E

46. An HIV-positive patient asks if you can tell him the chances of him progressing to
symptomatic AIDS. Which one of the following tests would be most useful?
A) CD4 lymphocyte count
B) HIV antibody test
C) HIV RT PCR
D) Neopterin
E) HIV p24 antigen

Answer: C

47. A 33-year-old female patient treated with haloperidol for a history of schizophrenia is seen in
the ED because of complaints of fever, stiffness, and tremor. Her temperature is 104°F, and her
serum creatine kinase (CK) level is elevated. What has occurred?
A) Overdose
B) Allergy
C) Neuroleptic malignant syndrome (NMS)
D) Tardive Dyskinesia
E) Parkinsonism

Answer: C

48. A 43-year-old male is recovering from an infectious disease and experiences a marked
instability in his blood pressure with episodes of spiking of blood pressure. After a series of
extensive examinations, it was concluded that this disorder was due to the effects of the
infectious agent upon a component of the peripheral nervous system. Logical sites where an
infectious agent could produce such an effect include the
A) Superior ganglia of cranial nerves IX and X
B) Geniculate and trigeminal ganglia
C) Otic and superior salivatory ganglia
D) Carotid sinus and aortic arch
E) Carotid and aortic bodies

Answer: D

49. A 25-year-old female suspected of having vitamin D–resistant rickets has decreased blood
phosphate levels. Aside from high-dose vitamin D and oral phosphate, an alternative therapeutic
approach might be the use of which of the following?
A) Estrogen
B) Pamidronate
C) Hydrochlorothiazide
D) Prednisone
E) Calcitrol

Answer: E

50. A 30-year-old female stored her contact lenses in tap water. She noticed deterioration of
vision and visited an ophthalmologist who diagnosed her with severe retinitis. Culture of the
water as well as vitreous fluid would most likely reveal
A) Naegleria
B) Pneumocystis
C) Acanthamoeba
D) Babesia
E) Entamoeba coli

Answer: C

MULTIPLE CHOICE QUESTIONS-5

1. A 38-year-old woman is referred to you by a neurologist who was seeing her for severe
headaches. His final diagnosis was stress headaches, but because a head MRI showed an empty
sella, the neurologist wanted you to see her. On questioning, you learn that she has 7 children
and all are healthy. When the first child was born, she experienced a difficult delivery and
considerable bleeding that required a transfusion. All subsequent deliveries were uneventful. The
review of systems was otherwise negative. Physical examination shows normally pigmented
abdominal striae. The blood pressure is 100/60. You order some blood tests and find that the
electrolytes, glucose, and TSH are normal. Which of the following is the most correct thing for
you to do next?
A) Diagnose Sheehan syndrome and begin replacement therapy.
B) Evaluate for pituitary hypofunctioning.
C) Evaluate for pituitary hyperfunctioning.
D) Refer her to a neurosurgeon for exploration of her pituitary gland.
E) Tell her that everything is normal and discharge her from your clinic

Answer: E

2. The condition that characteristically results in recurrent axillary and groin abscesses is:
A) Regional enteritis.
B) Diabetes mellitus.
C) Ulcerative colitis.
D) Hidradenitis suppurativa.
E) Acne conglobata

Answer: D

3. The most important initial therapy for a patient with toxic epidermal necrolysis (TEN) is:
A) Corticosteroids.
B) Antibiotics.
C) Anticoagulants.
D) Analgesics.
E) Crystalloids

Answer: E

4. In evaluating the skin lesions of patients with AIDS:


A) Kaposi sarcoma is common but seldom more than a cosmetic problem.
B) Candidiasis may occur but is easily treated with standard medications.
C) Tinea corporis is no more likely in these patients than in the general population.
D) Lichen planus of the oral mucosa is associated with the disease.
E) Seborrheic dermatitis-like eruptions are frequently found

Answer: A

5. A 57-year-old man comes to the emergency department because of excruciating pain in his
right big toe. He describes the pain as so severe that it woke him from a deep sleep. He has no
chronic medical conditions, does not take any medications, and denies any similar episodes in
the past. He admits to a few "drinking binges" over the past 2 weeks. His temperature is 38.1 C
(100.5 F), blood pressure is 130/90 mm Hg, and pulse is 80/min. Examination shows an
erythematous, warm, swollen, and exquisitely tender right great toe. The skin overlying the first
metatarsophalangeal joint is dark red, tense, and shiny. Synovial fluid analysis reveals negatively
birefringent, needle-shaped crystals within polymorphonuclear leukocytes (PMNs). Laboratory
studies show:
Serum Leukocytes........16,000/mm3; Uric acid...........15 mg/dL; Calcium.............9 mg/dL
Which of the following is the most appropriate pharmacotherapy?
A. Allopurinol
B. Ceftriaxone
C. Cyclooxygenase-2 inhibitors but in higher dosages than are typically used.
D. Probenecid
E. Sulfinpyrazone

Answer: C

6. A 28-year-old woman is diagnosed with lupus nephritis, World Health Organization (WHO)
type IV. She has a malar rash, diffuse arthritis, and edema. Her blood pressure is 190/110 mm
Hg. Her creatinine is 2.1 mg/dL with a blood urea nitrogen of 28 mg/dL. Her urine reveals 25 red
blood cells per hpf, and 3+ protein. One red blood cell cast is seen. A 24-hour urine collection
reveals a protein of 11 grams with a creatinine of 1 gram. Which of the following would be the
most appropriate management?
A) Oral azathioprine
B) Oral cyclophosphamide
C) Oral gold
D) Oral prednisone
E) Pulse IV cyclophosphamide

Answer: E

7. A 50-year-old man is evaluated for a routine physical examination. He was diagnosed with
rheumatoid arthritis 5 years ago, and his condition is well controlled with methotrexate and
hydroxychloroquine. He does not have fatigue, morning stiffness, or systemic symptoms. His
weight is stable. On physical examination, there are no joint deformities. There is minimal soft-
tissue swelling around the second and third metacarpophalangeal joints. Grip strength is normal,
but squeezing the forefeet elicits mild discomfort. On laboratory studies, hemoglobin is 14.2
g/dL (142 g/L), alanine and aspartate aminotransferase and alkaline phosphatase are normal, and
erythrocyte sedimentation rate is 65 mm/h. Which of the following is the most appropriate
management for this patient?
A. Prednisone, 10 mg daily
B. Prednisone, 40 mg daily
C. Anti–tumor necrosis factor therapy
D. CT scan of the chest, abdomen, and pelvis
E. No additional therapy

Answer: D

8. A family physician cares for a family consisting of a 45-year old husband, 43-year-old wife
and a 12-year-old daughter. The family reports that recently the 77-year-old maternal
grandmother who lived with them died after a prolonged respiratory infection. Autopsy
subsequently confirms that she had active pulmonary tuberculosis at the time of death. The
organism tested sensitive to all anti-tuberculosis drugs. In responding to the grandmother's illness,
which of the following is the most appropriate step in management?
A) Obtain leukocyte counts on all family members
B) Obtain sputum cultures for acid fast bacilli
C) Obtain chest computerized tomograms on all members
D) Place protein purified derivative (PPD) test on all members
E) Schedule bronchoscopy lavage for the adults

Answer: D

9. A patient with a history of papillary thyroid cancer is noted to


have an enlarged lateral neck lymph node on routine
examination. Which one of the following is the most sensitive and specific indicator of
metastatic disease within the enlarged lymph
node?
A. Neck ultrasound
B. Fine needle aspiration cytology
C. Stimulated serum thyroglobulin
D. PET‐CT scanning
E. Thyroglobulin wash‐out testing

Answer: D

10. A 60-year-old woman is establishing care in your clinic. She has occasional numbness and
tingling in her fingers and toes and has noted some numbness around her mouth, especially when
she is stressed or anxious. She had thyroid surgery for Graves’ disease about 2 years ago and
takes 100 µg of levothyroxine and 1 tablet of calcium daily. On exam, her blood pressure is
130/80, pulse 80, and she has cramping in her right forearm and fingers when the blood pressure
cuff is attached. Based on this history and exam, which of the following is most likely?
A) Calcium 8.5, PTH 65, PO4 4.5
B) Calcium 9.5, PTH 35, PO4 4.0
C) Calcium 6.0 (8.5-10.5), PTH 2 (10-65), PO4 6.0 (2.7-4.5)
D) Calcium 10.8, PTH 108, PO4 2.3
E) Calcium 8.0, PTH 98, PO4 2.1

Answer: C

11. Which of the following sets of drug–drug interaction and mechanism is accurately described?
A. Ibuprofen and warfarin: increased risk of GI bleeding; ibuprofen inhibition of CYP2C9
B. Sotalol and furosemide: increased risk of QT prolongation and torsades de pointes;
furosemideinduced inhibition of CYP3A4
C. Sildenafil and sublingual nitroglycerin: increased risk of hypotension; sildenafil inhibition of
the phosphodiesterase type 5 isoform that inactivates cyclic guanosine monophosphate
D. Ritonavir and lovastatin: increased risk of myotoxicity; ritonavir inhibition of CYP2C19
E. Allopurinol and azathioprine: increased risk of blood dyscrasias; allopurinol inhibition of
Pglycoprotein

Answer: C

12. A 47-year-old truck driver with a history of HIV, hypertension, coronary artery disease, atrial
fibrillation, and ischemic cardiomyopathy. He is on antiretroviral therapy. He presents today
complaining of a new rash on his chest and axilla, which you astutely diagnose as tinea corporis.
You
would like to prescribe a course of oral ketoconazole for therapy. You should consider dose
adjustment for all of the following medicines that he is already taking EXCEPT:
A. Carvedilol
B. Lovastatin
C. Mexiletine
D. Ritonavir
E. Saquinavir

Answer: A

13. A 17-year-old patient who is known to have Wolff-Parkinson-White syndrome presents with
a regular narrow complex tachycardia with a cycle length of 375 milliseconds (160 bpm) that
occurred with a sudden onset. You note that there is a 1:1 atrial-to-ventricular relationship and
that the RP interval is 100 milliseconds. The best initial treatment is
A) IV procainamide.
B) atropine.
C) vagal maneuvers.
D) catheter ablation

Answer: C

14. A 65-year-old man presents after an arrest while eating at a local restaurant. On arrival,
paramedics documented ventricular fibrillation (VF), and he was successfully resuscitated. He
has a history of myocardial infarction (MI) and congestive heart failure (CHF). Serum
electrolytes are remarkable only for mild hypokalemia. MI is ruled out by ECG and serial blood
tests of myocardial enzymes. Subsequent evaluation includes cardiac catheterization, which
shows severe three-vessel coronary artery disease (CAD) and severe left ventricular (LV)
systolic dysfunction. A nuclear myocardial perfusion scan shows a large area of myocardial scar
without significant viability in the territory of the left anterior descending coronary artery. The
decision is made to treat the CAD medically. Which of the following is the best management
strategy for his arrhythmia?
A) PO amiodarone
B) Implantable cardioverter defibrillator (ICD) implantation if an electrophysiologic (EP) study
shows inducible VT or VF
C) ICD implantation
D) β-Blocker medication

Answer: C

15. A 56-year-old woman is undergoing a cadaveric renal transplant. After revascularization of


the transplanted kidney the transplanted renal parenchyma becomes swollen and blue. Which of
the following statements is most accurate regarding her transplanted kidney?
A) The donor had pre-formed antibodies against the recipient’s HLA antigens.
B) It is characterized pathologically by fibrin and platelet thrombosis of renal arterioles and
small arteries and necrosis of the glomerular tufts.
C) Biopsies should not be obtained intraoperatively.
D) This form of rejection is associated with disseminated intravascular coagulation (DIC).
E) The rejection process can be treated with a steroid bolus and OKT3

Answer: C

16. A 55-year-old man comes to the physician with the chief complaint of weight loss and a
depressed mood. He feels tired all the time and is no longer interested in the normal activities he
previously enjoyed. He feels quite apathetic overall. He has also noticed that he has frequent,
nonspecific abdominal pain. Which of the following diagnoses needs to be ruled out for this man?
A) Pheochromocytoma
B) Pancreatic carcinoma
C) Adrenocortical insufficiency
D) Cushing syndrome
E) Huntington disease

Answer: B

17. You are seeing a 78-year-old man who was brought to the office by his daughter. The
daughter says her father is becoming increasingly forgetful. His medical history is significant for
a 20-year history of type 2 diabetes and well-controlled hypertension. On examination, he is
mildly hypertensive with otherwise normal vital signs. He is oriented to time, place, and person,
but is unable to complete “serial sevens” on a mini-mental status examination. Which of the
historical features make this diagnosis more consistent with dementia as opposed to delirium?
A) His history of hypertension
B) His history of diabetes
C) His current level of orientation
D) His inability to complete serial sevens
E) The recent onset of his symptoms

Answer: C

18. A 45-year-old woman with Crohn’s disease and a small intestinal fistula develops tetany
during the second week of parenteral nutrition. The laboratory findings include Ca 8.2 meq/L;
Na 135 meq/L; K 3.2 meq/L; Cl 103 meq/L; PO4 2.4 meq/L; albumin 2.4; pH 7.48; 38 kPa; P 84
kPa; bicarbonate 25 meq/L. The most likely cause of the tetany is
A) Hyperventilation
B) Hypocalcemia
C) Hypomagnesemia
D) Essential fatty acid deficiency
E) Focal seizure

Answer: C

19. A 60 year-old woman has had a mitral valve replacement for chronic mitral stenosis and is on
cardiac ITU post-operatively. She is awake and self ventilating. SPO2 91%, FiO2= 0.6.
Monitoring shows atrial fibrillation at 90/min, blood pressure 88/60 and pulmonary artery
pressures of 45/15. Which single drug is most appropriate first line treatment to reduce the
pulmonary artery pressure?
A) Inhaled nitric oxide
B) Inhaled prostacyclin
C) Oral sildenafil
D) Intravenous isoprenaline
E) Intravenous milrinone

Answer: E

20. A 62-year-old man is admitted to the emergency department with abdominal pain. The
patient has a past history of ischaemic heart disease and atrial fibrillation. Computed tomography
scan features are highly suggestive of ischaemic bowel. The patient's blood gases are as follows:
pH = 7.25; paO2 = 10; paCO2 = 2.8; HCO3 = 18; Base excess = –8
Which of the following best describes the patient's acid–base status?
A) Metabolic acidosis
B) Metabolic acidosis with respiratory compensation
C) Respiratory acidosis with metabolic compensation
D) Metabolic acidosis with inadequate respiratory compensation
E) Cannot be sure without a serum lactate level

Answer: D

21. Two weeks ago, a 54-year-old man developed a cold sore, which resolved uneventfully. He
now complains of a nonpruritic skin rash. Examination reveals raised red lesions resembling
hives, some with clear fluid bullae. They are located on his hands, including the palms, and his
forearms and anterior tibia. The best way to confirm your suspected diagnosis is by:
A) Viral culture of blister fluid.
B) Smear of blister fluid for Gram stain.
C) Tzanck preparation of blister fluid to look for multinucleate giant cells.
D) Full thickness skin biopsy of involved area.
E) Wood’s light examination of involved areas

Answer: D

22. A 54-year-old man presents for a periodic health examination. His family history is
significant for his mother who died of a cerebrovascular accident at age 72, his father who died
of a myocardial infarction at age 68, and a brother who developed sigmoid cancer at age 60. The
patient is on no medications except for aspirin, 81 mg daily. His physical examination is
unremarkable. The patient asks for a recommendation regarding current cancer screening. Which
of the following is the most appropriate screening test for this patient?
A) Annual digital rectal examination and fecal occult blood testing
B) Flexible sigmoidoscopy
C) Flexible sigmoidoscopy and barium enema
D) Colonoscopy
E) Genetic testing for the p53 gene

Answer: D

23. Which of the following treatment options has been most consistently shown to be effective
for the primary prevention of sudden cardiac death in patients with CAD and recent MI?
a. D-Sotalol
b. β-Blocker medications
c. Amiodarone
d. Dofetilide

Answer: B

24. A 72-year-old woman is about to undergo an elective total hip replacement for osteoarthritis.
She has a history of hypertension and type 2 diabetes mellitus but no ischaemic heart disease or
peripheral vascular disease. Which of the following is the most appropriate thromboembolic
prophylaxis?
A) Intermittent pneumatic calf compression
B) Calf-length thromboembolic deterrent elastic stockings and early ambulation
C) Full-dose unfractionated heparin to increase the activated partial thromboplastin time to two
time control
D) Insertion of an inferior vena cava filter
E) Subcutaneous low-molecular-weight heparin

Answer: E
25. An AIDS patient under treatment with a nucleoside analog and a protease inhibitor comes to
medical attention with complaints of leg weakness and incontinence. His vital signs are within
normal limits. Physical examination reveals reduced strength in the lower extremities with
accompanying mild spasticity. There is also diminished sensation in the feet and legs bilaterally.
Lumbar puncture shows:
Opening pressure.....100 mm H20
Cell count................5 lymphocytes/mm3
Glucose...................48 mg/dL
Proteins, total..........33 mg/dL
Gamma globulin.......8% total protein
Additional laboratory investigations show normal hematologic parameters, vitamin B12 within
normal values, and negative serology for syphilis. MRI of the head fails to reveal any focal
abnormality. Which of the following is the most likely diagnosis?
A) AIDS dementia complex
B) CMV polyradiculopathy
C) Cryptococcal meningoencephalitis
D) Vacuolar (HIV) myelopathy
E) Zidovudine-related toxicity

Answer: D

26. A 59-year-old man presents to the ED with left-sided chest pain and shortness of breath that
began 2 hours prior to arrival. He states the pain is pressure-like and radiates down his left arm.
He is diaphoretic. His BP is 160/80 mm Hg, HR 86 beats per minute, and RR 15 breaths per
minute. ECG reveals 2-mm ST-segment elevation in leads I, aVL, V3 to V6. Which of the
following is an absolute contraindication to receiving thrombolytic therapy?
A) Systolic BP greater than 180 mm Hg
B) Patient on Coumadin and aspirin
C) Total hip replacement 3 months ago
D) Peptic ulcer disease
E) Previous hemorrhagic stroke

Answer: E

27. Which of the following statements is true about the patient with a bleeding disorder?
A) Factor deficiencies generally present with mucosal bleeding/petechiae.
B) Hemarthrosis generally reflects a platelet deficiency.
C) Hematomas are usually the result of a factor deficiency.
D) All of the above are true

Answer: C

28. Flu season is right around the corner and you are preparing your clinic for the onslaught.
First things first … you need to know how much vaccine to order and who will be receiving it.
The Centers for Disease Control and Prevention (CDC) annually publishes recommendations for
administering influenza vaccine to the American public. The CDC recommends vaccination for
all of the following groups, EXCEPT:
A) Health-care workers.
B) Nursing home residents.
C) Egg-allergic, febrile neonates.
D) Diabetics.
E) The elderly

Answer: C

29. During an outbreak, what intervention(s) is/are most appropriate for all your unvaccinated,
frail nursing home patients who have no symptoms of febrile respiratory illness?
A) Antiviral prophylaxis with oseltamivir.
B) Antiviral prophylaxis with amantadine.
C) Influenza immunization.
D) A and C given together.
E) B and C given together

Answer: D

30. Which of the following is indicated when evaluating for a suspected inherited coagulopathy?
A) CBC.
B) PT and PTT.
C) Platelet count.
D) PFA-100.
E) All of the above

Answer: E

31. A 43-year-old woman with rheumatoid arthritis visits for a scheduled follow-up. Plain X-ray
of the hands reveals marginal erosions at the metacarpal heads. Why are these erosions marginal?
A) Random localization
B) Plain X-rays fail to show central erosions
C) Rotation of the film
D) Marginal joint area is devoid of overlying cartilage
E) Presence of sesamoid bones

Answer: D

32. A 23-year-oldman attends his GP complaining of reduced exercise tolerance, nocturia and
frequent hiccups over the course of the past 4 months. He also has generalised bony pain. His BP
is 165/90 mmHg and his BMI is stable at 25.
The results of routine blood testsshow:
Creatinine 250 μmol/l
Haemoglobin 14g/dl
Phosphate 2.0 mmol/l
Calcium 2.1 mmol/l
What is the most likely cause of these results?
A) Adult polycystic kidney disease
B) Diabetes
C) Contrast nephropathy
D) Hypertensive nephropathy
E) Diffuse proliferative glomerulonephritis

Answer: A

33. A 41-year-old man with a history of nasal congestion, breathlessness, cough and wheeze
presents with a left foot drop. Which one of the following is the most likely diagnosis?
A) Diabetes mellitus
B) Wegener’s granulomatosis
C) Churg–Strauss syndrome
D) Pulmonary eosinophilia
E) Polyarteritis nodosa

Answer: C

34. The preferred initial therapy for elderly patients with arthralgia due to osteoarthritis is which
of the following?
A) NSAIDs.
B) COX-2 inhibitors.
C) Acetaminophen.
D) Combination narcotic analgesics.
E) Early joint replacement.

Answer: C

35. According to the Eighth Joint National Committee (JNC-8) guideline on managing
hypertension in 60 years old patient or more (with no DM or chronic kidney disease) the BP
lowering goal is:
A) <150/90 mm Hg
B) <140/90 mm Hg
C) <160/90 mm Hg
D) <120/90 mm Hg
E) <130/90 mm Hg

Answer: A

36. In general, all of the following are risk factors for gout EXCEPT:
A) Tobacco use.
B) Alcohol use.
C) Obesity.
D) Diuretic use.
E) Family history.
Answer: A

37. A 59-year-old woman had three episodes of right renal colic over the last year. Review of
system was remarkable for joint pain and fatigability. Past and family history were unremarkable.
She is 7 years postmenopause and takes no medication. On physical examination BMI 28.5
kg/m2, pulse 72 beat per minute, BP 150/95 mmHg. Laboratory data revealed. Fasting plasma
glucose 89 mg/dl: Serum creatinine 1.1 mg/dl: Serum total calcium 11.9 mg/dl: Serum ionized
calcium 6.8 mg/dl: Ultrasound of the abdomen revealed 2 stones at the right renal pelvis: 24 hour
urinary calcium 365 mg (N < 250 mg) The most appropriate next diagnostic test is:
A) Measurement of serum 25 hydroxyvitamin D.
B) Measurement of serum PTH.
C) Meseurement of serum parathyroid related protein.
D) Genetic testing for mutation in parathyroid calcium sensing receptor.
E) 24 hour urinary uric acid.

Answer: B

38. A woman presents with headache, lethargy and weight loss. Which one of the following
would make the diagnosis of giant cell arteritis unlikely?
A) A raised ESR
B) Bilateral headache
C) Non-tender temporal arteries
D) Papilloedema on fundoscopy
E) Patient is 60 years old

Answer: D

39. Which of the following statements regarding polycythemia vera is correct?


A) An elevated plasma erythropoietin level excludes the diagnosis.
B) Transformation to acute leukemia is common.
C) Thrombocytosis correlates strongly with thrombotic risk.
D) Aspirin should be prescribed to all these patients to reduce thrombotic risk.
E) Phlebotomy is used only after hydroxyurea and interferon have been tried.

Answer: A

40. A 49-year-old female with a 5-year history of diabetes mellitus type 2 presents for an initial
visit. She has no known complications of diabetes. She takes metformin, glyburide, andaspirin.
On examination, you find a pleasant, obese female in no distress. Her blood pressure is 136/86
mm Hg, pulse 86, respirations 14, and temperature 37°C. As you discuss monitoring her diabetes,
you recommend screening for early kidney disease. Which of the following approaches is the
recommended way to screen for diabetic kidney disease?
A) Obtain a 24-hour urine collection for albumin now and again in 3 years.
B) Obtain a spot urine microalbumin every year.
C) Obtain a spot urine microalbumin/creatinine ratio every year.
D) Obtain a urinalysis every year.
E) Obtain a serum creatinine every year.

Answer: C

41. A 34-year-old female presents with recurrent episodes of severe headaches, palpitations,
tachycardia, and sweating. A physical examination reveals her blood pressure to be within
normal limits; however, during one of these episodes of headaches, palpitations, and tachycardia,
her blood pressure is found to be markedly elevated. Workup finds a small tumor of the right
adrenal gland. Which one of the following is most likely to be increased in the urine of this
individual?
A) Acetone
B) Aminolevulinic acid (ALA)
C) Hydroxy-indoleacetic acid (HIAA)
D) N-formiminoglutamate (FIGlu)
E) Vanillylmandelic acid (VMA)

Answer: E

42. One of your 50-year-old male patients with CKD 5 is considering PD as one of his choices
for renal replacement therapy. He asks you about indications and contraindications about PD.
Which one of the following choices is a contraindication for PD?
A) Abdominal hernia not amenable to surgery
B) Adhesions from previous surgery
C) Severe inflammatory bowl disease
D) Diaphragmatic fluid leak
E) All of the above

Answer: E

43. Foreign bodies and/or food boluses can lodge in the esophagus in any of the following four
areas of narrowing except:
A) Hiatal hernia
B) Upper esophageal sphincter
C) Level of the aortic arch
D) Level of the mainstem bronchus
E) Gastroesophageal junction

Answer: A

44. A 62-year-old man with CKD 5 (eGFR 11 mL/min) due to HTN comes to the clinic for
routine follow-up with no complaints of nausea, vomiting, fatigue, or poor appetite. An arterio-
venous fistula (AVF) was placed 1 year ago when his eGFR was 12 mL/min and the fistula is
ready for use. His BP is 134/80 mmHg. He walks 2 miles everyday without shortness of breath,
or chest pain, or fatigue. Pertinent labs include: Na+ 139 mEq/L, K+ 4.4 mEq/L, HCO3 22
mEq/L, BUN 68 mg/dL, Ca2+ 8.8 mg/dL, phosphate 4.2 mg/dL, and albumin 4.1 g/dL. He
expresses hemodialysis (HD) as his choice of renal replacement therapy (RRT). According to the
KDIGO guideline, which one of the following is the MOST appropriate management in this
patient?
A) Start HD in 2 weeks at the outpatient dialysis unit
B) Start peritoneal dialysis (PD) in 4 weeks
C) Convince for preemptive kidney transplantation
D) Start HD when signs and symptoms of kidney failure are present
E) Suggest no RRT at any time, as he may do well with conservative management

Answer: D

45. A 75-year-old man underwent surgery to correct a large abdominal aortic aneurysm. The
procedure appeared to go well, but you are called a few hours later to evaluate the patient who
states that he cannot move or feel his legs. On the way to the ICU, you consider the possible
causes of his symptoms and plan your physical examination. What is the most important test to
help localize the lesion?
A) MRI of the spine
B) Sensory level
C) Reflexes in lower extremities
D) Plantar flexion reflex
E) Toe position sense

Answer: E

46. Which one of the following statements in CKD 5 patients regarding early (eGFR 10–14
mL/min) versus late (eGFR 5–7 mL/min) initiation of HD is CORRECT?
A) Early initiation of HD improves mortality and morbidity of patients
B) Late initiation of HD improves mortality and morbidity of patients
C) No difference between early and late initiation of HD either in survival or other outcomes
such as hospitalizations or quality of life
D) Compared to early initiation, late initiation is better in controlling mortality only
E) None of the above

Answer: D

47. A 69-year-old man was successfully defibrillated after an episode of ventricular fibrillation
secondary to an ST elevation myocardial infarction (STEMI) and transferred to the cardiac
catheter laboratory for primary coronary intervention. After the procedure began he had a further
episode of ventricular fibrillation. Regarding defibrillation, which is true?
A) A single direct current shock of 360 joules with a biphasic waveform is the most likely to
restore spontaneous circulation
B) Defibrillation is no more likely to be successful than a properly delivered praecordial thump
C) It is safe to continue with the coronary angiogram while the shock is delivered to the patient
D) Three shocks delivered with minimal interruptions should be given before any other
intervention
E) Two minutes of chest compressions before defibrillation is recommended to optimize
coronary perfusion

Answer: D

48. Which of the following statements regarding pillinduced esophagitis is true?


A) A bronchoesophageal fi stula is a common complication of NSAID-induced esophageal
ulceration.
B) Tetracycline or its derivatives cause pillinduced esophagitis by production of a caustic
alkaline solution.
C) Antibiotics as a class are uncommon causes of medication-induced esophagitis.
D) Esophageal damage from a bisphosphonate medication, such as alendronate, can be
minimized by ingestion of a full 8-oz glass of water taken in the upright position.
E) Chemotherapeutic agents are unlikely causes of pill-induced esophagitis.

Answer: D

49. A 32-year-old nurse presents with symptoms of dizziness, jittery behavior, and headaches
before meals. Which of the following supports the diagnosis of factitious hypoglycemia?
A) Elevated sulfonylurea levels
B) Normal proinsulin levels
C) Normal C-peptide levels
D) Plasma insulin-to-glucose ratio <0.3
E) All of the above

Answer: E

50. All of the following paraneoplastic syndromes are associated with gastric cancer except:
A) Acanthosis nigrans
B) Disseminated intravascular coagulation
C) Thrombophlebitis (Trosseau’s sign)
D) Pyoderma gangrenosum
E) Nephrotic syndrome

Answer: D

MULTIPLE CHOICE QUESTIONS-6

1. A broncho-pleural fistula is an abnormal communication or a passage between the bronchial


tree and the pleural space, causing a persistent leak. If these patients are mechanically ventilated,
the management strategy should be:
A) Low tidal volumes and high respiratory rate
B) Reduced inspiratory pressures
C) High tidal volumes and low respiratory rate
D) Low inspiratory times and high PEEP
E) High inspiratory times and low PEEP
Answer: B

2. Which of the following is a recognized feature of pulmonary embolism except?


A) S1,Q3,T3
B) an increase in serum troponin levels
C) an arterial pH greater than 7.2
D) increased PCO2 on air
E) positive D-dimer levels

Answer: D

3. A 45-year-old surgeon sustained a needle stick injury from a known HIV-positive individual
during a routine gall bladder operation and subsequently commenced a course of post-exposure
prophylaxis (PEP). Which of the following statements regarding his management is CORRECT?
A) He can discontinue PEP if it is established that the source patient has an undetectable HIV
viral load
B) He must refrain from operating until he has proven to be HIV-negative after completion of
PEP
C) If he has successfully completed a course of PEP, there is no requirement for him to have a
subsequent HIV antibody test
D) He cannot be considered HIV-negative until he tests HIV antibody-negative at three months
post-PEP
E) He cannot be considered HIV-negative until he tests HIV antibody-negative at six months
post-PEP

Answer: E

4. A patient presents with gradually worsening weakness of the proximal arm and leg muscles
symmetrically over several months. On examination, neck flexors and extensors are found to be
weak also. There is no muscle pain or tenderness. What is the most likely site of dysfunction in
the nervous system?
A) Peripheral nerve
B) Brachial plexus
C) Spinal nerve root
D) Internal capsule
E) Muscle

Answer: E

5. A 67 year old lady is referred to the bone clinic following a fractured wrist. A DEXA scan is
performed which shows a T-score of −2.0. She has no other osteoporotic risk factors. Which one
of the following treatments would you offer?
A) Lifestyle advice and vitamin D supplementation
B) Alendronic acid
C) HRT
D) Risedronate
E) No treatment

Answer: A

6. A 56-year-old man is admitted to the intensive care unit for acute respiratory distress
syndrome (ARDS). The patient was transferred from an outside hospital today after a 2-week
hospitalization for pneumonia. During that time, the patient's pulmonary status continued to
deteriorate. One week ago he was intubated and placed on mechanical ventilation and over the
past week, his oxygenation has worsened with a PaO2 of 66 on an inspired concentration of
100% oxygen. The patient has no other medical history except for rheumatoid arthritis. On
transfer to the ICU, the patient is intubated and sedated on a mechanical ventilator. His chest
radiograph shows patchy, bilateral, diffuse interstitial infiltrates. The most important intervention
that will most benefit this patient is to
A) keep the patient in a prone position during mechanical ventilation
B) keep tidal volumes greater than 15 cc/kg
C) limit peak inspiratory pressure to 45 cm H2O or less
D) limit PEEP levels to less than 10 cm H2O
E) limit tidal volumes to 6cc/kg

Answer: E

7. A previously fit and well 50-year-old male underwent a decompressive craniectomy for an
acute subdural haematoma following a traumatic head injury 24 hours ago. He is now on the
neurosurgical intensive care unit and remains intubated and ventilated. You have been asked to
review his urine output, which is recorded as 800 ml over the last 2 hours, despite appropriate
intravenous fluid administration. You suspect neurogenic diabetes insipidus.
Which of the following features would support your diagnosis?
A) Urine specific gravity >1.005
B) Serum Na+ <135 mmol/l
C) Urine osmolality <350 mmol/kg
D) Serum osmolality <295mmol/kg
E) Serum creatinine 200 mmol/l

Answer: B

8. A 35 year old man presents to his GP with a 2-week history of painful, swollen knees and a
painful left heel. He also reveals that he has been experiencing a burning pain when he urinates
and has a red, itchy eye. Which one of the following is the most likely diagnosis?
A) Gout
B) Osteoarthritis
C) Enteropathic arthritis
D) Ankylosing spondylitis
E) Reactive arthritis

Answer: E
9. A 40-year-old female who is intubated and ventilated following a subarachnoid hemorrhage
(SAH) 7 days previously has a serum sodium concentration of 128mmol/l and serum osmolality
of 270 mOsm/kg. Which of the following statements is true?
A) Cerebral salt-wasting syndrome (CSWS) is rarely associated with SAH
B) Cerebral salt-wasting syndrome is associated with a reduced serum osmolality
C) To diagnose SIADH, the patient must be clinically dehydrated
D) SIADH almost always requires pharmacological treatment
E) To diagnose SIADH urine osmolality must be greater than serum osmolality

Answer: E

10. A 47-year-old woman who is 2 weeks post triple bypass surgery presents to the emergency
department with a chief complaint of sudden onset, sharp chest pain for several hours. She is
fatigued and short of breath. On physical examination she has distended neck veins that grow
more distended on inspiration. Muffled heart sounds are heard. Her temperature is 37.0°C
(98.6°F), pulse is 133/min, blood pressure is 70/50 mm Hg, respiratory rate is 30/min, and
oxygen saturation is 100% on room air. An echocardiogram shows a large pericardial effusion
and chamber collapse; therefore, pericardiocentesis is performed. Although a large amount of
blood is aspirated, the patient’s clinical picture acutely worsens. Her pain level increases
substantially; pulse is 150/min, blood pressure is 60/41 mm Hg, respiratory rate is 30/min, and
oxygen saturation is 100%. Repeat echocardiography shows an even larger pericardial effusion
with chamber collapse. Which complication of pericardiocentesis is most likely in this patient?
(A) Acute left ventricular failure with pulmonary edema
(B) Aspiration of 10 mL air into the pericardium
(C) Laceration of a coronary vessel
(D) Pneumothorax
(E) Puncture of the left ventricle

Answer: C

11. During this outbreak, what intervention(s) is/are most appropriate for all your unvaccinated,
frail nursing home patients who have no symptoms of febrile respiratory illness?
A) Antiviral prophylaxis with oseltamivir.
B) Antiviral prophylaxis with amantadine.
C) Influenza immunization.
D) A and C given together.
E) B and C given together

Answer: D

12. A 38-year-old male presents with episodic wheeze and non-productive cough which occurs
particularly at night. He has been employed in the plastics industry. Which of the following
suggests a diagnosis of occupational lung disease?
A) Absent family history of asthma
B) Commencement of symptoms on his first day in this employment
C) Elevated serum IgE concentration
D) Improved symptomatology when on holiday
E) Increased bronchial reactivity

Answer: D

13. A 68-year-old male is admitted with a two months history of difficulty raising his arms,
ascending stairs, and is also aware of a dry mouth. He smokes 15 cigarettes daily and admits to
heavy alcohol consumption. On examination he has proximal weakness affecting all four limbs
with absent tendon reflexes. His chest X-ray shows a right pleural effusion. What is the most
likely diagnosis?
A) Alcohol induced myopathy
B) Eaton- Lambert syndrome
C) Myasthenia gravis
D) Polymyalgia rheumatica
E) Polymyositis

Answer: B

14. A 65-year-old woman, a heavy smoker for many years, has had worsening dyspnoea for the
past five years, without a significant cough. A chest x ray shows increased lung size along with
flattening of the diaphragm, consistent with emphysema. Over the next several years she
develops worsening peripheral oedema. Her BP 115/70 mmHg. Which of the following cardiac
findings is most likely to be present?
A) Constrictive pericarditis
B) Left ventricular aneurysm
C) Mitral valve stenosis
D) Non-bacterial thrombotic endocarditis
E) Right ventricular hypertrophy

Answer: E

15. A young male developed spontaneous DVT of right lower limb and he is having a sister who
had DVT 5 years back and having protein C deficiency. He was started on warfarin and the INR
came to 2-3 after which he had started following at local hospital where his warfarin dose was
steadily increased as his INR never went higher than 1.5 and is currently on warfarin dose of 20
mg/day. He was referred back to hospital and was evaluated and found to be having warfarin
level of 2.385 mg/l (therapeutic range 0.7 – 2.3 mg/l), PIVKA is > 10
(Ref. range < 0.2). What is the most likely explanation for the subtherapeutic INR?
A) Cytochrome p450 mutation
B) VKORC 1 mutation
C) Not taking warfarin
D) Local hospital INR testing quality control is not good

Answer: D
16. Which of the following anatomical considerations is correct?
A) Optic chiasm lesions characteristically produce a bitemporal hemianopia.
B) Central scotoma occurs early in papilloedema.
C) In cortical blindness pupillary reactions are abnormal.
D) Optic tract lesions produce an ipsilateral homonymous hemianopia.
E) Opticokinetic nystagmus is found with bilateral infarction of the parieto-occipital lobes

Answer: A

17. The most common non-opportunistic protozoon parasite is AIDS patients is:
A) Cryptosporidium
B) Gardia lamblia
C) Blastomyces hominis
D) Entamoeba histolytica
E) Toxoplasma gondii

Answer: D

18. To assist in the decision whether to hospitalize a patient with community acquired
pneumonia (CAP), each of the following may be a factor in favor of hospitalization except for
which one?
A) The patient is confused
B) Serum creatinine >2.0 mg/dL
C) Respiratory rate >30
D) Blood pressure < 90 mm Hg
E) Age >64 years

Answer: B

19. A 35-year-old African-American woman complains of red and irritated eyes with
photophobia for about 2 months. Visual acuity is 20/25 for each eye separately, and she says this
is her “normal.” On questioning she admits to shortness of breath with exertion, which she
attributes to neglecting physical training and advancing age. A chest x-ray shows perihilar
adenopathy. A lung biopsy finds noncaseous granuloma. Angiotensinconverting enzyme is
elevated. On spirometry, FEV 1 is 80% of predicted normal for her (percent of vital capacity
expired in 1 second). Which of the following would be the best therapeutic approach.
A) Non-steroidal anti-inflammatory drugs (NSAIDs)
B) Bronchodilators
C) Inhaled glucosteroids
D) Observation for 4–6 months while treating the eyes symptomatically
E) Systemic glucosteroids

Answer: D

20. A 30 year old lady underwent prosthetic valve replacement surgery done for rheumatic heart
disease and she is started on UFH by continuous infusion plus warfarin. Six days after she
developed DVT of right lower limb. Her CBC was normal preoperatively and now she has
dropped her platelet count to 50,000/mm3. Her INR is 2.3 and APTT is 90 (control 35). The best
therapeutic option would be:
A) IVIg followed by steroids
B) Fondaparinux
C) Stop heparin and add aspirin to warfarin
D) Change to low molecular weight heparin

Answer: B

21. A 59-year-old male presents with a 1 hour history of central crushing chest pain. He is
known to be diabetic, hypertensive and is a non-smoker. On examination his pulse rate is 90
beats/min, blood pressure 130/85 mmHg, S1 S2 are audible with no murmurs. There is no
evidence of cardiac failure. An ECG is performed. Which of the following would be an
indication for thrombolysis?
A ) Right bundle branch block
B ) Supraventricular tachycardia
C ) ST elevation of 2mm in V4-V6
D ) ST depression of 2mm in leads II,III, avF
E ) Atrial fibirillation >150min-1

Answer: C

22. A 46-year-old Caucasian woman complains of increasingly severe fatigue that she believe
emanates from poor sleep quality. Her husband notes that she moves frequently during her sleep,
and the patient notes that recently as she begins to ready herself for sleep she has discomfort in
her legs that is momentarily relieved by moving them. Each of the following may be helpful in
the treatment of this condition except?
A) Two month trial of ferrous sulfate
B) Stretching exercises before bedtime
C) A glass of red wine before bedtime
D) Gabapentin
E) Oxycodone 5–10 before bedtime

Answer: C

23. A 35-year-old woman complains of tingling and numbness of the little finger on the left hand.
On examination, you corroborate hypesthesia of the 5th finger and also all of the ring finger, and
you also find left-side weakness of flexion of fingers and wrist. The patient is able to spread the
fingers adequately. Which of the following diagnoses is likely to account for these symptoms?
A) Ulnar nerve injury
B) Carpal tunnel syndrome
C) C6 nerve root compression
D) C7 nerve root compression
E) C8 nerve root compression
Answer: E

24. Characteristic features of Mitral valve prolapse include


A) Early systolic murmur at the apex
B) Mid-systolic click
C) Valve Replacement gives better results than Valve repair
D) A poor prognosis
E) Higher incidence in males

Answer: B

25. A known case of chronic obstructive pulmonary disease presents to A & E, distressed and
cyanosed. Arterial blood gases reveal a pH 7.2, PaO2 55 mmHg and PaCO2 60 mmHg. He is
given high concentration oxygen together with a salbutamol nebulizer. Intravenous
hydrocortisone is also given. The patient becomes even worse with poorer breathing effort
although pulse oximetry showed SaO2 of 93%. What is the cause of patient deterioration?
A) Constriction of bronchioles in response to salbutamol nebulizer
B) High concentration oxygen administration
C) Pulmonary artery relaxation causing mismatch between perfusion and ventilation
D) Pulmonary vein relaxation causing mismatch between perfusion and ventilation
E) Reaction to IV hydrocortisone

Answer: B

26. Which of the following is a recognised treatment for complications of cystic fibrosis?
A) DNAase to assist in reinflating collapsed lung segments
B) Hypotonic saline drinks for hypernatraemic dehydration
C) Nebulised tobramycin for pseudomonas colonisation of the lower respiratory tract
D) Pancreatic transplant for diabetes mellitus
E) Rectal pull-through and anastamosis for rectal prolapse

Answer: C

27. A 45-year-old busy male attorney complains of headaches and generalized pruritus. He flew
into an eastern American city from Denver, Colorado, to attend a high-level legal conference. He
smokes 1 pack of cigarettes per day. He denies recent upper respiratory tract infection, foci of
specific pain, and urinary and GI symptoms. On examination, he manifests a blood pressure of
160/105, a temperature of 98.6 F, a ruddy complexion, and splenomegaly and hepatomegaly.
Blood gases are not immediately available. His CBC showed the hemoglobin level to be 18 g/dL,
the white blood cell count to be 14,000, and the platelet count to be 7,00,000. Red blood cells are
normochromic and normocytic. The BUN level is 18 mg/dL and the creatinine level is 1.1 mg/dL.
Urinalysis is within normal limits with a specific gravity of 1.015. Which of the following is the
diagnosis?
A) Polycythemia vera
B) Secondary polycythemia from altitude accommodation
C) Secondary polycythemia caused by chronic obstructive pulmonary disease
D) Spurious polycythemia from dehydration caused by air travel
E) Secondary polycythemia compensating for carboxyhemoglobin in a heavy smoker

Answer: A

28. A 26-year-old man has noted painless swelling in the left side of his neck. Furthermore, he
has noted afternoon chills and night sweats for several weeks. He has stopped smoking because
inhaling tobacco smoke makes him ill. Although he seldom drinks alcohol, during the past week,
he took a glass of wine and only then noted pain in the swelling about the neck. Examination is
unremarkable except for the neck,
which exhibits a firm irregular mass measuring approximately 3 cm 5 cm located in the left
anterior cervical region. The CBC and laboratory chemical battery are within normal limits.
Which of the following is the most likely significant cause of these symptoms?
A) Hodgkin disease
B) Sialoadenitis
C) Viral respiratory tract infection
D) Streptococcal pharyngitis
E) Carotid artery aneurysm

Answer: A

29. A 36-year-old male assembly line worker complains that 2 days ago, while hefting a bucket
of metal parts that slipped from his right hand while still grasping the other handle, experiences
sudden pain in the left side of his neck that radiates into the left lateral upper arm into the thumb,
associated with paresthesias in the thumb. Strength testing shows that his left wrist extension is
3/5. Which of the following is the nerve root involved?
A) C5
B) C6
C) C7
D) C8
E) T1

Answer: B

30. A 55-year-old man is scheduled for abdomen CT with contrast. He has type 2 diabetes; well
controlled with pioglitazone, metformin, and glimepride. On the day of the procedure he is to
receive nothing by mouth all morning except for medications until after the procedure. Which of
the following would be the most appropriate oral regimen on the day of the procedure?
A) Continue all medications
B) Hold metformin, and continue glimepride and pioglitazone
C) Hold glimepride and metformin, and continue pioglitazone
D) Hold glimepride and pioglitazone and decrease metformin dose by half
E) Hold metformin and pioglitazone, and continue glimepride

Answer: C
31. A 65-year-old woman, has smoked 50 cigarettes a day for 40 years. She has had increasing
dyspnoea for the several years, but no cough. A Chest X-ray shows increased lung size along
with flattening of the diaphragms, consistent with emphysema. Over the next several years she
develops worsening peripheral oedema. Her vital signs show T° 36.7 C, P 80, RR 15, and BP
120/80 mm Hg. Which of the following cardiac findings is most likely to be present?
A) Mitral valve stenosis
B) Constrictive pericarditis
C) Right ventricular hypertrophy
D) Left ventricular aneurysmm
E) Non-bacterial thrombotic endocarditis

Answer: C

32. Which of the following is a typical feature of Farmer's lung?


A) basal crackles
B) Eosinophilia
C) Haemoptysis
D) Increased pCO2
E) Positive serum paraproteins

Answer: A

33. A 19-year-old female developed pleural effusions, ascites and ankle swelling. Her blood
pressure was 112/76 mmHg.
Investigations revealed:
• serum alanine transferase 17 U/L (5 - 15)
• serum total bilirubin 17 umol/L (1 - 22)
• serum albumin 21 g/L (34 - 94)
• serum total cholesterol 9.8 mmol/L (<5.2)
What is the next most appropriate investigation?
A) Antinuclear antibody
B) Pregnancy test
C) Prothrombin time
D) Serum protein electrophoresis
E) Urinary protein estimation

Answer: E

34. The following are recognized features of Pancoast's tumour except:


A) ipsilateral Horner's syndrome
B) wasting of the dorsal interossei
C) pain in the arm radiating to the fourth and fifth fingers
D) erosion of the first rib
E) weakness of abduction at the shoulder

Answer: E
35. Which of the following is a recognised cause of a phrenic nerve palsy?
A) Aortic aneurysm
B) Dermoid
C) Ganglioneuroma
D) Pericardial cyst
E) Sarcoidosis

Answer: A

36. A 58 year old woman falls and suffers a vertebral compression fracture. BMD confirms
osteoporosis. Prior history of hysterectomy/oopherectomy at age 40 and prior DVT. Pain
controlled with Tylenol. Best treatment:
A) Estrogen
B) Estrogen/progesterone
C) SERM
D) Bisphosphonate
E) Calcitonin

Answer: D

37. A 56-year-old man is admitted to the intensive care unit for acute respiratory distress
syndrome (ARDS). The patient was transferred from an outside hospital today after a 2-week
hospitalization for pneumonia. During that time, the patient's pulmonary status continued to
deteriorate. One week ago he was intubated and placed on mechanical ventilation and over the
past week, his oxygenation has worsened with a PaO2 of 66 on an inspired concentration of
100% oxygen. The patient has no other medical history except for rheumatoid arthritis. On
transfer to the ICU, the patient is intubated and sedated on a mechanical ventilator. His chest
radiograph shows patchy, bilateral, diffuse interstitial infiltrates. The most important intervention
that will most benefit this patient is to
A) keep the patient in a prone position during mechanical ventilation
B) keep tidal volumes greater than 15 cc/kg
C) limit peak inspiratory pressure to 45 cm H2O or less
D) limit PEEP levels to less than 10 cm H2O
E) limit tidal volumes to 6cc/kg

Answer: E

38. A 56-year-old woman presents with a 3-day history of fever, headache, fatigue, and myalgia,
along with nausea and vomiting. She reports cough with minimal hemoptysis but denies
abdominal pain and dysuria. About a week ago, she came back from a month-long missionary
trip to a small village in Liberia. She had received appropriate pre-travel vaccines including the
yellow fever vaccine. She was also compliant with her malaria prophylaxis as prescribed. On
examination, she is tachypneic, tachycardic, and in mild respiratory distress. She has generalized
petechiae but no other rashes or lymphadenopathy. What is the most likely infectious pathogen?
A) Plasmodium malariae
B) Salmonella typhi
C) Influenza virus
D) Ebola virus
E) Mycobacterium tuberculosis

Answer: D

39. In a 21 year old man with symptoms of chronic back pain, pain in his feet, particularly the
great toe and metatarsophalangeal joints, and bilateral sacroiliitis on plain films, the most likely
diagnosis is:
A) Ankylosing spondylitis
B) Gout
C) Inflammatory bowel disease-related arthropathy
D) Reiter’s syndrome
E) Psoriatic arthritis

Answer: D

40. A 49-year-old woman comes to the office because of difficulty breathing, fevers reaching 40
C (104 F), and a productive cough with blood tinged sputum. She was recently diagnosed with
ductal carcinoma of the breast and underwent a radical mastectomy with four rounds of adjuvant
chemotherapy with vinblastine and doxorubicin. Her last infusion of chemotherapy was 5 days
ago. A chest radiograph shows focal infiltrates in both lungs. Laboratory studies show: You
admit her to the hospital and start her on vancomycin and ceftazidime. Over the next 3 days, her
clinical status continues to worsen. Blood cultures are negative. A bronchoscopy is performed
and biopsy samples are obtained. The biopsy specimen shows septated, branching hyphae that
are locally invading tissue. The most appropriate pharmacotherapy at this time is
A) amphotericin B, intravenously
B) fluconazole, intravenously
C) fluconazole, orally
D) itraconazole, intravenously
E) rifampin plus isoniazid, orally

Answer: A

41. An 82-year-old woman with a history of Hashimoto's thyroiditis is evaluated for a rapidly
expanding thyroid mass and progressive dysphagia and dyspnea. She has lost 2.2 kg over the
preceding 6 weeks. O/E: BP118/78 mm Hg, PR 86/min. Thyroid examination reveals a firm 5-
cm left thyroid mass that moves poorly with swallowing. Respiratory stridor is evident. Fine
needle aspiration of the mass shows numerous uniform lymphocytes. CT scan of the neck shows
the thyroid to completely encircle the trachea.
Which of the following is the most likely diagnosis?
A) Medullary thyroid cancer
B) Anaplastic thyroid cancer
C) Thyroid hemorrhage
D) Thyroid lymphoma
E) Laryngeal cancer
Answer: D

42. Thiazide diuretics can contribute to all of the following metabolic effects EXCEPT:
A) Hypomagnesemia
B) Hypourecemia
C) Hypercalcemia
D)Hypercholesrolemia
E) Hyponatremia

Answer: B

43. A 55 year old women is referred for evaluation of hypercalcemia. Three years ago, she had a
left mastectomy for breast carcinoma, after which she underwent treatment with radiation and
adjuvant chemotherapy because 2 of 12 lymph nodes were found to be positive for carcinoma.
She has been clinically well since then, but on a routine postoperative check-up 3 months ago,
she was found to have a serum calcium level of 2.75 mmol/L (N:2.2-2.6 mmol/L). On repeat
testing, the serum calcium level is 2.9 mmol/L
The best first diagnostic step to identify the cause of this patient’s hypercalcemia is:
A) serum alkaline phosphatase level
B) serum parathyroid hormone level
C) serum 1,25- dihydroxyvitamin D level
D) Serum 25 hydroxyvitamin D level
E) Bone Scan.

Answer: B

44. All of the following bedside maneuvers are useful to distinguish the murmur of mitral valve
prolapse from other valvular lesions EXCEPT ?
A) Isometric hangrip
B) Standing from a supine position
C) Carotid sinus massage
D) Valsalva maneuver
E) Squating from standing position

Answer: C

45. One of your 52-year-old female patients with an eGFR of 12 mL/min comes to your office
with complaints of fatigue and poor appetite due to metallic taste in the mouth for the last 2 years.
She has no chest pain or hyperkalemia. Her BP is 130/80 mmHg. Her serum [HCO3] is 20
mEq/L. During her previous visit 3 months ago, you discussed about the choice of her future
renal replacement therapies, including transplantation. After prolonged discussion, she feels that
PD is a reasonable choice in view of her daily work in the school. Based on her clinical history,
which one of the following choices is MOST appropriate regarding the management of her
symptoms and selection of dialysis modality is
CORRECT?
A) Admit to hospital and start HD with a central vein catheter (CVC), and then start PD
B) Tell her that she would receive in-center HD once acute HD treatment is over
C) Admit to hospital and place a PD catheter, and then train her for continuous ambulatory PD
with small volume exchanges initially
D) Change her diet and increase her NaHCO3 to 1350 mg every 8 h
E) Tell her that she may not need any renal replacement therapy (either HD or PD) until her
eGFR falls to 7 mL/min

Answer: C

46. Each of the following combinations has the potential for significant pharmacological
interaction and drug toxicity except
A) Simvastatin and Erythromycin
B) Sildenafil and Nitroglycerin
C) Pravastatin and Ketconazole
D) Cyclosporine and St. johns wort
E) Digoxin and Verapamil

Answer: C

47. A 40 year old man with recurrent severe headaches. They appeared to occur episodically
every 3-6 months, with headaches occuring daily for up to 8 weeks at a time. They often woke
him from sleep at 2 am, with sever right peri-oricular pain and would typically last for 30-60
mins. He also described right nasal congestion and lacrimation but no vomiting or photophobia.
Neurological exam., MRI brain were normal. What is the most appropriate prophylactic
treatment:
A) Carbamazepine
B) Erogotamine
C) Propranolol
D) Sumatriptan
E) Verapamil

Answer: E

48. A 57-year-old female school cleaner is undergoing investigation for breathlessness. All the
following would be in keeping with a diagnosis of constrictive pericarditis except:
A) Ascites
B) Elevated JVP with absent y descent
C) Orthopnoea
D) Peripheral oedema
E) Previous cardiac surgery

Answer: B

49. A 45-year-old woman undergoes upper endoscopy for symptoms of dyspepsia. The
endoscopic examination is normal. Physical examination, routine laboratory studies, serologic
tests forHelicobacter pylori, and abdominal ultrasonography are also normal. The patient has not
received any medications for her symptoms. Which of the following is the most appropriate
initial empiric therapy for this patient?
A) A proton pump inhibitor
B) Alosetron
C) Ondansetron
D) Tegaserod
E) Sumatriptan

Answer: A

50. By definition, an episode of pancreatitis is considered chronic pancreatitis if


A) It is a recurrent episode.
B) It occurs in the face of alcohol use.
C) There are radiographic findings of ductal irregularity and parenchymal fibrosis.
D) It is associated with steatorrhea.
E) A pseudocyst is present.

Answer: C

MULTIPLE CHOICE QUESTIONS-7

1. A 37-year-old woman with a history of intravenous drug use, hepatitis B, asthma, and
acquired immunodeficiency syndrome (AIDS) is admitted to the hospital because of fever, night
sweats, and malaise. Her last CD4 count was 1 month ago and measured 180/mm3. Vital signs
are: temperature 38.5 C (101.3 F), blood pressure 145/76 mm Hg, and pulse 90/min. Physical
examination is significant for a soft diastolic murmur heard best at the lower left sternal border.
Auscultation of the lungs reveals diffuse rhonchi. The abdominal and neurologic exams are
unremarkable. The next step in managing this patient is
A) analysis and culture of spinal fluid
B) a blood culture
C) a CT of the head
D) a urinalysis
E) an x-ray of the chest

Answer: E

2. An 18-year-old man presents with pain in the right side of the neck, veering to the right, and
numbness on the right side of the face and in the left arm and leg that came on acutely with
lifting heavy weights. His examination shows right hemiataxia, right miosis and ptosis, poor
palatal elevation on the right, and decreased sensation to pinprick of the right side of the face and
the left arm and leg. To which of the following structures does the lesion best localize?
A) Right lateral medulla
B) Left lateral medulla
C) Right cerebellum
D) Left midbrain
E) Central pons
Answer: A

3. A 78-year-old woman is admitted to the hospital because of a fever, productive cough, and a
chest x-ray demonstrating right lower lobe consolidation. Her past medical history is significant
for seasonal allergies. She has been taking estrogen/progesterone replacement since menopause
19 years ago and occasional acetaminophen for headaches. The patient lives alone at her home
and she does not drink alcohol or smoke. Review of systems is significant for weakness
attributed to "old age". On the day prior to discharge, a repeat chest x-ray shows the pneumonia
to be resolving. An incidental note is made of severe osteoporosis involving all of the bones
visualized on the film. Vital signs are temperature 38.8 C (101.8 F), blood pressure 100/50 mm
Hg, pulse 90/min, and respirations 10/min. Physical examination is significant only for decreased
breath sounds at the right lung base. The patient is neurologically intact and wants to return
home. Laboratory studies show a leukocyte count 15,000/mm3, hematocrit 28%, and platelets
150,000 mm3. The next step in the management of this patient is to
A) discharge her and do a bone marrow biopsy as an outpatient
B) discharge her and send her for a bone scan as an outpatient
C) discharge her and order serum protein electrophoresis as an outpatient
D) do a bone marrow biopsy before discharge
E) order a bone scan and serum protein electrophoresis before discharge

Answer: C

4. Eosinophilia is frequently seen with fever of unknown origin from all but 1 of the following
illnesses. Which illness is not frequently associated with eosinophilia?
A) Systemic lupus erythematosus (SLE)
B) Drug fever
C) Tuberculosis
D) Myeloproliferative disease
E) Polyarteritis nodosa (PAN)

Answer: C

5. In a patient with a known Chiari I malformation, burning pain develops in both shoulders. On
examination, strength is normal in the limbs, but the sensation to pinprick is reduced in a
capelike distribution around the shoulders extending to the upper part of the arms. To what
structure does the lesion best localize?
A) Bilateral dorsal columns
B) Right lateral funiculus
C) Central spinal cord
D) Bilateral ventral funiculus
E) Bilateral ventral horns

Answer: C

6. A 33-year-old asymptomatic woman was referred because of abnormal ALT. History of


migraine headache for which she takes Sumatriptan. No risk factors or history of liver disease.
Examination is notable for obesity.AST 60, ALT 75, ALP , bilirubin , INR, iron studies and
albumin are normal. What would be the most appropriate next step ?
A) Observe and repeat ALT in 3 months
B) Ultrasound abdomen
C) Check viral markers
D) Liver biopsy
E) Stop Sumatriptan

Answer: A

7. A 63-year-old woman with 4 hour history of abdominal pain , fever and nausea. On
examination , patient has fever, jaundice and mild epigastric tenderness . TC 18000, with shift to
left, bilirubin elevated, ALP 150,(normal upto 125) AST 745 IU, ALT 650 IU, USG multiple
small stones in gall bladder, no bile duct dilatation , normal pancreas. Started on antibiotics, but
following day still has fever. Repeat labs bilirubin elevated, AST 284 IU, ALT 200 IU, WBC
25000, blood culture positive for E.Coli. which of the following would you advice next ?
A) Doppler of hepatic vessels
B) Lab cholecystectomy
C) MRCP
D) Endoscopic ultrasound
E) ERCP

Answer: E

8. A 18-year-old woman presents with acute onset of jaundice and somnolence. On examination ,
jaundiced , sleepy but arousable. Labs INR 1.6, AST 240, ALT 210, total bilirubin increased ,
mostly in direct, hemoglobin 9.4, ceruloplasmin 8ng/L(normal more than 22), 24 hr urine copper
563mcg/L (normal less than 60).which of the following would you advise now ?
A) Trientine
B) Penicillamine
C) Urgent liver transplantation
D) Intracranial pressure monitoring
E) Liver biopsy

Answer: C

9. Warfarin is administered to a 56-year-old man following placement of a prosthetic cardiac


valve. The warfarin dosage is adjusted to maintain an INR of 2.5. Subsequently, trimethoprim-
sulfamethoxazole therapy is begun for a recurring urinary tract infection. In addition to
monitoring prothrombin time, which of the following actions should the physician take to
maintain adequate anticoagulation?
A) Begin therapy with vitamin K
B) Increase the dosage of warfarin
C) Make no alterations in the dosage of warfarin
D) Decrease the dosage of warfarin
E) Stop the warfarin and change to low dose aspirin
Answer: D

10. You are called emergently to the medical floor where a 66-year-old man was found to be
minimally responsive. His past medical history is unclear but his arm band lists allergies to
penicillin and sulfa medications. On arrival, chest compressions are being performed and 2
operators are mask ventilating the patient. Evaluation with an electrocardiogram reveals sinus
tachycardia and the diagnosis of pulseless electrical activity is made. Volume is infused and
compressions are continued. The patient remains apneic, so mask ventilation continues. During
masking, the patient appears to regurgitate large volumes of gastric contents. The most
appropriate immediate next step in the management of this patient is to
A) cease mask ventilation and suction the mouth
B) continue masking the patient
C) insert a nasogastric tube
D) intubate the trachea and suction the airway
E) intubate the trachea and ventilate

Answer: D

11. Which statement about evaluating a patient with fever of unknown origin (FUO) is true?
A) Bone marrow cultures have a high yield when the fever is greater than 39 ° C and other
testing has been unrevealing.
B) A temporal artery biopsy is a reasonable test to perform next for a 75-year-old man who has
FUO and no localizing complaints, an erythrocyte sedimentation rate greater than 100 mm/h,
mild anemia, normal blood chemistry results, and negative blood cultures at 3 days and who has
not had any other evaluation.
C) Lumbar puncture should be done on every patient with FUO.
D) When there are no clues to the underlying diagnosis, extensive serologic testing for unusual
infections is often helpful.
E) Liver biopsy is unlikely to be helpful in a patient with FUO and miliary tuberculosis

Answer: B

12. A 31-year-old man describes a burning sensation in the substernal area after eating
chocolates, caffeine, or alcohol. The symptoms are exacerbated at night, and he has woken on
several occasions from sleep because of coughing. He has tried over-the-counter antacids and H2
receptor antagonists with little relief. He often takes antacids before dinner. Which of the
following is the most likely explanation for his symptoms of coughing?
A) Acid-induced bronchoconstriction
B) Aspiration of antacids taken before bedtime
C) Aspiration of solid food ingested with dinner
D) H2 receptor antagonist-induced bronchoconstriction
E) Tracheoesophageal fistula

Answer: A
13. A 24-year-old lady at 10 weeks of gestation during her first pregnancy. She has had 2 days of
nausea and mild fever. She vomited once this morning. On examination jaundiced, with tender ,
mild hepatomegaly. Labs AST 56) IU, ALT 1150 IU, bilirubin increased, platelets 140. most
likely diagnosis
A) Hyperemesis gravidarum
B) Acute viral hepatitis
C) Acute cholecystitis
D) Intra hepatic cholestasis of pregnancy
E) Fatty liver of pregnancy

Answer: B

14. A 36-year-old male with alcoholic hepatitis. Which clinical or lab parameter is least
predictive of patient survival ?
A) Bilirubin
B) Prothrombin time
C) AST/ALT ratio
D) Encephalopathy
E) Creatinine

Answer: C

15. A 64-year-old male executive with remote history of heavy alcohol use is noted to have
cirrhosis on liver biopsy. There are no varices detected on upper GI Endoscopy. When would
you recommend that the patient return for a repeat upper Endoscopy to screen for esophageal
varices ?
A) 6 months
B) 12 months
C) 18 months
D) 24 months
E) Never

Answer: D

16. Which of the following statements about pulmonary function during pregnancy is true?
A) Total lung capacity is reduced.
B) Functional residual capacity is reduced.
C) FEV1 /FVC ratio is reduced.
D) Lung compliance is reduced.
E) The single breath diffusing capacity for carbon monoxide is reduced

Answer: B

17. Which one of the following is the commonest cause of death in SLE?
A) Infection
B) Cardiovascular disease
C) Lupus nephritis
D) Cerebral lupus
E) Thromboembolism

Answer: A

18. Each of the following is a solid indication for neuroimaging in a patient with headache
EXCEPT:
A) Onset of headaches over the age of 50 years
B) Seizures associated
C) Prolonged aura
D) Nausea and vomiting
E) Headache worsening with movement

Answer: D

19. A lady is newly diagnosed with osteoporosis. She has never been on treatment. Which one of
the following is the first-line treatment?
A) Alendronic acid
B) Etidronate
C) Risedronate
D) Strontium ranelate
E) Zoledronate

Answer: A

20. A 48 year-old lady is referred to the rheumatology outpatient clinic with a 4-month history of
fatigue, aches and weakness of her thighs.She has a past medical history of
hypercholesterolaemia and is on simvastatin. On examination she has normal muscle bulk.
Active hip flexion was reduced. Neurological examination was otherwise normal.
Investigations:
Hb 98 g/L (115–165)
WCC 12.0 × 109 /L (4.0–11.0)
Platelets 400 × 109/L (150–400)
Plasma viscosity 2.0 (1.5–1.72)
Creatine kinase 15000 IU/L
The diagnosis is likely to be which one of the following?
A) Polymyalgia rheumatica
B) Statin-induced myopathy
C) Polymyositis
D) Dermatomyositis
E) Hip osteoarthritis

Answer: C
21. A 78-year-old man has collapsed several hours ago and been found on the floor by the
paramedics. He was found to be hypothermic at 32°C. His creatinine kinase is 10,000 IU. His
serum potassium is 5mmol/L, urea is 20mmol/L and creatinine is 300μmol/L. The most
important form of initial treatment should be:
A) Intravenous infusion of warmed crystalloid.
B) Active warming.
C) Intravenous administration of mannitol 0.25-0.5g/kg.
D) Intravenous furosemide.
E) Urgent haemodialysis

Answer: A

22. A 25-year-old woman with a history of epilepsy presents to the emergency room with
impaired attention and unsteadiness of gait. Her phenytoin level is 37. She has white blood cells
in her urine and has a mildly elevated TSH. Examination of the eyes would be most likely to
show which of the following?
A) Weakness of abduction of the left eye
B) Lateral beating movements of the eyes
C) Impaired convergence
D) Papilledema
E) Impaired upward gaze

Answer: B

23. A 35-year-old female is referred for evaluation of positive antinuclear antibodies (ANA). She
is asymptomatic. Her lab work reveals ANA 1:320 homogeneous pattern; extractable nuclear
antigens are negative. Hematologic and renal function values are normal and urinalysis is
without sediment. Her examination is unremarkable apart from a smooth nontender goiter. The
past medical history is significant for Hashimoto’s thyroiditis. No rash, synovitis, or serositis is
observed. What further treatment or investigations are warranted?
A) Start prednisolone 40 mg daily
B) Start prednisolone 20 mg daily
C) Repeat ANA test
D) Start hydroxychloroquine
E) No further intervention is warranted

Answer: E

24. A 41-year-old man is diagnosed with iron deficiency anemia and is found to have heme-
positive stools. Colonoscopy reveals a large ulcerated tumor in his transverse colon. He also has
two smaller polyps in his ascending colon. Pathologic examination of the tumor biopsy reveals
adenocarcinoma, while biopsies of the polyps confirm that these are adenomas. His sister has
been diagnosed with uterine cancer, and two cousins have died of colon cancer. All of the
following are true statements about this case except
A) Referral for genetic counseling is indicated
B) He is at increased risk for other epithelial-derived tumors
C) He likely has familial adenomatous polyposis (FAP), with a germ line mutation in the APC
gene
D) His condition is often associated with a defect in DNA mismatch repair

Answer: C

25. A 60-year-old man presents with abdominal pain and a cupful of haematemesis. On
examination he is noted to have ascites, hepatomegaly and a very enlarged spleen extending to
the right iliac fossa. His initial blood tests reveal a leukoerythroblastic picture with a
haemoglobin of 8, white cell count (WCC) of 3 and platelets of 120. A diagnosis of
myelofibrosis is made. What is most likely to be seen on the peripheral blood smear?
A) Schistocytosis
B) Sickle cells
C) Spherocytes
D) Dacrocytes
E) Target cells

Answer: D

26. All the following conditions are associated with high output heart failure EXCEPT
A) Iron overload
B) Hyperthyroidism
C) Systemic arteriovenous fistula
D) Thiamine deficiency
E) Paget disease

Answer: A

27. Primary prophylaxis to prevent acute infection in susceptible patients is recommended for all
the following HIV Infected patients EXCEPT
A) Pneumocystis jiroveci (formerly carinii) pneumonia.
B) Disseminated Mycobacterium avium complex.
C) Mucocutaneous candidiasis.
D) Malaria for persons traveling to areas where malaria is endemic.
E) Toxoplasma gondii encephalitis

Answer: C

28. A 30-year-old male with Alports and gradually worsening renal dysfunction has reached
ESRF and is on hemodialysis. His family is being evaluated for donation for renal transplant. His
57 year old mother and one aunt have microscopic hematuria but normal renal functions. Two of
his uncles also have ESRD and are on dialysis. He has a 40 brother with normal urine analysis, a
38 year old sister with normal urine analysis and a 25 year old sister with microscopic hematuria,
all with normal renal function.
Which of the following can most safely donate kidney for this patient?
A) Brother as he has extremely low probability of having Alports.
B) Sister with normal urine analysis, as she has no probability of having Alports.
C) Sister with microscopic hematuria as she is younger.
D) Mother
E) None of family members should be taken as donor

Answer: B

29. A 48-year old female with rheumatoid arthritis presents to the emergency department with 2-
week pain and tightness behind the left knee. Examination reveals cystic swelling over the left
popliteal fossa. Which of the following is the most appropriate next action?
A) Arthrogram of the left knee
B) Synovial biopsy of left knee
C) Ultrasound study of left knee and popliteal fossa
D) Venogram of left lower limb
E) None of the above

Answer: C

30. A 37-year-old woman is admitted to accident and emergency with severe facial burns.
Despite prompt management, she develops acute respiratory distress syndrome (ARDS).
Which of the following is not associated with the diagnostic criteria for ARDS?
A) Bilateral infiltrates on chest x-ray
B) Acute onset
C) Pulmonary capillary wedge pressure >19
D) Refractory hypoxemia (PaO2:FiO2 <200)
E) Lack of clinical congestive heart failure

Answer: C

31. A 55-year-old man with chronic hepatitis C is being considered for liver transplantation. The
patient has cirrhosis that was documented by liver biopsy 10 years ago. For the past 3 months, he
has had ascites and edema, which are poorly controlled with diuretics. Lactulose was recently
begun because of confusion. Which of the following combinations of laboratory studies will be
most helpful in estimating his survival over the next 6 months?
A) Serum total bilirubin and INR
B) Serum aspartate aminotransferase and gamma globulin
C) Serum alanine aminotransferase and hepatitis C RNA (HCV RNA)
D) Serum alkaline phosphatase and ammonia
E) Serum albumin and ?-glutamyltransferase

Answer: A

32. A 67-year-old African American man with bronchogenic carcinoma returns to the office for
follow-up of confusion and lethargy that have been gradually increasing during the past 3 weeks.
CT scan of the head 4 weeks ago showed no metastases. Current medications include inhaled
bronchodilator medication and oxycodone for pain. The patient is 185 cm (6 ft 1 in) tall and
weighs 61 kg (135 lb); BMI is 18 kg/m2. Vital signs are normal. Physical examination shows
generalized muscle wasting. Auscultation of the lungs discloses scattered rhonchi in all fields
and expiratory wheezes. Mini- Mental State Examination score is 24/30. Results of laboratory
studies are shown:
Serum Na+ 125 mEq/L
K+ 3.2 mEq/L
Cl− 100 mEq/L
HCO3 − 25 mEq/L
Blood
Hematocrit 32.2%
Hemoglobin 11.2 g/dL
Which of the following is the most appropriate study to order at this time?
A) 24-Hour urine collection for creatinine clearance
B) Determination of AM serum cortisol concentration
C) Determination of serum iron concentration
D) Determination of serum magnesium concentration
E) Determination of urine sodium concentration

Answer: B

33. Which of the following β-adrenergic antagonists is a nonselective β1 and β2 blocker?


A) Atenolol
B) Betaxolol
C) Esmolol
D) Metoprolol
E) Nadolol

Answer: E

34. Signs and symptoms of opioid withdrawal include all of the following EXCEPT
A) Increased blood pressure (BP) and heart rate
B) Seizures
C) Abdominal cramps
D) Jerking of the legs
E) Hyperthermia

Answer: B

35. A 45-year-old man is evaluated for heartburn with water brash, usually occurring at bedtime.
He notes worsening of symptoms when lifting weights, but he is asymptomatic during his routine
5-km run. He has not had dysphagia, bleeding, vomiting, or weight loss. He has been taking
nonprescription omeprazole once daily for the past two weeks. BMI is 32. Vital signs and
physical examination are normal. Complete blood count, serum electrolytes, and
electrocardiogram are normal.
Which of the following should you recommend?
A) An increase of omeprazole dosage to twice daily
B) Exercise stress test
C) Computed tomography of the abdomen
D) Ultrasonography of the abdomen
E) Esophagogastroduodenoscopy

Answer: A

36. A pregnant woman is being followed for gestational diabetes. She walks 30 minutes per day
and is following a diabetic diet. She is not gaining weight. Fasting sugars are 5 - 6.5;
postprandial, 7 - 8.5. She now has ketonuria in the mornings. What should you advise?
A) Increase caloric intake, plus begin insulin NPH 10u QHS
B) Increase caloric intake, plus begin insulin lispro ac meals
C) Increase caloric intake and exercise before breakfast
D) Increase calorie intake and postprandial exercise
E) Increase calorie intake, plus an oral agent

Answer: B

37. A 60-year-old man is admitted to the hospital for management of acute pancreatitis. Results
of laboratory studies are shown: Serum Blood Amylase 1000 U/L Hematocrit 42%; Calcium 8.4
mg/dL WBC 14,000/mm3; Urea nitrogen 5 mg/dL. Results of serum liver chemistry profile are
within the reference ranges. After 48 hours of fluid therapy and observation, a poor prognosis
would be indicated by which of the following laboratory study results?
A) Serum alanine aminotransferase (ALT) concentration of 106 U/L
B) Serum amylase concentration of 2000 U/L
C) Serum bilirubin concentration of 4.2 mg/dL
D) Serum calcium concentration of 6.6 mg/dL
E) Serum glucose concentration of 200 mg/dL

Answer: D

38. Linezolid has the following characteristics as compared to vancomycin:


A) Decreased nephrotoxicity at higher doses
B) Increased intrapulmonary penetration
C) Increased incidence of thrombocytopenia
D) All of the above
E) None of the above

Answer: D

39. A 19-year-old female developed pleural effusions, ascites and ankle swelling. Her blood
pressure was 112/76 mmHg.
Investigations revealed:
• serum alanine transferase 17 U/L (5 - 15)
• serum total bilirubin 17 umol/L (1 - 22)
• serum albumin 21 g/L (34 - 94)
• serum total cholesterol 9.8 mmol/L (<5.2)
What is the next most appropriate investigation?
A) Antinuclear antibody
B) Pregnancy test
C) Prothrombin time
D) Serum protein electrophoresis
E) Urinary protein estimation

Answer: E

40. The following are recognized features of Pancoast's tumour except:


A) ipsilateral Horner's syndrome
B) wasting of the dorsal interossei
C) pain in the arm radiating to the fourth and fifth fingers
D) erosion of the first rib
E) weakness of abduction at the shoulder

Answe: E

41. A 43-year-old woman has the following laboratory test results: serum bilirubin, total 4.6
mg/dL and direct 0.3 mg/dL; serum alkaline phosphatase, 108 U/L; serum aminotransaminase,
aspartate aminotransferase (AST) 18 U/L and alanine aminotransferase (ALT) 22 U/L. What is
the most likely
diagnosis?
A) Chronic hepatitis C virus infection
B) Dubin-Johnson syndrome
C) Hemolytic anemia
D) Primary biliary cirrhosis
E) Primary sclerosing cholangitis

Answer: C

42. A 52 year old woman referred because of palpitation, weight loss and increased sweating. 2
years back she was admitted to CCU because of Acute MI complicated by ventricular
tachycardia and was discharged on amiodarone. Her TFT revealed TSH 0.01, FT4 > 100. RAIU
showed increased uptake and U/S thyroid showed increased vascularity. What is the next
immediate step:
A) Stop amiodarone
B) Start carbimazole
C) Start steroids
D) Give appointment with cardiologist to change amiodarone
E) Observe and reassure the patient

Answer: B
43. You are asked to see a woman recently admitted to the ICU with hypotension. She is unable
to give a history, and no family members or friends are available. Witnesses report that she was
sitting at the airport waiting for a connecting flight when she passed out. The person sitting next
to her noticed that she became quite agitated when it was announced that the flight would be
delayed for 6 hours, just before she passed out. There is no evidence of a pulmonary embolism.
During your examination, you note that she is very thin, lacks axillary hair, has sparse pubic hair,
and has extra pigmentation on her gums and buccal mucosa. The blood pressure was 40/palp
with a heart rate of 130 when she first arrived, but is now 100/64 with a heart rate of 78 with
saline running. Her temperature is 98.2° F; the cardiac rhythm is sinus; the routine chemistry
tests show a hyperkalemic metabolic acidosis; and the renal status is compatible with prerenal
azotemia.
Which of the following should you do now?
A) Begin hydrocortisone 100 mg IV every 8 hours and order an ACTH stimulation test.
B) Measure cortisol and FT4 levels before beginning thyroxine and methylprednisolone.
C) Begin hydrocortisone 100 mg IV every 8 hours.
D) Begin dexamethasone 4 mg IV and order an ACTH stimulation test.
E) Measure ACTH and TSH before beginning prednisone 60 mg and thyroxine 100 μg every day.

Answer: D

44. A 45-year-old woman is hospitalized for a cholecystectomy. She appears to be anxious and
worried about the surgery. Which type or types of information provided to the patient will be
most effective in reducing stress?
A) Sensory information
B) Procedural information
C) Coping information
D) Sensory and procedural information
E) Sensory and coping information

Answer: E

45. A 60-year-old woman was hospitalized with a severe respiratory infection for several weeks.
Afterward, she displayed symptoms of myalgia and weakness of the lower limbs. In addition, she
also showed loss of muscle tone and some flaccidity with loss of tendon reflexes.
Examination also revealed a weakness of facial muscles. This constellation of symptoms
progressed for approximately 2 weeks and persisted for more than a year, at which time,
recovery took place at a slow rate. There was also some demyelination coupled with lymphatic
inflammation at the site of demyelination. The most likely cause of this patient’s condition is
A) Myasthenia gravis
B) Muscular dystrophy (MD)
C) Multiple sclerosis (MS)
D) Guillain-Barré syndrome
E) Lumbar disk prolapse

Answer: D
46. A 40-year-old patient with a recent viral infection presents with a significantly tender gland,
low radioiodine uptake, and signs and symptoms of thyrotoxicosis. This presentation is most
likely
A) Graves’ disease
B) Subacute thyroiditis
C) Toxic multinodular goiter
D) Hashimoto’s thyroiditis
E) Toxic adenoma

Answer: B

47. A 30-year-old male presents with multiple soft, raised, beefy-red superficial ulcers in his left
groin. Physical examination reveals several enlarged left inguinal lymph nodes. A histologic
section from an enlarged lymph node that is stained with a silver stain reveals characteristic
Donovan bodies within macrophages. What is the most likely diagnosis?
A) Chancroid
B) Gonorrhea
C) Granuloma inguinale
D) Lymphogranuloma venereum
E) Syphilis

48. A 55-year-old woman comes to the emergency department because of abdominal pain. She
had just finished eating a steak dinner with her family when she suddenly experienced sharp,
crampy pain in the upper right and middle of her abdomen. The pain has lasted for the past 3
hours and she is starting to feel nauseous. On physical examination, she is obese and in obvious
discomfort. Her temperature is 38.8 C (101.8 F), blood pressure is 140/87 mm Hg, pulse is
90/min, and respirations are 16/min. Abdominal examination is significant for focal tenderness
and guarding in her right upper quadrant. She is particularly tender when you palpate her right
upper quadrant as she takes in a deep breath. The most appropriate next step in the evaluation of
her abdominal pain is
A) an abdominal x-ray
B) a CT of the abdomen
C) an endoscopic retrograde cholangiopancreatography (ERCP)
D) serum liver function tests including bilirubin
E) an ultrasonography of the abdomen

Answer: E

49. A 32-year-old female smoker presents with acute severe asthma. The SaO2 are 80% on 15 L
of oxygen and the pO2 is 8.2kPa (10.5-13). There is widespread expiratory wheeze throughout
the chest. She is given IV hydrocortisone, 100% oxygen and 5 mg of nebulised salbutamol.
What is the next step in your management
A) IV augmentin
B) IV magnesium
C) IV potassium
D) IV theophylline
E) Oral prednisolone

Answer: B

50. A previously healthy16 year old girl brought by her parents to ER because of repeated
vomiting, lethargy, breathlessness and confusion of 1 day duration. She developed 2 attacks of
tonic clonic seizures up on arrival to ER. Physical examination was remarkable for confusion,
tachypnea and temp. of 38.5 C. CT brain, CXR and CSF analysis were normal. Blood
biochemistry revealed Na 132 mmol/L, K 3.0 mmol/l, BUN 12 mmol/l, Creatinine 74 mmol/l
and HCO3 12 mmol/L. ABG showed ph 7.5 , PaCO2 16 mmHg, PaO2 120 mmHg, HCO3 13
mmol/L. What is the best management option:
A) start dexamethasone, ceftriaxone and vancomycin
B) Start dexametahsone, ceftriaxone, vancomycin and oseltamivir
C) Start N-acetylcysteine infusion
D) Urinary alkalinization followed by hemodialysis
E) Treat with septrin and request HIV testing

Answer: D

MULTIPLE CHOICE QUESTIONS-8

1. A 24-year-old Asian man presents to his family physician with one episode of painless gross
hematuria. He is otherwise healthy with no significant family history. On physical examination,
his blood pressure is normal and there are no other abnormalities. Laboratory examination shows
a normal metabolic profile, hemogram, liver enzymes, and renal function tests, as well as a
negative urine culture. Serologic tests including complement evels, antinuclear antibody,
antineutrophil cytoplasmic antibody, and a hepatitis panel are all negative. Urine examinations
show 3+ blood, 3+ protein, and many RBCs and RBC casts on microscopic examination. His
renal ultrasound is normal. A repeat physical examination and laboratory tests after 1 week show
1+ heme, 5 to 10 RBCs, and trace urinary protein, which quantitates to 150 mg/day. Your
recommendation at this point should be:
A) Continue observation.
B) Renal biopsy.
C) Empiric therapy with alternate day steroids.
D) Genetic testing for patient and his family.
E) Urology referral

Answer: A

2. A 69-year-old man presents with confusion. His carers state that over the last month he has
become increasingly lethargic, irritable and confused. Despite maintaining a good appetite, he
has lost 10 kg in the last month. Blood results are as follows:
Sodium 125 mmol/L
Potassium 4 mmol/L
Urea 3
Glucose (fasting) 6 mmol/L
Urine osmolality 343 mmol/L
The most likely diagnosis is:
A) Hypothyroidism
B) Dilutional hyponatraemia
C) Addison’s disease
D) Acute tubulointerstitial nephritis
E) Syndrome of inappropriate anti-diuretic hormone (SIADH)

Answer: E

3. You see a 19-year-old Caucasian man in your clinic who presents with a history of transient
jaundice. On direct questioning, you ascertain that the jaundice is noticeable after periods of
increased physical activity and subsides after a few days. The patient has no other symptoms and
physical examination is unremarkable. Full blood count is normal (with a normal reticulocyte
count) and liver function tests reveal a bilirubin of 37 μmol/L. The most appropriate management
is:
A) Reassure and discharge
B) Start on a course of oral steroids
C) Request abdominal ultrasound
D) Request MRCP
E) Refer to Hematology

Answer: A

4. A 60-year-old man presented with epigastric and right hypochondrial pain associated with
nausea, vomiting and fever. He consumed alcohol almost daily for the last 30 years. He recalled
that a surgeon had previously suggested a gall bladder surgery but he did refused to have it. On
exam: temp. 38°C, B.P. 120/70, pulse 100/min. Sclerae were yellow. He had eight spider nevi on
his upper body. Chest and heart exam were normal. Abdomen was tender in the right upper
quadrant but no mass was palpable. There was no detectable ascites and rectal examination was
unremarkable. Hemoglobin 14.2, WBC count 17,000, Platelets 110,000, Serum albumin 30 g/L,
total bilirubin 70 (1-22), ALT 70 (< 40), alkaline phosphatase 500 U/L (45-105), amylase 100
U/L (60-180), INR 1.5 What is the likely diagnosis?
A) Cholangitis
B) Acute on chronic pancreatitis
C) Alcoholic hepatitis
D) Acute cholecystitis
E) Empyema of the gall bladder

Answer: A

5. A 45-year-old man collapses at home and is brought to accident and emergency. He has a
fever at 39.5°C and blood pressure is 90/60 mmHg, although he is in a lucid state. Bruises can be
seen on his skin which he remembers being present before he fell. Blood tests show the patient to
have a normocytic anaemia with a low platelet count and increased fibrin split products. The
most likely diagnosis is:
A) Warm autoimmune haemolytic anaemia
B) Cold autoimmune haemolytic anaemia
C) Paroxysmal nocturnal haemoglobinuria
D) Disseminated intravascular coagulation
E) Thalassaemia minor

Answer: D

6. A 40-year-old man with cryptogenic liver cirrhosis presents with acute onset of confusion. His
wife reports that over the last few days his abdomen has become increasingly distended, however
she is more concerned about him being disoriented to time and place. He takes spironolactone
100 mg daily, furosemide 40 mg bid and lactulose 10 mg bid. Exam: He is confused to time and
place and is afebrile. Icteric sclerae , spider nevi and asterixis . There is tense ascites with
distended abdominal veins is also noted. WBC count 10,200, Hemoglobin 10.5, Platelets 35,000,
INR 1.7, Serum sodium 128, Serum creatinine 90, ALT 60 What should be the next step?
A) Abdominal paracentesis
B) CT-scan of the brain
C) Start IV cefotaxime
D) Start oral neomycin
E) Increase dose of diuretics

Answer: A

7. A 62-year-old woman presents to accident and emergency with a 1-day history of sudden
onset back pain and difficulty walking. She has not opened her bowels or passed urine for the
previous day. She has a past medical history of breast cancer, diagnosed two years earlier and
staged as T2N1M0 disease with oestrogen receptor positive status. She has been treated for her
cancer with a wide local excision and axillary node clearance, followed by radiotherapy,
chemotherapy and tamoxifen. On examination, there is reduced tone in the lower limbs. Power is
diminished throughout the lower limbs, but especially on hip flexion. There is reduced sensation
below the L1 dermatome. What is the most appropriate diagnostic investigation?
A) A full set of bloods, including bone profile
B) Computed tomography (CT) thorax, abdomen and pelvis
C) Magnetic resonance imaging (MRI) spine
D) Bone scan
E) Positron emission tomography (PET) CT

Answer: C

8. A 35-year-old woman comes to your clinic complaining of shortness of breath. It is


immediately apparent that she has a bluish tinge of her face, trunk, extremities, and mucus
membranes. Which of the following is most likely?
A) Atrial septal defect
B) Myocarditis
C) Raynaud’s phenomenon
D) Vasospasm due to cold temperature
E) Connective tissue disease

Answer: A

9. A 50-year-old woman, who has received a recent diagnosis of rheumatoid arthritis, presents to
her GP with ongoing pain and stiffness in her hands and feet. Which joints are usually spared at
onset of rheumatoid arthritis?
A) Proximal interphalangeal joints
B) Distal interphalangeal joints
C) Metacarpophalangeal joints
D) Wrists
E) Metatarsophalangeal joints

Answer: B

10. A 45-year-old woman with unexpected weight loss, loss of appetite and shortness of breath
presents to you in clinic. On examination, there is reduced air entry and dullness to percussion in
the right lung. A pleural tap is performed and the aspirate samples sent for analysis. You are told
that the results reveal a protein content of >30 g/L. From the list below, select the most likely
diagnosis:
A) Bronchogenic carcinoma
B) Congestive cardiac failure
C) Liver cirrhosis
D) Nephrotic syndrome
E) Meig’s syndrome

Answer: A

11. A 79-year-old woman presents to her GP with pain in the left knee. This is particularly bad in
the evenings and is stopping her from sleeping. The GP explains that her discomfort is most
likely due to osteoarthritis and arranges for her to have
an x-ray of the knee. Which of the following descriptions are most likely to describe the x-ray?
A) Reduced joint space, subchondral sclerosis, bone cysts and osteophytes
B) Increased joint space, subchondral sclerosis, bone cysts and osteophytes
C) Reduced joint space, soft tissue swelling and peri-articular osteopenia
D) Increased joint space, soft tissue swelling and peri-articular osteopenia
E) Normal x-ray

Answer: A

12. A 52-year-old man is referred for newly diagnosed diabetes. He reports a two month history
of polyuria and polydipsia. He has no past history of note and was not taking any regular
medications, although he reports recent weak erections. On examination, liver edge is palpable 2
cm below the right costal margin and his testes are found to be small. HbA1c 10.8,
Hemoglobin15.1 g/dL, Platelets 341 x109/L, Serum iron 50 µmol/L (12-30), Transferrin
saturation 85% (<50%). Which investigation is most likely to assist in making a diagnosis?
A) Genetic analysis for C282Y and H63D mutations
B) Red cell transketolase
C) Serum ferritin
D) Total iron binding capacity (TIBC)
E) Ultrasound of the abdomen

Answer: A

13. A 49-year-old man complains of sudden onset, painless unilateral visual loss lasting about a
minute. He describes ‘a black curtain coming down’. His blood pressure is 158/90, heart rate 73
bpm. There is an audible bruit on auscultation of his neck. His past medical history is
insignificant other than deep vein thrombosis of his right leg ten years ago. The most likely
diagnosis is:
A) Retinal vein thrombosis
B) Retinal artery occlusion
C) Amaurosis fugax
D) Optic neuritis
E) Acute angle glaucoma

Answer: C

14. A 69-year-old man present with a 2-week history of abdominal pain which has worsened
over the last few days. On examination, the patient is jaundiced and the abdomen is distended
with tenderness in the epigastric region. In addition, there is a smooth hepatomegaly and shifting
dullness. Which of the following is a cause of hepatomegaly?
A) Iron deficiency anaemia
B) Budd–Chiari syndrome
C) Ulcerative colitis
D) Crohn’s disease
E) Left-sided heart failure

Answer: B

15. A 79-year-old woman is admitted to the coronary care unit (CCU) with unstable angina. She
is started on appropriate medication to reduce her cardiac risk. She is hypertensive, fasting
glucose is normal and cholesterol is 5.2. She is found to be in atrial fibrillation. What is the most
appropriate treatment?
A) Aspirin and clopidogrel
B) Digoxin
C) Cardioversion
D) Aspirin alone
E) Warfarin
Answer: E

16. A 53-year-old healthy man was found to have a fasting plasma glucose of 7.3 mmol. Further
testing showed HbA1c of 6.2. What should be done next?
A) Repeat HbA1c
B) Repeat fasting glucose
C) Advise home glucose monitoring
D) lifestyle changes, repeat all tests in 3 months
E) Measure 2-hour postprandial glucose

Answer: B

17. A 66-year-old woman presents to accident and emergency with a 2-day history of shortness
of breath. The patient notes becoming progressively short of breath as well as a sharp pain in the
right side of the chest which is most painful when taking a deep breath. The patient also
complains of mild pain in the right leg, though there is nothing significant on full cardiovascular
and respiratory examination. Heart rate is 96 and respiratory rate is 12. The patient denies any
weight loss or long haul flights but mentions undergoing a nasal polypectomy 3 weeks ago. The
most likely diagnosis is:
A) Muscular strain
B) Heart failure
C) Pneumothorax
D) Angina
E) Pulmonary embolism

Answer: E

18. A 78-year-old woman is admitted with heart failure. The underlying cause is determined to
be aortic stenosis. Which sign is most likely to be present?
A) Pleural effusion on chest x-ray
B) Raised jugular venous pressure (JVP)
C) Bilateral pedal oedema
D) Bibasal crepitations
E) Atrial fibrillation

Answer: D

19. A 22-year-old woman complains of dizziness and feeling light-headed. She works in an
office and most frequently experiences this when standing up to visit the toilet. She has never
fainted. The patient has lost 5 kg, but attributes this to eating more healthily. She has noticed a
recent scar on the back of her hand which has started to turn very dark. The most appropriate
investigation is:
A) Synacthen test
B) Low-dose dexamethasone test
C) Cortisol measurement
D) Urinary free cortisol measurement
E) Abdominal ultrasound (US) scan

Answer: A

20. A 71-year-old man with atrial fibrillation is seen in clinic following an episode of syncope.
He describes getting a poor night’s sleep and, as he got out of bed in the morning, feeling dizzy
for a couple of seconds before the lights dimmed around him. He was woken a couple of seconds
later by his wife who had witnessed the event. She says he went pale and fell to the floor and his
arms and legs jerked. After waking, he was shaken but was ‘back to normal’ a few minutes
after the event. His medication includes aspirin, atenolol and frusemide. What is the most likely
diagnosis?
A) Vasovagal syncope
B) Orthostatic hypotension
C) Cardiogenic syncope
D) Transient ischaemic attack (TIA)
E) Seizure

Answer: B

21. A 29-year-old man presents with a 4-week history of polyuria and extreme thirst. The patient
denies difficulty voiding, hesitancy or haematuria, although the urine is very dilute. The patient
does not believe he has lost any weight and maintains a good diet. No findings are found on
urine dipstick. The most appropriate investigation is:
A) Serum osmolality
B) Fasting plasma glucose
C) Urinary electrolytes
D) Magnetic resonance imaging (MRI) scan of the head
E) Water deprivation test

Answer: E

22. You are discussing a patient with your registrar who has become acutely short of breath on
the ward. After performing an arterial blood gas, you have high clinical suspicion that the patient
has a pulmonary embolism. Which of the following is the investigation of choice for detecting
pulmonary embolism?
A) Magnetic resonance imaging (MRI) of the chest
B) High-resolution CT chest (HRCT)
C) Chest x-ray
D) Ventilation/perfusion scan (V/Q scan)
E) CT pulmonary angiogram (CT-Pa)

Answer: E

23. A patient is admitted with pneumonia. A murmur is heard on examination. What finding
points to mitral regurgitation?
A) Murmur louder on inspiration
B) Murmur louder with patient in left lateral position
C) Murmur louder over the right 2nd intercostal space midclavicular line
D) Corrigan’s sign
E) Narrow pulse pressure

Answer: B

24. A 43-year-old woman suffers from Crohn’s disease. A blood test shows the following results:
Haemoglobin 10.5 g/dL
MCV 120 fL
Platelet count 300× 109/L
The most likely diagnosis is:
A) Vitamin B12 deficiency
B) Iron deficiency
C) Hypothyroidism
D) Folic acid deficiency
E) Anaemia of chronic disease

Answer: A

25. A 27-year-old woman develops left leg swelling during week 20 of her pregnancy. Left
lower extremity ultrasonogram reveals a left iliac vein deep vein thrombosis. What is the best
next step?
A) bed rest
B) catheter-directed thrombolysis
C) enoxaparin
D) inferior vena cava filter placement
E) Repeat doppler U/S after 1 week

Answer: E

26. A 67-year-old woman is admitted to accident and emergency with pyrexia (38.1°C) and a
cough productive of green sputum. The observations show a pulse rate of 101, BP 80/60 and
respiratory rate of 32. She is alert and orientated in space and time. Blood results reveal a WCC
of 21, urea of 8.5 and chest x-ray shows a patch of consolidation in the lower zone of the right
lung. She is treated for severe community-acquired pneumonia. Which of the following is the
correct calculated CURB-65 score?
A) 6
B) 8
C) 4
D) 0
E) 1

Answer: C
27. A 69-year-old man presents to clinic with a six-month history of progressive lower back pain
which radiates down to his buttock. He found the pain was exacerbated while taking his daily
morning walk and noticed that it eased going uphill but worsened downhill. He stopped his daily
walks as a result and he now walks only slowly to the shops when he needs to, taking breaks to
sit down and ease the pain. He has a history of hypertension, diabetes and prostatic hyperplasia.
What is the diagnosis?
A) Peripheral vascular disease
B) Osteoporotic fracture
C) Spinal stenosis
D) Sciatica
E) Metastatic bone disease

Answer: C

28. A 47-year-old woman is evaluated for difficult-to-control hypertension. She was previously
treated for hypokalemia. On physical examination, temperature is 36.0 °C (96.8°F), blood
pressure is 178/100 mm Hg, pulse rate is 58/min, respiration rate is 16/min, and BMI is 29. No
abdominal bruit is detected. Funduscopic examination shows mild arteriolar narrowing.
Laboratory studies:
Electrolytes
Sodium 143 meq/L (143 mmol/L)
Potassium 3.5 meq/L (3.5 mmol/L) (after replacement
therapy)
Chloride 101 meq/L (101 mmol/L)
Bicarbonate 33 meq/L (33 mmol/L)
Aldosterone
Baseline 23 ng/dL (635 pmol/L)
3 Days after high salt intake 15 ng/dL (414 pmol/L)
Renin activity
Baseline <0.1 ng/mL/h (0.1 μg/L/h)
3 Days after high salt intake <0.1 ng/mL/h (0.1 μg/L/h)
Aldosterone to renin activity ratio >50
Which of the following is the most appropriate next step in management?
A) Adrenalectomy
B) Bilateral adrenal vein catheterization
C) CT of the adrenal glands
D) Duplex ultrasonography of the renal arteries

Answer: C

29. Decreased or absent haptoglobin levels are seen in the following conditions EXCEPT
A) Hemolytic anemia
B) Genetic disorders
C) Acute hepatitis
D) Pregnancy
E) Burns
Answer: C

30. A 41-year-old man is diagnosed with iron deficiency anemia and is found to have heme-
positive stools. Colonoscopy reveals a large ulcerated tumor in his transverse colon. He also has
two smaller polyps in his ascending colon. Pathologic examination of the tumor biopsy reveals
adenocarcinoma, while biopsies of the polyps confirm that these are adenomas. His sister has
been diagnosed with
uterine cancer, and two cousins have died of colon cancer. All of the following are true
statements about this case except
A. Referral for genetic counseling is indicated
B. He is at increased risk for other epithelial-derived tumors
C. He likely has familial adenomatous polyposis (FAP), with a germ line mutation in the APC
gene
D. His condition is often associated with a defect in DNA mismatch repair

Answer: B

31. Concerning adrenal physiology, which of the following hormones is secreted by the zona
glomerulosa?
A) Adrenaline
B) Noradrenaline
C) Cortisol
D) Aldosterone
E) Angiotensin II

Answer: D

32. A 76-year-old gentleman is referred with an international normalized ratio (INR) of 8.4. He
normally takes warfarin for a prosthetic mitral valve. He had an epistaxis lasting 15 minutes this
morning. He is otherwise asymptomatic and haemodynamically stable. Having stopped his
warfarin for tonight, Which of the following is the most appropriate course of action?
A) 5 mg iv vitamin K
B) 0.5 mg orally (po) vitamin K
C) 4 units of fresh frozen plasma iv
D) 50 units/kg of prothrombin complex concentrate
E) Recheck INR in 24 hours

Answer: E

33. A 64-year-old who suffers from haemochromatosis is seen in A&E with a 2-day history of
pain and swelling in his right knee. On examination it is swollen and he has decreased range of
movement. You aspirate his knee. What will the aspirate be most likely to show?
A) Gram-positive cocci in clusters
B) Positively bifringent crystals
C) Gram-positive cocci in chains
D) Inflammatory cells with haemosiderin deposition
E) Negatively bifringent crystals

Answer: B

34. A 50-year-old male with a history of alcohol dependence and chronic hepatitis C presents to
your clinic with three months of fatigue, weakness and weight loss. He has also noted some
“yellowing of my eyes.” You suspect cirrhosis. All the following are clinical signs of cirrhosis
except
A) Arthralgia
B) Asterixis
C) Dupuytren’s contracture
D) Haemorrhoids
E) Testicular atrophy

Answer: A

35. A 75-year-old lady presents with persistent diarrhoea and weight loss. Twenty years
previously she had radiotherapy for carcinoma of the ovary. Thirty years ago she lived in Hong
Kong for 18 months. Her father died of cancer of the colon aged 62 years.
Hb 9.4 g/dL (11.5-16 g/dL)
WCC 9.6 x 109/L (4-11 x 109/L)
Platelets 234 x 10/L (150-400 x 109/L)
B12 153 pmol/L (160-900pmol/L)
Red cell folate 21 nmol/L (130-630 nmol/L)
Ferritin 5 ug/L (20-250 ug/L)
Erythrocyte sedimentation rate (ESR) 36 mm/hour
Which of the following is the most likely diagnosis?
A) Radiation enteritis
B) Carcinoma of the colon
C) Crohn's disease
D) Tropical sprue
E) Acquired lactose intolerance

Answer: A

36. A phlebotomist sustains a needlestick injury whilst taking blood from a HIV-positive patient.
Which one of the following has been shown to most reduce the risk of HIV transmission?
A) Thorough immediate washing of the injury site with water
B) Oral lamivudine therapy for 1 month
C) Oral zidovudine therapy for 1 month
D) Oral ritonavir therapy for 1 month
E) Oral triple therapy for 1 month

Answer: E
37. A 33-year-old female is seen in the maternity department.she suffers from SLE. Her child
was born with profound bradycardia. Which of the following antibodies is most likely to be
present?
A) Anti-Ro
B) Anti-La
C) Anti-Jo
D) Anti-Scl-70
E) Antiphospholipid antibodies

Answer: A

38. A 67-year-old man presents with nausea and pain in his right upper quadrant. On
examination, he has tender hepatomegaly with an irregular edge. His blood tests are as follows:
Hb 8 g/dL (11.5-16 g/dL)
WCC 17.8 x 109/L (4-11 x 109/L)
Platelets 104 x 109/L (150-400x109/L)
Neutrophils 68%
Normoblasts 8%
Myeloblasts 8%
Myelocytes 5%
Metamyelocytes 5%
Lymphocytes 15%
What is the most likely cause of his anaemia?
A) Gaucher's disease
B) Leukaemoid reaction secondary to acute infection
C) Myelosclerosis
D) Malignant infiltration of bone marrow
E) Osteopetrosis

Answer: D

39. A 32-year-old woman presents with left loin pain and haematuria. She is known to suffer
with Crohn's disease.
Which of the following is likely to be the aetiology of this presentation?
A) Hypercalciuria
B) Hyperbilirubinaemia
C) Hyperuricaemia
D) Hyperoxaluria
E) Type 1 renal tubular acidosis

Answer: D

40. Regarding glucocorticoid-induced hyperglycemia all the following statements are correct
EXCEPT.
A) The risk of new onset diabetes mellitus is related to the total glucocorticoid dose and d)ration
of therapy.
B) Raised fasting plasma glucose is the earliest abnormality.
C) Reduced insulin sensitivity is the predominant mechanism.
D) Insulin is the most effective and safety therapy
E) Long term use of thiazolidinediones in combination with glucocorticoid is contraindicated.

Answer: B

41. A 37-year-old asymptomatic woman presents for a routine physical examination. Physical
examination reveals a 2-cm right-sided thyroid nodule that is firm, nontender and moves with
swallowing. The rest of the physical examination is unremarkable. Lab Work: TSH 1.8 μU/mL.
She is referred for fine-needle aspiration biopsy of the thyroid nodule. Which of the following is
true regarding interpreting the cytologic results of thyroid fine-needle aspiration biopsy?
A) Follicular carcinoma can be accurately diagnosed.
B) Most fine-needle aspiration biopsy specimens reveal malignant cells.
C) Follicular adenoma can be accurately diagnosed.
D) Papillary carcinoma can be accurately diagnosed.
E) Cystic nodules are more likely than solid nodules to provide diagnostic material.

Answer: D

42. A 40 year old construction laborer admitted to the hospital with sudden onset of dysphagia
and inability to open his mouth. During examination he was conscious but unable to speak, in
pain and pointing to his back. His abdomen was rigid. His upper and lower limb examination
was normal. The most important initial step in the management of this patient is:
A) Perform an urgent CT scan of the neck to exclude pharyngeal abscess
B) Start immediate Ceftriaxone and Vancomycin
C) Start I.V Penicillin,Tetanus Ig and Lorazepam
D) Start I.V Metronidazole,Tetanus Ig and Lorazepam
E) Search for a wound, perform wound cleaning or debridement followed by I.V penicillin and
Tetanus Ig

Answer: D

43. 74-year-old man 70 Kg enters the ICU with fever and hypotension. He has a history of
kidney stones with multiple previous bouts of urinary tract infection. 1.5 L of Normal saline
bolus started, Urine and blood cultures are obtained and he was started on Tazocin and
Vancmycin, his BP remain 70/40 and additional 500 ml of Normal saline were infused, CVC
placed and norepinephrine started 10 mcg/min, He was intubated to decrease work of breathing
and his hemodynamic data as follow:
BP: 80/45 (55), CVP: 6, Urine output 20 ml/hr, Lactate: 6, ScVO2: 50%.
What is the next step in managing this patient?
A) Start stress dose steroids for refractory septic shock.
B) Add vasopressin to the current regimen for refractory septic shock.
C) Start Dobutamine to increase cardiac output and oxygen delivery.
D) Bolus IV normal saline to keep CVP 8-12.
E) Start furosamide to keep urine output more than 35 ml/hr.
Answer: D

44. A 59-year-old lady is admitted with a 30 minute history of heavy central chest pain
associated with nausea and sweating. Her ECG shows ST elevation in leads V1, V2, V3 and V4.
Which of the following coronary arteries is most likely to be occluded?
A) Circumflex artery
B) Left anterior descending artery
C) Obtuse marginal artery
D) Posterior descending artery
E) Right coronary artery

Answer: B

45. A 68-year-old man presents with a one-month history of dyspnoea and a 3kg weight loss. On
examination there were signs of a large left pleural effusion, confirmed on Chest X-ray.
Pleural fluid analysis:
Protein 38 g/L
Cytology a few lymphocytes and red blood cells.
Which one of the following investigations should be considered next?
A) Bronchoscopy
B) CT scan of thorax Correct
C) Repeat pleural aspiration with biopsy
D) Thoracoscopic pleural biopsy
E) Tuberculin test

Answer: B

46. A 65-year-old male is admitted with a two hour history of central chest pain associated with
sweating and nausea.
A myocardial infarction is suspected. Which of the following is an indication for thrombolytic
therapy?
A) 1mm ST elevation in leads II, III and aVF.
B) 1mm ST elevation in leads V2 and V6
C) 2mm ST depression in leads V2-4
D) Q waves in leads V2-4
E) T wave inversion in lead V3-5

Answer: A

47. A 32-year-old female presents with 3-day history of colicky abdominal pain and fatigue.
Lab: HB 2.9 g/dl, Haematocrit 22%, MCV 78, platelets 60000, retic count 9%, direct and
indirect Coomb’s test (-), ferritin 10, abdominal US shoes portal vein thrombosis.
Urine analysis: hemosiderin
The most likely diagnosis is:
A) Factor V Leiden mutation
B) Paroxysmal cold hemoglobinuria
C) Warm autoimmune hemolytic anemia
D) Aplastic anemia
E) Paroxysmal nocturnal hemoglobinuria

Answer: E

48. An 18-year-old attending the A+E department is noted to have central cyanosis. She is
perfectly well but was told to go to A+E by her friends who said she looked blue. What is the
most likely cause?
A) Carbon Monoxide Poisoning
B) Lead Poisoning
C) Drinking water contaminated with nitrates Correct
D) Anorexia Nervosa
E) Severe Anaemia

Answer: C

49. A 58-year-old woman comes to the emergency department complaining of crampy left upper
quadrant pain that is exacerbated by fatty foods. She has a history of diabetes, hyperlipidemia,
and gallstones and her medications include glyburide, simvastatin, and aspirin. She denies any
alcohol or drug use. She is morbidly obese and her temperature is 37.9 C (100.2 F), blood
pressure is 102/87 mm Hg, pulse is 105/min, and respirations are 23/min. On examination, her
lungs are clear to auscultation bilaterally. Her cardiac sounds are muffled, although her cardiac
rhythm is regular. No murmurs are audible. She has definite left upper quadrant tenderness to
palpation, without rebound or guarding. Rectal examination shows guaiac-negative brown stool.
Her amylase and lipase levels are elevated. The most appropriate next step is to order a
A) an abdominal ultrasound
B) a chest radiograph
C) an electrocardiogram
D) a HIDA scan
E) an upper endoscopy

Answer: A

50. Which of the following statements is NOT true of primary pulmonary tuberculosis:
A) It is characteristically asymptomatic
B) Miliary spread is commoner in a younger age group
C) The initial immunological response causes hilar lympadenopathy
D) Pleural effusion occurs before tuberculin skin testing is positive
E) A positive tuberculin skin test develops within two weeks of infection

Answer: E

MULTIPLE CHOICE QUESTIONS-9


1. A 78-year-old woman is treated for depression with nortriptyline. Which of the following is
the most common side effect of nortriptyline?
A) impaired cardiac contractility
B) heart block
C) weight loss
D) anticholinergic side effects
E) diarrhea

Answer: D

2. During a neurologic examination, you ask a patient to stand with both arms fully extended and
parallel to the ground with his eyes closed for 10 seconds. What is the name of this test?
A) Babinski sign
B) Dysdiadochokinesis
C) Lhermitte symptom
D) Pronator drift
E) Romberg sign

Answer: D

3. A person with liver disease caused by Schistosoma mansoni would be most likely to have
what condition?
A) Ascites
B) Esophageal varices
C) Gynecomastia
D) Jaundice
E) Spider nevi

Answer: B

4. All of the following hormones is produced by the anterior pituitary EXCEPT:


A) Adrenocorticotropic hormone
B) Growth hormone
C) Oxytocin
D) Prolactin
E) Thyroid-stimulating hormone

Answer: C

5. The following statements concerning pulmonary Stenosis are correct


A) It is the commonest cardiac abnormality in Turner's syndrome
B) The chest x-ray plethoric lung fields
C) There is a recognized association with carcinoid syndrome
D) An ejection click indicates that the Stenosis is sub-valvular
E) The pulmonary component of the second sound is accentuated when the Stenosis is severe
Answer: C

6. A 36-year-old woman with acquired immunodeficiency syndrome (AIDS) and a CD4 count of
35/μL presents with odynophagia and progressive dysphagia. The patient reports daily fevers and
a 20-lb weight loss. The patient has been treated with clotrimazole troches without relief. On
physical examination, the patient is cachectic with a body mass index (BMI) of 16 and a weight
of 86 lb. The patient has a temperature of 38.2°C (100.8°F). She is noted to be orthostatic by
blood pressure and pulse. Examination of the oropharynx reveals no evidence of thrush. The
patient undergoes esophagogastroduodenoscopy (EGD), which reveals serpiginous ulcers in the
distal esophagus without vesicles. No yellow plaques are noted. Multiple biopsies are taken that
show intranuclear and intracytoplasmic inclusions in large endothelial cells and fibroblasts. What
is the best treatment for this patient’s esophagitis?
A) Ganciclovir
B) Glucocorticoids
C) Fluconazole
D) Foscarnet
E) Thalidomide

Answer: A

7. A 45-year-old man reports to his primary care physician that his wife has noted coarsening of
his facial features over several years. In addition, he reports low libido and decreased energy.
Physical examination shows frontal bossing and enlarged hands. An MRI confirms that he has a
pituitary mass. Which of the following screening tests should be ordered to diagnose the cause of
the mass?
A) 24-Hour urinary free cortisol
B) Adrenocorticotropic hormone (ACTH) assay
C) Growth hormone level
D) Serum insulin-like growth factor-1 (IGF-1) level
E) Serum prolactin level

Answer: D

8. Bronchopulmonary aspergillosis is characterized by all of the following EXCEPT:


A) Underlying asthma
B) Central bronchiectasis
C) Elevated serum immunoglobulin E level
D) Positive serum precipitins for Aspergillus
E) Positive delayed hypersensitivity skin test to Aspergillus antigens

Answer: B

9. The following are complications of nephrotic syndrome with the exception of


A) Acute renal failure
B) Accelerated hypertension
C) Hypocalcaemia
D) Pneumococcal infection
E) Venous thrombosis

Answer: B

10. You have a febrile 37-year-old male with a very high WBC count, most of which are blasts.
Most likely, he has an acute leukemia. A bone marrow has not yet been done, and it is unknown
if this is an acute myeloid leukemia or an acute lymphoblastic leukemia. Which of the following
statements is NOT TRUE?
A) The patient’s fever is likely due to the leukemic cells, and antibiotics should be started only if
you identify a specific infection.
B) The patient is at risk for tumor lysis syndrome and should be started on allopurinol.
C) Aggressive inpatient chemotherapy is required for both the treatment of acute myeloid
leukemia and acute lymphoblastic leukemia.
D) You should consult a hematologist/oncologist as soon as possible.

Answer: A

11. A 48-year old female with rheumatoid arthritis presents to the emergency department with 2-
week pain and tightness behind the left knee. Examination reveals cystic swelling over the left
popliteal fossa. Which of the following is the most appropriate next action?
A) Arthrogram of the left knee
B) Synovial biopsy of left knee
C) Ultrasound study of left knee and popliteal fossa
D) Venogram of left lower limb
E) None of the above

Answer: C

12. Concerning pseudomyxoma peritonei: All are true EXCEPT


A) Begins with a mucinous cystadenocarcinoma, most commonly involving either the ovary or
appendix.
B) Rupture of the lesion results in diffuse metastatic spread with implantation of the mucin
secreting lesions on the omentum as well as the peritoneum.
C) Average age of presentation of this disease process is 45-55, with men making up 80% of
cases.
D) Presenting symptoms are non-specific, generally including pain, but also usually with a
palpable mass.
E) A 5-year survival of these patients is only approximately 50%.

Answer: C

13. A 60-year-old man with end stage renal failure from chronic glomerunephritis presents with
acute onset of gross hematuria and mild flank pain. He has been on hemodialysis for 4 years, and
his course otherwise been uneventful. He was afebrile and the hematuria resolved without
intervention. Which of the following is the most appropriate action at this time?
A) Angiography
B) Computed tomography (to rule out acquired cystic kidney disease)
C) Renal ultrasound
D) Intravenous pyelography
E) None of the above

Answer: B

14. A 30-year-old female with systemic lupus erythematosus is recovering from a fracture of the
right femur and right radius following a motor vehicle accident. She has been in the hospital for
5 days. She has a temperature spike of 39.0 °C (102.2 °F). Blood cultures are drawn which
grow yeast, species to be identifi ed. She does not appear ill or toxic. No obvious source of
infection is found. The patient has an indwelling central catheter. In addition to changing the
patient’s central line, which of the following do you recommend?
A) Continued observation.
B) Computed tomography of the chest and abdomen.
C) Start fluconazole.
D) Start liposomal amphotericin B.
E) Repeat blood cultures and treat if positive

Answer: C

15. A 78-year-old woman is admitted to the medical intensive care unit (ICU) with multilobar
pneumonia. On initial presentation to the emergency department, her initial oxygen saturation
was 60% on room air and only increased to 82% on a non-rebreather face mask. She was in
marked respiratory distress and intubated in the emergency department. Upon admission to the
ICU, she was sedated and paralyzed. The ventilator is set in the assist-control mode with a
respiratory rate of 24 breaths/min, tidal volume of 6 mL/kg, FiO2 of 1.0, and positive end-
expiratory pressure of 12 cmH2O. An arterial blood gas measurement is performed on these
settings; the results are pH of 7.20, PCO2 of 32 mmHg, and PO2 of 54 mmHg. What is the cause
of the hypoxemia?
A) Hypoventilation alone
B) Hypoventilation and ventilation-perfusion mismatch
C) Shunt
D) Ventilation-perfusion mismatch

Answer: C

16. Which of the following may be responsible for a hypokalaemic hypertension?


A) Non-classical congenital adrenal hyperplasia
B) Barter's syndrome
C) Diabetic nephropathy
D) Liddle's syndrome
E) Type IV renal tubular acidosis

Answer: D
17. A 63-year-old male recently admitted with sepsis is noted to have a urine output of
approximetely 20 mls per hour. The oliguria is more likely to be due to prerenal failure than
intrinsic renal failure if:
A) A urine free of red blood cells or casts Correct
B) A urine:plasma urea ratio < 3
C) Urine osmolality <350 mOsm/l
D) A blood pressure of 150/90 and good tissue perfusion.
E) Urinary sodium >10mmol/l

Answer: A

18. A 50-year-old female presents with acute chest pain and dyspnea. Examination reveals
bilateral ankle oedema with 24 hr urine protein assessment showing 8g/d (<0.2). Which is the
most likely explanation for these findings?
A) Factor V Leiden Incorrect answer selected
B) Reduced antithrombin III activity This is the correct answer
C) Reduced concentration of Von Willebrand’s factor
D) Reduced fibrinogen concentration
E) Reduced factor VIII

Answer: B

19. A 19-year-old woman became breathless while travelling on an aeroplane. Which one of the
following features most strongly supports a diagnosis of acute hyperventilation related to a panic
disorder?
A) Carpal spasm.
B) Finger paraesthesiae.
C) Hypotension.
D) Light-headedness.
E) Loss of conciousness

Answer: A

20. Which ONE of the following is a recognised feature of achondroplasia?


A) Autosomal recessive inheritance
B) May be diagnosed radiologically at birth
C) Increased liability to pathological fractures
D) Shortened spine
E) Subfertility

Answer: B

21. You are asked to see a 67-year-old woman who was recently discharged from the hospital
following internal fixation of fragility fracture of left femoral neck. She had hypertension and
primary hypothyroidism. Her medications include lisinopril 10 mg and L-thyroxine 100ug per
day. Family history was unremarkable. On physical examination BMI 23.9 kg/m2 BP 130/84
mmHg. Laboratory studies showed: Serum creatinine 1.2 mg/dl Serum calcium 9.2 mg/dl: Serum
PTH 69 pg/ml (N 10-58): Serum 25 hydroxy vitamin D 42 ng/mg (N 30-70): Serum TSH 1.9
mU/l (N 0.4-4.0 u): Bone density measured by DXA revealed: T score : Hip-2.9 Lumbar spine -
2.7. In addition to vitamin D and calcium supplement, which of the following you recommend to
reduce the incidence of another hip fracture?
A) Zoledronate 5mg IV yearly
B) Combined estrogen and progestin.
C) Raloxifene 60 mg per day.
D) Alphacalcidol 1 µg per day

Answer: A

22. A 29-year-old man with episodic abdominal pain and stress-induced edema of the lips,
tongue, and occasionally larynx is likely to have low functional or absolute levels of which of the
following proteins?
A. C1 esterase inhibitor
B. C5A (complement cascade)
C. Cyclooxygenase
D. Immunoglobulin (Ig) E
E. T-cell receptor, α chain

Answer: A

23. Pseudopseudohypoparathyroidism is characterized by all of the following EXCEPT:


A) Hypocalcemia
B) G5 mutation
C) Albright’ s hereditary osteodystrophy
D) inheritance of the genetic defect from the father
E) Normal urinary c-AMP response to parathyroid hormone

Answer: A

24. Most of the manifestations of acute rheumatic fever present approximately 3 weeks after the
precipitating group A streptococcal infection. Which manifestation may present several months
after the precipitating infection?
A) Chorea
B) Erythema marginatum
C) Fever
D) Polyarthritis
E) Subcutaneous nodules

Answer: A

25. A 57-year-old woman with depression and chronic migraine headaches reports several years
of dry mouth and dry eyes. Her primary complaint is that she can no longer eat her favorite
crackers, although she does report photosensitivity and eye burning on further questioning. She
has no other associated symptoms. Examination shows dry, erythematous, sticky oral mucosa.
All of the following tests are likely to be positive in this patient EXCEPT:
A) La/SS-B antibody
B) Ro/SS-A antibody
C) Schirmer I test
D) Scl-70 antibody
E) Sialometry

Answer: D

26. Which of the following has a well-established association with gastroesophageal reflux?
A) Chronic sinusitis
B) Dental erosion
C) Pulmonary fibrosis
D) Recurrent aspiration pneumonia
E) Sleep apnea

Answer: B

27. A 68-year-old woman presents with a left arm tremor. She has had the tremor for about 3
years and recently was noticed to have developed a side-to-side head tremor as well. Her tremor
worsens when she performs movements that involve her bringing her hand to her face, such as
smoking or drinking coffee. Of note is that if she drinks beer, the tremor seems to improve.
Which of the following is the likely etiology?
A) Gilles de la Tourette syndrome
B) Parkinson disease
C) Myasthenia gravis
D) Benign essential tremor
E) Huntington disease

Answer: D

28. The following are recognized features of Pancoast's tumour except:


A) ipsilateral Horner's syndrome
B) wasting of the dorsal interossei
C) pain in the arm radiating to the fourth and fifth fingers
D) erosion of the first rib
E) weakness of abduction at the shoulder

Answer: E

29. Major risk factor for osteoporosis includes all of the following EXCEPT.
A) Tobacco smoking
B) Glucocorticoid therapy.
C) Primary hyperparathyroidism.
D) History of previous fragility fracture after the age 40 year.
E) Hypogonadism

Answer: D

30. A 20-year-old male student is assessed for shortness of breath that occurs whilst running. He
has no other symptoms and does not smoke. Examination, full blood count, and chest X-ray are
normal. Which of the following is most likely to be helpful in confirming the suspected diagnosis?
A) Arterial blood gas studies before and after exercise
B) Determination of lung volumes and diffusing capacity
C) Measurement of venous blood lactate before and after exercise
D) Spirometry before and after administration of bronchodilators
E) Spirometry before and after exercise

Answer: E

31. A 65-year-old man has the following blood results on admission to hospital:
Na 132 mmol/L (135-145 mmol/L)
K 8.6 mmol/L (3.5-5 mmol/L)
Urea 42.4 mmol/L (2.5-6.7 mmol/L)
Creatinine 1178 um0l/L (70-150 umol/L)
The patient appears unwell and the ECG shows tented T waves, absent P waves and widened
QRS. Which of the following therapeutic interventions should be performed first?
A) Immediate haemodialysis
B) Intravenous insulin and dextrose
C) Intravenous calcium gluconate
D) Nebulized salbutamol
E) Oral calcium resonium

Answer: C

32. A 55-year-old man with chronic hepatitis C is being considered for liver transplantation. The
patient has cirrhosis that was documented by liver biopsy 10 years ago. For the past 3 months, he
has had ascites and edema, which are poorly controlled with diuretics. Lactulose was recently
begun because of confusion. Which of the following combinations of laboratory studies will be
most helpful in estimating his survival over the next 6 months?
A) Serum total bilirubin and INR
B) Serum aspartate aminotransferase and gamma globulin
C) Serum alanine aminotransferase and hepatitis C RNA (HCV RNA)
D) Serum alkaline phosphatase and ammonia
E) Serum albumin and ?-glutamyltransferase

Answer: A

33. Signs and symptoms of opioid withdrawal include all of the following EXCEPT
A) Increased blood pressure (BP) and heart rate
B) Seizures
C) Abdominal cramps
D) Jerking of the legs
E) Hyperthermia

Answer: B

34. Which of the following β-adrenergic antagonists is a nonselective β1 and β2 blocker?


A) Atenolol
B) Betaxolol
C) Esmolol
D) Metoprolol
E) Nadolol

Answer: E

35. In primary hypothyroidism increased requirements for L thyroxine occur in the following
conditions EXCEPT:
A) Nephrotic syndrome with massive proteinuria.
B) Pregnancy.
C) Achalasia
D) Helicobacter pylori associated chronic gastritis.
E) Use of estrogen replacement therapy.

Answer: C

36. A 70-year-old man is admitted with new-onset atrial fibrillation with an apical rate of
120/minute and a BP of 100/70 mm Hg. Shortly after admission he develops moderate lower
abdominal pain and diarrhea with dark red blood.
Which is the most appropriate course of action?
A) Cardioversion
B) Unfractionated heparin
C) Mesenteric angiography
D) Laparotomy
E) Digoxin

Answer: A

37. A 73-year-old male presents to the clinic with 3 months of increasing back pain. He localizes
the pain to the lumbar spine and states that the pain is worst at night while he is lying in bed. It is
improved during the day with mobilization. Past history is notable only for hypertension and
remote cigarette smoking. Physical examination is normal. Laboratory
studies are notable for an elevated alkaline phosphatase. A lumbar radiogram shows a lytic lesion
in the L3 vertebra. Which of the following malignancies is most likely?
A) Gastric carcinoma
B) Non-small cell lung cancer
C) Osteosarcoma
D) Pancreatic carcinoma
E) Thyroid carcinoma

Answer: B

38. A 36-year-old African-American woman with systemic lupus erythematosus presents with
the acute onset of lethargy and jaundice. On initial evaluation, she is tachycardic, hypotensive,
appears pale, is dyspneic, and is somewhat difficult to arouse. Physical examination reveals
splenomegaly. Her initial hemoglobin is 6 g/dL, white blood cell count is 6300/μL, and platelets
are 294,000/ μL. Her total bilirubin is 4 g/dL, reticulocyte count is 18%, and haptoglobin is not
detectable. Renal function is normal, as is urinalysis. What would you expect on her peripheral
blood smear?
A) Macrocytosis and PMN’s with hypersegmented nuclei
B) Microspherocytes
C) Schistocytes
D) Sickle cells
E) Target cells

Answer: B

39. A 22 year-old lady presents to her GP with facial swelling, most marked in the morning, and
shortness of breath. Urinalysis shows protein +++. Results were as follows:
24-hour urinary protein 4.2 g;
Haemoglobin(Hb) 10.8 g/l (11.5-16.0 g/L);
Urea 22.8 mmol/L (2.5-6.5 mmol/L);
Creatinine 374 µmol/L (70-150 umol/L)
C3 0.24 g/L (0.55-1.2g/L)
C4 0.29g/L (0.2-0.5 g/L)
Which of the following is the most likely diagnosis?
A) Minimal change nephropathy
B) Systemic lupus erythematosus
C) Rheumatoid arthritis
D) Cryoglobulinaemia
E) Diabetes mellitus

Answer: B

40. A 31-year-old male with hemophilia A is admitted with persistent gross hematuria. He denies
recent trauma or any history of genitourinary pathology. The examination is unremarkable.
Hematocrit is 28%. All the following are treatments for hemophilia A except
A) desmopressin (DDAVP)
B) fresh-frozen plasma (FFP)
C) cryoprecipitate
D) recombinant factor VIII
E) plasmapheresis
Answer: E

41. A 50-year-old male is taken to the General Practitioner by his long suffering wife. His
snoring (which has been steadily increasing in loudness over the past 18 months) is troublesome
at home. She says that he makes noises and moves around whilst asleep. He reports no problems
with sleeping. He does admit to gaining 20 kg in weight over the past one year, and to falling
asleep during the day. A sleep study is performed. Which of the following findings would be
most compatible with this man's clinical presentation?
A) Fragmented sleep, cessations of airflow measured at the nose accompanied by an increase in
oesophageal pressure swings and episodic oxygen desaturation.
B) Normal sleep quality, bradycardic episodes, oxygen desaturation but normal airflow.
C) Normal sleep quality but cessations of airflow measured at the nose with decreased abdominal
wall motion during these flow cessations.
D) Progressive oxygen desaturation during the night and alternating periods of hyperventilation
and hypoventilation.
E) Tachycardia, sleep fragmentation, episodes of hypoventilation with minimal oxygen
desaturation.

Answer: A

42. A 50-year-old female presents with acute chest pain and dyspnoea. Examination reveals
bilateral ankle oedema with 24 hr urine protein assessment showing 8g/d (<0.2). Which is the
most likely explanation for these findings?
A) Factor V Leiden
B) Reduced antithrombin III activity
C) Reduced concentration of Von Willebrand’s factor
D) Reduced fibrinogen concentration
E) Reduced factor VIII

Answer: B

43. A 60-year-old woman is establishing care in your clinic. She has occasional numbness and
tingling in her fingers and toes and has noted some numbness around her mouth, especially when
she is stressed or anxious. She had thyroid surgery for Graves disease about 2 years ago and
takes 100 µg of levothyroxine and 1 tablet of calcium daily. On exam, her blood pressure is
130/80, pulse 80, and she has cramping in her right forearm and fingers when the blood pressure
cuff is attached. Based on this history and exam, which of the following is most likely?
A) Calcium 8.5, PTH 65, PO4 4.5
B) Calcium 9.5, PTH 35, PO4 4.0
C) Calcium 6.0 (8.5-10.5), PTH 2 (10-65), PO4 6.0 (2.7-4.5)
D) Calcium 10.8, PTH 108, PO4 2.3
E) Calcium 8.0, PTH 98, PO4 2.1

Answer: C
44. A 70-year-old man was referred by his GP with difficult to treat hypertension. He had long-
standing hypertension which had been well controlled over many years but recently he was
found to have a blood pressure of 190/110 mmHg which proved resistant to additional treatment.
He was generally asymptomatic and complied with medication. Investigations showed normal
U+Es. Which one of the following is the most likely cause?
A) Chronic pyelonephritis
B) Conn’s syndrome (primary hyperaldosteronism)
C) Phaeochromocytoma
D) Polycystic kidney disease
E) Renovascular disease

Answer: E

45. Regarding differentiated thyroid carcinoma the following statements are correct EXCEPT.
A) Radiation is a recognized risk factor.
B) Palpillary carcinoma is the most common type.
C) Fine needle aspiration biopsy is very sensitive for the diagnosis of minimally invasive
follicular carcinoma.
D) Serum thyroglobulin is a sensitive tumor marker for patients who underwent total
thyroidectomy and radioiodine remnant ablation

Answer: C

46. Risk factors for foot ulcers in patient with diabetes includes all the following EXCEPT:
A) Foot deformity.
B) Peripheral neouropathy.
C) Tobacco smoking.
D) Female sex.
E) Poor glycemic control.

Answer: D

47. A 51-year-old man is post-procedure day number 2 from an upper endoscopy and banding
for bleeding esophageal varices. The patient has a 7-year history of chronic active hepatitis and
over the past few years has developed stigmata associated with cirrhosis and worsening portal
hypertension. Three days ago, he presented to the emergency department with bright red blood
per mouth and rectum and a nasogastric tube evacuated bright red blood and coffee grounds from
the patient's stomach. He was admitted to the hospital, transfused with 2 units of red blood cells
and underwent an endoscopy. On preparation for the patient's discharge, you have a long
discussion with your patient about the course of events. In counseling this patient on his future
risks and course of therapy, you should advise him that:
A) The risk of rebleeding is between 50% and 80% and medical therapy is indicated
B) The risk of rebleeding is between 50% and 80% and surgical therapy is indicated
C) The risk of rebleeding is between 50% and 80%, but no therapy is indicated given the nature
of the problem
D) There is no concern for rebleeding
E) There is no concern for rebleeding, he is an imminent candidate for transplant

Answer: A

48. A 36-year-old woman complains of reflux symptoms and intermittent diarrhea. The diagnosis
of gastrinoma is suspected so a fasting serum gastrin is obtained and
found to be 280 pg/mL (normal <115 pg/mL). An abdominal CT is negative. What would you do
now?
A) Refer her for an exploratory laparotomy
B) Measure serum gastrin in response to secretin infusion
C) Treat her for H. pylori
D) Obtain a dedicated small bowel series
E) Measure gastric acid secretion

Answer: E

49. A phlebotomist sustains a needlestick injury whilst taking blood from a HIV-positive patient.
Which one of the following has been shown to most reduce the risk of HIV transmission?
A) Thorough immediate washing of the injury site with water
B) Oral lamivudine therapy for 1 month
C) Oral zidovudine therapy for 1 month
D) Oral ritonavir therapy for 1 month
E) Oral triple therapy for 1 month

Answer: E

50. A 60-year-old female has been on long-term treatment for rheumatoid arthritis. She presents
with dyspnoea on minimal exertion and a non-productive cough. Oxygen saturation on air is
84% and chest radiograph shows diffuse bilateral interstitial infiltrates. Extensive investigation
for an infective cause is negative. Which of the following drugs is most likely to be responsible?
A) Methotrexate
B) Penicillamine
C) Pednisolone
D) Cyclosporin
E) Hydroxychloroquine

Answer: A

MULTIPLE CHOICE QUESTIONS-10

1. Which one of the following is NOT a contraindication to intravenous (IV) tPA in patient with
ischemic stroke?
A) Age >75
B) INR >1.7
C) Platelets <100,000
D) Stroke within last 3 months
E) Glucose <50

Answer: A

2. A 28-year-old female presents to the emergency department with a severe headache and
lethargy. The headache came on fairly quickly whilst she was watching television. She is
otherwise fit and well, has no medical problems and takes no regular medications. She lives at
home with her husband and two-year-old daughter. Whilst in the emergency department she is
given two co-codamol for her persisting headache but then vomits. On examination, there is no
focal neurology but she is slightly drowsy and her GCS is 14/15. Hb 126 g/l; Platelets 274 *
109/l; WBC 10.9 * 109/l; Na+ 124 mmol/l; K+ 5.0 mmol/l; Urea 4.1 mmol/l; Creatinine 124
µmol/l. What is the most likely diagnosis?
A) Tension headache
B) Addison's disease
C) Pituitary apoplexy
D) Sub-arachnoid haemorrhage
E) Cranial diabetes insipidus

Answer: C

3. An 18-year-old woman with a microcytic anemia is diagnosed with alpha-thalassemia.


Formation of which of the following hemoglobins is increased in alpha-thalassemia?
A) H
B) A
C) F
D) A2
E) C

Answer: A

4. Which of the following is most characteristic of diabetic neuropathy?


A) it is usually bilateral
B) pain is not a feature
C) it most commonly affects the brain
D) it spares the autonomic system
E) it responds to meticulous control of blood glucose

Answer: A

5. A 19 year-old female is referred following a visit to the dentist where marked erosion of her
teeth was noted. She was entirely asymptomatic and her only medication was the oral
contraceptive pill. On examination her blood pressure was 110/70 mmHg and her body mass
index was 21.5 kg/m2 (18 - 25). Investigations sodium 135 mmol/l, potassium 2.1 mmol/l,
bicarbonate 42 mmol/l, urea 2.6 mmol/L corrected calcium 2.08 mmol/ alkaline phosphatase 201
iu/l (50-110) What is the most likely diagnosis?
A) Bulimia nervosa
B) Conn's syndrome
C) Laxative abuse
D) Pregnancy
E) Primary hypoparathyroidism

Answer: E

6. An 88-year-old man is taken to A&E by his daughter as he had been feeling unwell that day
with chest pains. An ECG reveals that he is suffering an inferior myocardial infarction. He has a
heart rate of 33 bpm and blood pressure of 90/60 mmHg. He is given 600mcg of atropine to no
effect, this is repeated up to 3mg. You commence an infusion of TPA but his rate is not
immediately improving. Which of the following treatments would be most suitable to use for his
arrhythmia?
A) Adrenaline
B) Glucagon
C) Temporary pacing
D) Dopamine
E) Dobutamine

Answer: C

7. A 55-year-old politician with no previous medical history is admitted to A&E. He has been
feeling unwell since Saturday morning and an ECG taken on admission to the Emergency
Department at 1am on the Monday morning shows atrial fibrillation at rate of 130 bpm with BP
150/80 mmHg. He is Echoed (trans-thoracic) and there is no evidence of the left atrial thrombus,
no abnormal ventricular wall movement and his ejection fraction is normal. What would be the
most appropriate initial treatment for this patient?
A) Anticoagulate with IV heparin and carry out DC cardioversion
B) Arrange a TOE next morning and if negative give 300 mg flecainide IV per day
C) Anticoagulate with warfarin and give 300 mg amiodarone IV
D) Start 300 mg amiodarone IV with no anticoagulation
E) Anticoagulate with LMWH and give 300 mg oral flecainide per day

Answer: E

8. A74-year-old woman has metastatic bone disease on x-ray. Which of the following mediators
is least likely to be involved?
A) interleukin-6 (IL-6)
B) ectopic parathyroid hormone (PTH)
C) tumor necrosis factor (TNF)
D) interleukin-1 (IL-1)
E) prostaglandins

Answer: B
9. The medication of choice to treat a patient in torsades de pointes is:
A) Epinephrine.
B) Flecainide.
C) Calcium gluconate
D) Magnesium sulfate.
E) Procainamide

Answer: D

10. A 43-year-old white male presents to the emergency room for intractable nausea and
vomiting. During inpatient admission paperwork, you complete his VTE risk assessment and
realize he is a high VTE risk. What would you choose for DVT prophylaxis (BMI = 45, CrCl =
75 ml/min)?
A) Early ambulation
B) Lovenox 40 mg SQ daily
C) Lovenox 40 mg SQ BID
D) Sequential compression device
E) Lovenox 100 mg SQ daily

Answer: C

11. A 61-year-old man has a non-ST-elevation MI and is admitted to the coronary care unit. The
following day, he develops bradycardia but no symptoms. His blood pressure is 126/84 mm Hg,
pulse 50/min, and on examination, the heart sounds are normal, with no extra sounds or rubs. His
ECG has changed. Which of the following ECG findings is the best indication for this patient to
receive a pacemaker?
A) persistent bradycardia
B) second-degree AV block Mobitz type I
C) first-degree AV block
D) new right bundle branch block
E) left bundle branch block (LBBB) and second-degree AV block Mobitz type II

Answer: E

12. A79-year-old man on quinidine for paroxysmal atrial fibrillation develops thrombocytopenia.
Which of the following is the most likely mechanism for this syndrome?
A) it is due to bone marrow suppression
B) it is due to sequestration in the spleen
C) it is due to intravascular destruction of platelets
D) it is immunologically mediated
E) there is cross-reactivity with penicillin

Answer: D

13. A 28-year-old man develops viridans group streptococci septicemia. Which of the following
cardiac lesions has the highest risk of developing endocarditis?
A) ventricular septal defect
B) atrial septal defect, secundum type
C) mitral valve prolapse with regurgitation
D) pure mitral stenosis
E) asymmetric septal hypertrophy

Answer: A

14. A 61-year-old man is taking over-the-counter pseudoephedrine for cold and flulike
symptoms. Over the course of the next few days, he experiences improvement in his rhinitis but
should be concerned about the possibility of which of the following problems?
A) Contraction of the iris causing visual changes
B) Constriction of the bronchioles causing increased pulmonary secretions
C) Erectile function improvement
D) Thinning of his salivary glandular secretions
E) Urinary retention

Answer: E

15. A 45-year-old woman develops symptoms of shortness of breath on exertion, easy fatigue,
and jaundice. On examination she is pale, and there is a palpable spleen, but no
lymphadenopathy. Her hemoglobin is 9.0 g/dL, the reticulocyte count 9%, and the direct
antibody test (Coombs’) is positive. Which of the following bone marrow findings is most likely
to be seen in this patient?
A) megaloblastic changes
B) giant metamyelocytes
C) increased erythroid-to-myeloid ratio
D) increased lymphocytes
E) shift to left of the myeloid series

Answer: C

16. A 68-year-old man presents with symptoms and signs of CHF. Which of the following is a
contraindication to use furosemide?
A) has hypoalbuminemia
B) is oliguric
C) has acidosis
D) had a rash with trimethoprimsulfamethoxazole
E) is on anticoagulants

Answer: D

17. Which of the following features of barbiturate induced coma is most likely correct?
A) requires at least 20–30 times the full sedative dose
B) is increasing in frequency
C) is characterized by an initial period of hyperventilation
D) causes death by depression of the cardiovascular system
E) causes death by pulmonary complications

Answer: E

18. A 62-year-old man presents with dark tarry stools and light-headedness. Upper endoscopy
finds an ulcerating lesion in his stomach and biopsies confirm gastric cancer. Which of the
following is a risk factor for carcinoma of the stomach?
A) Helicobacter pylori infection
B) high socioeconomic status
C) high protein diet
D) high alcohol consumption
E) high fat diet

Answer: A

19. An agitated and nervous 24-year-old woman has had severe wheezing and shortness of
breath for 2 days. After receiving oxygen, steroids, and salbutamol (Ventolin) in the emergency
room, her breathing improves. She is still wheezing and now feels tremulous and anxious with a
pulse of 110/min and respirations 30/min. Arterial blood gases on oxygen reveal a pH of 7.40,
PO2 340 mm Hg, PCO2 40 mm Hg, and bicarbonate of 24 mEq/L. She is hospitalized for further
treatment. Which of the following treatments or medications should be avoided in her?
A) theophylline
B) sedatives
C) corticosteroids
D) sympathomimetic amines
E) intravenous (IV) fluids

Answer: B

20. A 63-year-old man developed a transient episode of vertigo, slurred speech, diplopia, and
paresthesias. He is symptom-free now, and clinical examination is entirely normal. His past
medical history is significant for hypertension and dyslipidemia. Which of the following is the
most likely cause for symptoms?
A) posterior circulation transient ischemic attack (TIA)
B) anterior communicating artery aneurysm
C) hypertensive encephalopathy
D) pseudobulbar palsy
E) occlusion of the middle cerebral artery

Answer: A

21. A 19-year-old man is recently diagnosed with type 1 diabetes and attends your clinic to ask
about possible complications in the future. He mentions an uncle who has end-stage renal disease
due to poorly controlled diabetes and specifically inquires about testing for early signs of renal
impairment. The most appropriate investigation is:
A) Blood pressure
B) Microalbuminuria
C) Serum creatinine
D) Serum electrolytes
E) Urine dipstick for glucose

Answer: B

22. Which of the following features is least likely to be found in a patient with tuberous
sclerosis?
A) Adenoma sebaceum
B) Cafe-au-lait spots
C) Retinal hamartomas
D) 'Ash-leaf' spots
E) Lisch nodules

Answer: E

23. Which of the following statements is true about gastric lavage?


A) Except in extraordinary circumstances it should be done only in the first 1.5 hours after an
overdose.
B) Patients who have had gastric lavage have higher incidence of pulmonary aspiration than
patients who have not.
C) The maximum volume that should be used is 5 liters.
D) It can push pill fragments beyond the pylorus.
E) All of the above are true

Answer: E

24. Which ONE of the following is true regarding acute pulmonary embolism?
A) a normal ECG excludes the diagnosis
B) embolectomy is more effective than thrombolysis in improving survival
C) Heparin is as effective as thrombolytic therapy
D) the presence of hypoxaemia is an indication for thrombolysis
E) thrombolysis administered through a peripheral vein is as effective as through a pulmonary
artery catheter

Answer: E

25. A 32-year-old woman is admitted to the intensive care unit (ICU) for sepsis due to
pyelonephritis. Her serum creatinine is 0.5 mg/dL at time of admission. 24-h later her urine
output started to decline, but her creatinine remains stable. The intensivist feels that the patient is
in the process of developing acute kidney injury (AKI). Which one of the following has been
clinically validated as a biomarker for the early diagnosis of AKI?
A) Neutrophil gelatinase-associated lipocalin (NGAL)
B) N-acetyl-β-D-galactosaminidase (NAG)
C) Kidney injury molecule-1 (KIM-1)
D) Interleukin-18 (IL-18)
E) None of the above

Answer: E

26. A 51-year-old man with severe, recurrent, and extensive peptic ulceration is given a
diagnosis of Zollinger–Ellison syndrome. Which of the following is true with respect to this
syndrome?
A) Common cause of peptic ulcer disease
B) Never malignant
C) Should be treated by gastrectomy
D) Diarrhoea can be the presenting feature
E) Octreotide is contraindicated

Answer: D

27. For which of these overdoses is charcoal NOT indicated?


A) Acetaminophen
B) Aspirin
C) Iron
D) Digoxin
E) Opiates

Answer: C

28. Whichof the following antimicrobialsis associated with prolongation of the QT interval?
A) Coamoxiclav
B) Gentamicin
C) Cefuroxime
D) Erythromycin
E) Isoniazid

Answer: D

29. Whichof the following is associated with Hyperuricaemia?


A) is usually due to an excess purine consumption
B) occurs in association with acute lymphoblastic leukaemia
C) in primary gout is inherited in an autosomal dominant manner
D) can be reduced with low dose aspirin therapy
E) can be treated with uricosuric drugs even in renal failure

Answer: B

30. In an investigation for lung malignancy, all of the following may produce a false positive
result on a PET-CT except:
A) Pulmonary hamartoma
B) Intralobar sequestration
C) Tuberculosis
D) Pneumonia
E) Scarring

Answer: B

31. A 38-year-old woman presents with bilateral pitting ankle oedema. The jugular venous
pressure is not raised and the hepatojugular reflux was negative. The woman's ankle oedema
would not be explained by which of the following conditions?
A) Pelvic tumour
B) Amlodipine therapy
C) Cyclic oedema
D) Hypoalbuminaemia
E) Right heart failure

Answer: E

32. A 35-year-old healthy man was seen for an annual medical review as part of his company
employment policy. Examination reveals oral thrush in an otherwise fit and healthy man. Further
tests confirm oral candidiasis. He denies any illness and he is receiving no drugs. Which one of
the following is the most likely underlying disease?
A) Lymphoma
B) Alcoholism
C) HIV infection
D) Chronic renal failure
E) Heroin addiction

Answer: C

33. A 40-year-old Egyptian tourist is admitted with haematemesis. He is known to have chronic
liver disease caused by Schistosoma mansoni infection. Physical examination would reveal
which one of the following clinical signs?
A) Splenomegaly
B) Jaundice
C) Ascites
D) Spider nevi
E) Gynaecomastia

Answer: A

34. A 68-year-old man presents with recent onset headache and vomiting. Ophthalmoscopy
reveals optic atrophy in the right eye and papilloedema in the left one. The most likely diagnosis
is?
A) Pinealoma
B) Medulloblastoma
C) Acoustic neuroma
D) Sphenoid wing meningioma
E) Pituitary adenoma

Answer: D

35. A 62-year-old man presented with several non-healing ulcerated lesions on the lower
extremities and hand, present for approximately three years. He relates that each lesion began as
a tender erythematous papule that subsequently ulcerated. The lesions have failed to respond to
topical steroids and topical antifungal agents. He received skin grafts over the lower extremity
lesions one year ago but the grafts ulcerated. There are multiple ulcerate lesions bilaterally on the
lower extremities and one on the right hand. The lesions have dusky purple margins and a halo of
erythema. They range in size from 2.5 to 6 cm in diameter. The bases of the ulcers are verrucous
with regions of necrosis and granulation tissue. The most likely diagnosis is
A) Venous insufficiency
B) Pyoderma gangrenosum
C) Tropical ulcer
D) Bacterial skin infection
E) Ischaemic ulcers

Answer: B

36. A 50-year-old accountant is referred for investigation of fever and general ill health. His
initial renal function tests show: urea 45 mmol/I, creatinine 480 umol/l. Which one of the
following medication dose interval should be most prolonged when used in this patient?
A) Gentamicin
B) Amoxicillin
C) Vancomycin
D) Penicillin
E) Cefuroxime

Answer: C

37. A 58-year-old chronic alcoholic was admitted with increasing shortness of breath and a
distended abdomen. Physical examination revealed evidence for ascites and pitting oedema in
both legs. The pulse was 94 bpm, blood pressure 110/60 mmHg and the jugular venous pressure
was not raised. The condition least likely to be responsible for this clinical presentation is
A) Alcoholic cardiomyopathy
B) Liver cirrhosis
C) Bowel cancer with liver metastasis
D) Tuberculous peritonitis
E) Portal vein thrombosis

Answer: A
38. A 24-year-old male inpatient appears confused and is behaving in an unusual manner on the
ward. Nursing staff report a disrupted sleep pattern. The patient has no psychiatric history. You
wish to exclude an organic cause for his behaviour and order an electroencephalogram (EEG). In
which of the following disorders are EEG abnormalities least expected?
A) Herpes simplex encephalitis
B) Interictal psychosis
C) Metabolic encephalopathy
D) Hebephrenic schizophrenia
E) Subacute sclerosing panencephalitis

Answer: D

39. A 72-year-old woman is admitted to the Coronary Care Unit with an acute inferior
myocardial infarction. During ECG recording it is noticed that her heart rate is 40 bpm with
sinus rhythm and blood pressure is 87/55 mmHg. The most appropriate immediate action is
A) Keep monitoring the pulse and the blood pressure for a further 24 hours
B) Insert temporary pacemaker
C) Organise emergency percutaneous coronary angioplasty
D) Give 24-hour isoprenaline infusion
E) Give intravenous atropine sulphate (0.6 mg)

Answer: E

40. An 80-year-old man is referred from a nursing home with fever, rigors, nausea and vomiting.
He is bedridden and has sacral decubitus ulcers. Blood cultures grow Streptococcus milleri. The
most likely site of infection is?
A) Sacral bedsores
B) Urinary tract infection
C) Liver abscess
D) Endocarditis
E) Osteomyelitis

Answer: C

41. A 75-year-old lady presents with persistent diarrhoea and weight loss. Twenty years
previously she had radiotherapy for carcinoma of the ovary. Thirty years ago she lived in Hong
Kong for 18 months. Her father died of cancer of the colon aged 62 years.
Hb 9.4 g/dL (11.5-16 g/dL)
WCC 9.6 x 109/L (4-11 x 109/L)
Platelets 234 x 10/L (150-400 x 109/L)
B12 153 pmol/L (160-900pmol/L)
Red cell folate 21 nmol/L (130-630 nmol/L)
Ferritin 5 ug/L (20-250 ug/L)
Erythrocyte sedimentation rate (ESR) 36 mm/hour
Which of the following is the most likely diagnosis?
A) Radiation enteritis
B) Carcinoma of the colon
C) Crohn's disease
D) Tropical sprue
E) Acquired lactose intolerance

Answer: A

42. A 50-year-old male with a history of alcohol dependence and chronic hepatitis C presents to
your clinic with three months of fatigue, weakness and weight loss. He has also noted some
“yellowing of my eyes.” You suspect cirrhosis. All the following are clinical signs of cirrhosis
except
A) Arthralgia
B) Asterixis
C) Dupuytren’s contracture
D) Haemorrhoids
E) Testicular atrophy

Answer: A

43. A 64-year-old who suffers from haemochromatosis is seen in A&E with a 2-day history of
pain and swelling in his right knee. On examination it is swollen and he has decreased range of
movement. You aspirate his knee. What will the aspirate be most likely to show?
A) Gram-positive cocci in clusters
B) Positively bifringent crystals
C) Gram-positive cocci in chains
D) Inflammatory cells with haemosiderin deposition
E) Negatively bifringent crystals

Answer: B

44. A 76-year-old gentleman is referred with an international normalized ratio (INR) of 8.4. He
normally takes warfarin for a prosthetic mitral valve. He had an epistaxis lasting 15 minutes this
morning. He is otherwise asymptomatic and haemodynamically stable. Having stopped his
warfarin for tonight, Which of the following is the most appropriate course of action?
A) 5 mg iv vitamin K
B) 0.5 mg orally (po) vitamin K
C) 4 units of fresh frozen plasma iv
D) 50 units/kg of prothrombin complex concentrate
E) Recheck INR in 24 hours

Answer: E

45. Concerning adrenal physiology, which of the following hormones is secreted by the zona
glomerulosa?
A) Adrenaline
B) Noradrenaline
C) Cortisol
D) Aldosterone
E) Angiotensin II

Answer: D

46. A 33-year-old female is seen in the maternity department.she suffers from SLE. Her child
was born with profound bradycardia. Which of the following antibodies is most likely to be
present?
A) Anti-Ro
B) Anti-La
C) Anti-Jo
D) Anti-Scl-70
E) Antiphospholipid antibodies

Answer: A

47. A 67-year-old man presents with nausea and pain in his right upper quadrant. On
examination, he has tender hepatomegaly with an irregular edge. His blood tests are as follows:
Hb 8 g/dL (11.5-16 g/dL)
WCC 17.8 x 109/L (4-11 x 109/L)
Platelets 104 x 109/L (150-400x109/L)
Neutrophils 68%
Normoblasts 8%
Myeloblasts 8%
Myelocytes 5%
Metamyelocytes 5%
Lymphocytes 15%
What is the most likely cause of his anaemia?
A) Gaucher's disease
B) Leukaemoid reaction secondary to acute infection
C) Myelosclerosis
D) Malignant infiltration of bone marrow
E) Osteopetrosis

Answer: D

48. A 32-year-old woman presents with left loin pain and haematuria. She is known to suffer
with Crohn's disease.
Which of the following is likely to be the aetiology of this presentation?
A) Hypercalciuria
B) Hyperbilirubinaemia
C) Hyperuricaemia
D) Hyperoxaluria
E) Type 1 renal tubular acidosis
Answer: D

49. Regarding glucocorticoid-induced hyperglycemia all the following statements are correct
EXCEPT.
A) The risk of new onset diabetes mellitus is related to the total glucocorticoid dose and duration
of therapy.
B) Raised fasting plasma glucose is the earliest abnormality.
C) Reduced insulin sensitivity is the predominant mechanism.
D) Insulin is the most effective and safety therapy
E) Long term use of thiazolidinediones in combination with glucocorticoid is contraindicated.

Answer: B

50. A 65-year-old man has the following blood results on admission to hospital:
Na 132 mmol/L (135-145 mmol/L)
K 8.6 mmol/L (3.5-5 mmol/L)
Urea 42.4 mmol/L (2.5-6.7 mmol/L)
Creatinine 1178 um0l/L (70-150 umol/L)
The patient appears unwell and the ECG shows tented T waves, absent P waves and widened
QRS. Which of the following therapeutic interventions should be performed first?
A) Immediate haemodialysis
B) Intravenous insulin and dextrose
C) Intravenous calcium gluconate
D) Nebulized salbutamol
E) Oral calcium resonium

Answer: C

MULTIPLE CHOICE QUESTIONS-11

1. A 72-year-old male had the onset, within the last 60 minutes, of right hemiparesis and
expressive aphasia. Which of the following is the most important variable to monitor on this
patient while a decision is being made regarding whether to employ thrombolytic therapy?
A) WBC
B) Hemoglobin
C) Blood sugar
D) Serum electrolytes
E) Oxygenation status

Answer: E

2. A 29-year-old woman with a history of asthma presents with progressive worsening of


respiratory function. She reports that her symptoms have developed gradually over the past few
months, with occasional fevers and episodes of mild hemoptysis. She denies any weight loss,
skin changes, or diarrhea. Her medications include inhaled albuterol as needed and OCPs. She
has no family history of cancer, no recent travel, and no recent sick contacts. She works as an
accountant and does not smoke or use any illicit drugs. On examination, she has a low-grade
fever with scattered wheezes and rhonchi over bilateral lung fields. After coughing vigorously, a
brown mucus plug is expectorated. Her laboratory values show eosinophilia with an elevated
total serum IgE. A chest x-ray shows interstitial infiltrates in the upper lobes with some areas of
atelectasis bilaterally, and a CT scan shows enlarged airways primarily in the upper lobes with
bronchial wall thickening. Which of the following is the most likely diagnosis?
A) Tuberculosis
B) Worsening asthma
C) Strongyloides stercoralis
D) Allergic bronchopulmonary aspergillosis
E) Bronchial carcinoid tumor

Answer: D

3. All of the following are associated with increased levels of total T4 in the plasma with a
normal free T4 EXCEPT:
A) Cirrhosis
B) Pregnancy
C) Sick-euthyroid syndrome
D) Familial dysalbuminemic hyperthyroxinemia
E) Familial excess thyroid-binding globulin

Answer: C

4. A 49-year-old female with a 5-year history of diabetes mellitus type 2 presents for an initial
visit. She has no known complications of diabetes. She takes metformin, glyburide, andaspirin.
On examination, you find a pleasant, obese female in no distress. Her blood pressure is 136/86
mm Hg, pulse 86, respirations 14, and temperature 37°C. As you discuss monitoring her diabetes,
you recommend screening for early kidney disease. Which of the following approaches is the
recommended way to screen for diabetic kidney disease?
A) Obtain a 24-hour urine collection for albumin now and again in 3 years.
B) Obtain a spot urine microalbumin every year.
C) Obtain a spot urine microalbumin/creatinine ratio every year.
D) Obtain a urinalysis every year.
E) Obtain a serum creatinine every year.

Answer: C

5. Risk factors for foot ulcers in patient with diabetes include all the following EXCEPT:
A) Foot deformity.
B) Peripheral neouropathy.
C) Tobacco smoking.
D) Female sex.
E) Poor glycemic control.

Answer: D
6. A patient with ascites is suspected to have secondary hyperaldosteronism. Which one of the
following would be typical levels of electrolytes in an aliquot specimen of urine?
A) Sodium 2 mEq/L, potassium 40 mEq/L
B) Sodium 5 mEq/L, potassium 0 mEq/L
C) Sodium 40 mEq/L, potassium 40 mEq/L
D) Sodium 80 mEq/L, potassium 2 mEq/L
E) Sodium 100 mEq/L, potassium 20 mEq/L

Answer: A

7. Which of the following conditions should be considered if both the PT and PTT are prolonged
in a patient noted to be oozing from a surgical incision?
A) Severe liver disease, DIC, factor X deficiency.
B) Heparin effect, von Willebrand disease, factor XII deficiency.
C) Warfarin effect, factor VII deficiency, vitamin K deficiency.
D) All of the above.

Answer: A

8. A 27-year-old African-American woman with diabetes presents to the outpatient Neurology


clinic with subacute onset of bilateral facial weakness. She explains that the symptoms have
developed over the course of the last few days. She also reports having a slightly raised and
tender rash over the anterior aspects of both shins. On examination you find bilateral lower
motor neuron facial palsy as well as tender erythematous nodules over both shins. Which of the
following is the most likely diagnosis?
A) Guillain-Barré syndrome
B) Lyme disease
C) Neurosarcoidosis
D) Diabetes
E) Tuberculosis

Answer: C

9. Which of the following are the cardinal features of idiopathic Parkinson disease?
A) Tremor, bradykinesia, rigidity, and postural instability
B) Bradykinesia, dementia, tremor, and rigidity
C) Rigidity, hallucinations, tremor, and postural instability
D) Tremor, rigidity, bradykinesia, and gaze palsy
E) Tremor, autonomic dysfunction, bradykinesia, and rigidity

Answer: A

10. According to the Eighth Joint National Committee (JNC-8) guideline on managing
hypertension in adults, Initial Drugs of Choice for Hypertension include all of the following
EXCEPT:
A) ACE inhibitor
B) Angiotensin receptor blocker
C) Thiazide diuretic
D) Calcium channel blocker
E) β-blockers

Answer: E

11. A 42-year-old male who works in a hog confinement area presents to your office
complaining of cough, fever, wheeze, and dyspnea. He and some other workers were cleaning
the confinement area with high-pressure hoses (which aerosolized hog waste), and they all with
the same symptoms, which started between 4 and 8 hours after work. On examination, he is
febrile with a respiratory rate of 28. He is able to talk in complete sentences. There are slight
crackles when you auscultate the lungs. His chest x-ray is normal. The most likely diagnosis is:
A) “Farmer's lung” (hypersensitivity pneumonitis).
B) Organic dust toxicity syndrome.
C) Reactive airway disease.
D) Hydrogen sulfide poisoning.
E) Bronchiolitis obliterans.

Answer: B

12. Which of the following statements best describes Graves’ ophthalmopathy?


A) Although a cosmetic problem, Graves’ ophthalmopathy is rarely associated with major ocular
complications.
B) Diplopia may occur from periorbital muscle swelling.
C) It is never found without concomitant hyperthyroidism.
D) The most serious complication is corneal abrasion.
E) Unilateral disease is not found.

Answer: B

13. A 20-year-old woman with no significant past medical history presents with a 2-month
history of episodic shortness of breath. These symptoms began with an upper respiratory tract
infection. She has fits of coughing and trouble catching her breath with exertion. She states that
her breath “sounds like whistles” at times. She tried a friend's albuterol inhaler with some
improvement and wonders if she has asthma. On exam, she is breathing comfortably at 16 times
per minute and her oxygen saturation is 96% on room air. Her lungs are clear to auscultation, and
the remainder of her exam is unremarkable. You want to better categorize this patient's disease.
Which of the following tests is most appropriate to order now?
A) Spirometry.
B) Chest x-ray.
C) Arterial blood gas (ABG).
D) Methacholine challenge.
E) Chest CT.

Answer: A

14. Ataxia may be a manifestation of which vitamin deficiency?


A) Vitamin A
B) Vitamin B
C) Vitamin C
D) Vitamin D
E) Vitamin E

Answer: E

15. A 34-year-old woman with diabetes mellitus, hypertension, dyslipidemia, a family history of
premature coronary artery disease, and ischemic cardiomyopathy with an ejection fraction of
38% by recent radionuclide angiography is contemplating pregnancy and seeks your advice. She
is completely asymptomatic and physical examination demonstrates no evidence of
hypervolemia. She is taking low-dose aspirin, a beta-blocker, an ACE inhibitor, and a
hydroxymethylglutaryl-coenzyme A reductase inhibitor (statin). Which of the following is the
most appropriate statement?
A) Advise the patient to discontinue the ACE inhibitor and pursue pregnancy
B) Advise the patient to discontinue the ACE inhibitor and statin, and pursue pregnancy
C) Advise the patient to discontinue the aspirin and ACE inhibitor and pursue pregnancy
D) Advise the patient to discontinue the beta-blocker and statin and pursue pregnancy
E) Advise the patient not to become pregnant.

Answer: E

16. An asymptomatic 55-year-old male visits a health fair, where he has a panel of blood tests
done. He brings the results to you because he is concerned about the TSH level of 12.0 µU/mL
(N 0.45-4.5). His free T4 level is normal. Which one of the following is most likely to be
associated with this finding?
A) Atrial fibrillation
B) Reduced bone density
C) Systolic heart failure
D) Elevated LDL cholesterol
E) Type 2 diabetes mellitus.

Answer: D

17. An 18-year-old man with a 12-year history of type 1 diabetes mellitus comes to the physician
for a follow-up examination. Medications include 25 U of NPH insulin and 10 U of regular
insulin in the morning and 10 U of NPH insulin and 10 U of regular insulin before dinner. His
hemoglobin A1c was 14.5% 12 weeks ago. His current pulse is 80/min, respirations are 20/min,
and blood pressure is 145/95 mm Hg. Examination shows scattered retinal microaneurysms
bilaterally. The remainder of the examination shows no other abnormalities. Laboratory studies
show:
Hemoglobin A1c 13%
Serum Na+ 130 mEq/L
K+ 3.2 mEq/L
Cl− 101 mEq/L
HCO3− 23 mEq/L
Glucose 325 mg/dL
Creatinine 1.5 mg/dL
Cholesterol 350 mg/dL
Urine Blood negative
Glucose 4+
Protein 1+
Ketones Negative
Which of the following is the most likely renal diagnosis?
A) Cholesterol renal emboli
B) Diabetic nephropathy
C) Hypertensive glomerulosclerosis
D) Hypokalemic nephropathy
E) Sodium-losing nephropathy.

Answer: B

18. Which of the following is increased in patients with emphysema- dominant COPD?
A) DLCO
B) FVC
C) FEV1
D) RV
E) Alpha-1 antitrypsin

Answer: D

19. An 18-year-old man with a childhood diagnosis of small bowel Crohn’s disease attended the
eye hospital Emergency Department with a painless, erythematous right eye. His vision was
reported to be unchanged. What is the most likely diagnosis?
A) Anterior uveitis
B) Episcleritis
C) Intermediate uveitis
D) Posterior uveitis
E) Scleritis.

Answer: B

20. A 44-year-old woman undergoes preoperative evaluation prior to surgery to repair a


congenital defect of her pelvis. Her expected blood loss is 2.0 l. She has a prior history of severe
anaphylactic reaction to a prior erythrocyte transfusion that she received for postpartum
hemorrhage at age of 27 years. In addition she has a history of rheumatoid arthritis. On physical
examination, the temperature is 36.8 °C (98.5 °F), blood pressure is 140/70 mm Hg, and heart
rate is 76 bpm. Laboratory studies indicate a hemoglobin level of 12.0 g/dL, a leukocyte count of
6500 μL, and a platelet count of 150,000 μL. Previous laboratory studies indicate an IgG level of
800 mg/dL and an IgM level of 65 mg/dL. Which of the following is the most appropriate
erythrocyte transfusion product for this patient?
A) Leuko-reduced blood
B) Cytomegalovirus-negative blood
C) Irradiated blood
D) Phenotypically matched blood
E) Washed blood

Answer: E

21. A 75-year-old man underwent surgery to correct a large abdominal aortic aneurysm. The
procedure appeared to go well, but you are called a few hours later to evaluate the patient who
states that he cannot move or feel his legs. On the way to the ICU, you consider the possible
causes of his symptoms and plan your physical examination. What is the most important test to
help localize the lesion?
A) MRI of the spine
B) Sensory level
C) Reflexes in lower extremities
D) Plantar flexion reflex
E) Toe position sense.

Answer: E

22. A 35-year-old man is suspected of having a small right-sided pleural effusion. What imaging
modality is most sensitive to detect a small amount of pleural fluid?
A) chest CT
B) lateral chest film
C) left lateral decubitus chest film
D) standard upright chest film

Answer: A

23. A 70-year-old man with history of HTN and DM presents with a 2-month history of
increasing paroxysmal nocturnal dyspnea and SOB with minimal exertion. An echocardiogram
shows an ejection fraction of 25%. Which one of the patients current medications should be
discontinued?
A) Lisinopril
B) Pioglitazone
C) Glipizide
D) Metoprolol
Answer: B

24. A 40-year-old male is diagnosed with active TB. In educating the pt, he is told to avoid
tyramine containing foods and the possibility of the development of hepatitis and peripheral
neuropathy. Which of the following medication is the pt being educated about?
A) rifampin
B) Isoniazid
C) Pyrazinamide
D) ethambutol
E) streptomycin

Answer: B

25. A 32-year-old man with family history of HTN measures his own BP daily and keeps in his
diary. He visits his primary care physician, and his BP is found to be 124/72 mm Hg, which is
much lower than his home BP. He insists that his BP be measured 2 h later, which was found to
be 125/73 mm Hg. He asks you to clarify about this discrepancy between office and home BP
measurements, and you would say that he has:
A) Essential HTN
B) Prehypertension
C) White coat HTN
D) Masked HTN
E) Secondary HTN

Answer: D

26. An 88-year-old man is taken to A&E by his daughter as he had been feeling unwell that day
with chest pains. An ECG reveals that he is suffering an inferior myocardial infarction. He has a
heart rate of 33 bpm and blood pressure of 90/60 mmHg. He is given 600mcg of atropine to no
effect, this is repeated up to 3mg. You commence an infusion of TPA but his rate is not
immediately improving. Which of the following treatments would be most suitable to use for his
arrhythmia?
A) Adrenaline
B) Glucagon
C) Temporary pacing
D) Dopamine
C) Dobutamine

Answer: C

27. All of the following are common manifestations of cytomegalovirus (CMV) infection
following lung transplantation EXCEPT:
A) bronchiolitis obliterans
B) CMV esophagitis
C) CMV pneumonia
D) CMV retinitis
E) CMV syndrome (fever, malaise, cytopenias, transaminitis, and CMV viremia)

Answer: D

28. A 25-year-old man presents to the ED with a broad complex tachycardia that is irregularly
irregular. The patient is haemodynamically uncompromised. An anaesthetist is not available to
assist with immediate DC cardioversion. What is the best initial treatment option?
A) IV adenosine
B) IV verapamil
C) Oral beta-blocker
D) IV beta-blocker
E) IV flecainide

Answer: E

29. A young patient is diagnosed with multiple endocrine neoplasia (MEN) type 3 (also known
as type 2b) after an episode of bowel obstruction. Which one of the following features would he
be unlikely to have or develop in the future with this diagnosis?
A) Medullary carcinoma of the thyroid
B) Marfanoid appearance
C) Mucosal neuromas of the small bowel
D) Facial angiofibromas
E) Prognathism

Answer: D

30. All the following types of cancer commonly metastasize to the central nervous system (CNS)
EXCEPT:
A) ovarian
B) breast
C) hypernephroma
D) melanoma
E) acute lymphoblastic leukemia (ALL)

Answer: A

31. A 32-year-old woman presents with diarrhea, with mucus and blood in the stool, and
recurrent abdominal pain. Colonoscopy is performed and a mucosal biopsy is obtained. Which
one of the following pathological findings would make ulcerative colitis a more likely diagnosis
than Crohn's disease?
A) Ileal involvement
B) Crypt abscesses
C) Transmural involvement
D) Granulomas
E) Skip lesions
Answer: B

32. A 61-year-old man is admitted because of increasing confusion and inadequately controlled
lower backache. He is found to have multiple myeloma. Which of the following statements
regarding this patient's disease is correct?
A) Osteoporotic vertebral collapse fracture may be responsible for his backache
B) Radiotherapy is ineffective in treating bone pain
C) Isotope bone scanning is more sensitive for identifying early disease
D) Plain radiographs of the spine would show typical widespread osteoblastic bone lesions
E) The hypercalcaemia is usually steroid-resistant

Answer: A

33. A 64-year-old builder who had a surgical operation for inguinal hernia a develops a wound
infection a week later and methicillin-resistant Staphylococcus aureus (MRSA) is recovered
from the wound. Which one of the following isolation techniques do you think is appropriate in
this condition?
A) Enteric precautions
B) Blood and body fluid precautions
C) Respiratory isolation only
D) Strict isolation
E) Contact isolation

Answer: E

34. A teacher has brought his son to you because he is very worried that his son might be
suffering with haemophilia. Which one of the following best excludes a diagnosis of
haemophilia?
A) Normal activated partial thromboplastin time
B) Normal prothrombin time
C) Absence of excessive bleeding after tooth extraction
D) Absence of spontaneous bleeding episodes
E) Absence of family history

Answer: C

35. A 20-year-old car mechanic who has been abusing inhalants for many years was admitted
with sudden collapse. The ambulance crew said he was inhaling solvent by rebreathing from a
plastic bag. He is transferred to the Intensive Care Unit. A few minutes later he stops breathing
and the cardiac monitor records ventricular fibrillation rhythm. In resuscitating this patient which
one of the following should be avoided?
A) Direct current shock
B) Intubation
C) Oxygen therapy
D) Intravenous ß-blocker
E) Intravenous adrenaline (epinephrine)
Answer: E

36. An 88-year-old woman is referred because of a tense bullous eruption on the inner sides of
the upper and lower limbs as well as the abdomen. The most likely diagnosis is
A) Pemphigus
B) Bullous pemphigoid
C) Herpes zoster
D) Dermatitis herpetiformis
E) Stevens-Johnson syndrome

Answer: B

37. A 40-year-old male dancer presents with thrombophlebitis of the lower limb. Examination
reveals five painless, punched-out ulcers on the scrotum with concomitant oral ulcers. Which of
the following is the most likely diagnosis?
A) Granuloma inguinale
B) Syphilis
C) Herpes simplex genitalis
D) Belicet's disease
E) Systemic lupus erythematosus

Answer: D

38. A 50-year-old woman is admitted with high fever and a generalised headache. Examination
reveals chemosis and cyanosis of the upper face. There was proptosis and ophthalmoplegia of the
left eye. Pin prick sensation was lost from the left forehead. The most likely diagnosis is
A) Periorbital cellulitis
B) Osteomyelitis of the left maxillary sinus
C) Left retro-orbital tumour
D) Cavernous sinus thrombosis
E) Superior sagittal sinus thrombosis

Answer: D

39. A 59-year-old lady is being investigated for haematuria. A diagnosis of glomerulonephritis is


suspected. Which one of the following tests is not indicated?
A) X-ray kidney, ureter, bladder (KUB).
B) Urine microscopy.
C) Serum and urine immunoelectrophoresis.
D) Complement levels.
E) Antineutrophil cytoplasmic (ANCA) and antiglomerular basement membrane (anti-GM)
antibodies.

Answer: A
40. Which of the following is incorrect regarding specific treatment of overdose?
A) Naloxone for opiate overdose is often given as an infusion due to its short half-life.
B) Flumazenil can be given for benzodiazepine overdose.
C) N-acetylcysteine for paracetamol overdose can only be given intravenously.
D) Oral methionine is effective in aspirin overdose.
E) ’Digibind’ is a fragment of digoxin-specific antibody given in severe overdose.

Answer: D

41. A 55-year-old man with chronic hepatitis C is being considered for liver transplantation. The
patient has cirrhosis that was documented by liver biopsy 10 years ago. For the past 3 months, he
has had ascites and edema, which are poorly controlled with diuretics. Lactulose was recently
begun because of confusion. Which of the following combinations of laboratory studies will be
most helpful in estimating his survival over the next 6 months?
A) Serum total bilirubin and INR
B) Serum aspartate aminotransferase and gamma globulin
C) Serum alanine aminotransferase and hepatitis C RNA (HCV RNA)
D) Serum alkaline phosphatase and ammonia
E) Serum albumin and ?-glutamyltransferase

Answer: A

42. Signs and symptoms of opioid withdrawal include all of the following EXCEPT
A) Increased blood pressure (BP) and heart rate
B) Seizures
C) Abdominal cramps
D) Jerking of the legs
E) Hyperthermia

Answer: B

43. Which of the following β-adrenergic antagonists is a nonselective β1 and β2 blocker?


A) Atenolol
B) Betaxolol
C) Esmolol
D) Metoprolol
E) Nadolol

Answer: E

44. In primary hypothyroidism increased requirements for L thyroxine occur in the following
conditions EXCEPT:
A) Nephrotic syndrome with massive proteinuria.
B) Pregnancy.
C) Achalasia
D) Helicobacter pylori associated chronic gastritis.
E) Use of estrogen replacement therapy.

Answer: C

45. A 70-year-old man is admitted with new-onset atrial fibrillation with an apical rate of
120/minute and a BP of 100/70 mm Hg. Shortly after admission he develops moderate lower
abdominal pain and diarrhea with dark red blood.
Which is the most appropriate course of action?
A) Cardioversion
B) Unfractionated heparin
C) Mesenteric angiography
D) Laparotomy
E) Digoxin

Answer: A

46. An 18-year-old insulin-dependent diabetic is seen in A&E. Over the last 4 days she has
suffered from diarrhoea and vomitting, and has cut down on her insulin injections. Her mother
has brought her in because she has become drowsy and confused. Her blood results are as
follows:
WCC 16.4 X109/L (4-11X109/L)
Hb 17.8 * g/dL (12-16 g/dL)
Platelets 597 X10/L (150-400X109/L)
Na 146 mmpl/L (135-145 mmol/L)
K 6.6 mmol/L (3.5-5 mmol/L)
Urea 8.4 mmol/L (2.5-6.7 mmol/L)
Creatinine 145 umol/L (60-120 umol/L)
Random glucose 26 mmol/L (4.5-5.6 mmol/L)
You are with one nurse in the resuscitation room. What is the first thing you are going to ask her
to do?
A) Start an insulin sliding scale
B) Give 1 L of normal saline over 30 minutes
C) Run to the lab with blood gases
D) Draw up 10 mL of 10% calcium gluconate
E) Give 100 mmol bicarbonate intravenously (iv)

Answer: B

47. A 73-year-old male presents to the clinic with 3 months


of increasing back pain. He localizes the pain to the lumbar
spine and states that the pain is worst at night while he is lying
in bed. It is improved during the day with mobilization.
Past history is notable only for hypertension and remote cigarette smoking. Physical examination
is normal. Laboratory
studies are notable for an elevated alkaline phosphatase. A
lumbar radiogram shows a lytic lesion in the L3 vertebra.
Which of the following malignancies is most likely?
A) Gastric carcinoma
B) Non-small cell lung cancer
C) Osteosarcoma
D) Pancreatic carcinoma
E) Thyroid carcinoma

Answer: B

48. A 36-year-old African-American woman with systemic lupus erythematosus presents with
the acute onset of lethargy and jaundice. On initial evaluation, she is tachycardic, hypotensive,
appears pale, is dyspneic, and is somewhat difficult to arouse. Physical examination reveals
splenomegaly.
Her initial hemoglobin is 6 g/dL, white blood cell count is 6300/μL, and platelets are 294,000/
μL. Her total bilirubin is 4 g/dL, reticulocyte count is 18%, and haptoglobin is not detectable.
Renal function is normal, as is urinalysis. What would you expect on her peripheral blood smear?
A) Macrocytosis and PMN’s with hypersegmented nuclei
B) Microspherocytes
C) Schistocytes
D) Sickle cells
E) Target cells

Answer: B

49. A 22 year-old lady presents to her GP with facial swelling, most marked in the morning, and
shortness of breath. Urinalysis shows protein +++. Results were as follows:
24-hour urinary protein 4.2 g;
Haemoglobin(Hb) 10.8 g/l (11.5-16.0 g/L);
Urea 22.8 mmol/L (2.5-6.5 mmol/L);
Creatinine 374 µmol/L (70-150 umol/L)
C3 0.24 g/L (0.55-1.2g/L)
C4 0.29g/L (0.2-0.5 g/L)
Which of the following is the most likely diagnosis?
A) Minimal change nephropathy
B) Systemic lupus erythematosus
C) Rheumatoid arthritis
D) Cryoglobulinaemia
E) Diabetes mellitus

Answer: B

50. A 31-year-old male with hemophilia A is admitted with persistent gross hematuria. He denies
recent trauma or any history of genitourinary pathology. The examination is unremarkable.
Hematocrit is 28%. All the following are treatments for hemophilia A except
A) desmopressin (DDAVP)
B) fresh-frozen plasma (FFP)
C) cryoprecipitate
D) recombinant factor VIII
E) plasmapheresis

Answer: E

Das könnte Ihnen auch gefallen